N316B Pharmacology Midterm

Réussis tes devoirs et examens dès maintenant avec Quizwiz!

305) Which body system is affected by excessive use of amphetamine/dextroamphetamine? Renal Cardiac Musculoskeletal Gastrointestinal

Cardiac rationale Norepinephrine is released when amphetamine/dextroamphetamine is taken. Excessive doses of amphetamine/dextroamphetamine can cause an increase in heart rate and dysrhythmias. Amphetamine/dextroamphetamine overuse does not directly affect the renal, musculoskeletal, or the gastrointestinal systems. Test-Taking Tip: What happens if you find yourself in a slump over the examination? Take a time-out to refocus and reenergize! Talk to friends and family who support your efforts in achieving one of your major accomplishments in life. This effort will help you regain confidence in yourself and get you back on track toward the realization of your long-anticipated goal.

54. 54.ID: 20127755909 A client is receiving methylprednisolone 40 mg IV daily. The nurse should monitor which laboratory value closely? A. Serum glucose. Correct B. Serum calcium. C. Red blood cells. D. Serum potassium.

Methylprednisolone is a corticosteroid with glucocorticoid and mineralocorticoid actions. These effects can lead to hyperglycemia (must monitor closely), which is reflected as an increase in the serum glucose value. The client taking methylprednisolone is also at risk for hypocalcemia and hypokalemia (but not severely). These medications also alter the some of the body's immune responses by suppressing the migration of white blood cells decreasing inflammation response.

25. 25.ID: 20127755997 After abdominal surgery, a client is prescribed low molecular weight heparin (LMWH). During administration of the medication, the client asks the nurse the reason for the medication. Which is the best response for the nurse to provide the client? A. This medication is given to prevent blood clot formation. Correct B. This medication enhances antibiotics to prevent infection. C. This medication dissolves clots that develop in the legs. D. This medication enhances the healing of wounds.

Unfractionated heparin or low molecular weight heparin (LMWH) is an anticoagulant that inhibits thrombin-mediated conversion of fibrinogen to fibrin and is given prophylactically to prevent postoperative venous thrombosis in order to prevent pulmonary embolism or deep vein thrombosis following knee and abdominal surgeries.

64. 64.ID: 20127754579 A client being discharged is prescribed warfarin for the treatment following a pulmonary embolism. Which diagnostic test should the nurse instruct the client to receive once a month? A. Perfusion scan. B. Prothrombin Time (PT). Correct C. Activated partial thromboplastin (aPTT). D. Serum Coumadin level (SCL).

When used for a client with pulmonary embolus, the therapeutic goal for warfarin therapy is a PT 1 to 2 times greater than the control, or an INR of 2 to 3. A client prescribed warfarin should have the PT or INR levels checked at a minimum once a month.

358) Which statement accurately describes nortriptyline? Select all that apply. One, some, or all responses may be . Overdosage is often lethal. Constipation and urinary retention may occur. It is a selective serotonin reuptake inhibitor (SSRI). Weight gain is a common side effect. It increases effectiveness of monoamine oxidase inhibitors (MAOIs).

-Overdosage is often lethal -constipation and urinary retention may occur -weight gain is a common side effect rationale Nortriptyline is notoriously lethal; between 70% and 80% of people who die from overdose do so before reaching the hospital. Nortriptyline can cause constipation and urinary retention because it causes blockage of cholinergic receptors, and it tends to increase appetite and cause weight gain. Nortriptyline is not an SSRI; it is a tricyclic antidepressant and affects the balance of neurotransmitters in the brain. The interaction between nortriptyline and MAOIs is severe and possibly fatal.

68) Nortriptyline is prescribed for a depressed client. Which time period identifies when the nurse would expect a therapeutic response? 1 to 3 days 12 to 24 hours 30 minutes to 2 hours 2 to 3 weeks

2-3 weeks rationale As with other tricyclics, optimal therapeutic effects take 2 to 3 weeks to occur. One to 3 days, 12 to 24 hours, and 30 minutes to 2 hours are all too soon to expect a response to nortriptyline. Test-Taking Tip: Be aware that information from previously asked questions may help you respond to other examination questions.

33. 33.ID: 20127755972 Which change in data indicates to the nurse that the desired effect of the angiotensin II receptor antagonist valsartan has been achieved? A. Dependent edema reduced from

3 to +1. B. Serum HDL increased from 35 to 55 mg/dL. C. Pulse rate reduced from 150 to 90 beats/minute. D. Blood pressure reduced from 160/90 mmHg to 130/80 mmHg. Valsartan is an angiotensin receptor blocker, prescribed for the treatment of hypertension. The desired effect is a decrease in blood pressure. Awarded 0.0 points out of 1.0 possible points.

374) Which action would the nurse perform when beginning pyridostigmine bromide therapy for a client with myasthenia gravis? Administer the medication after meals. Administer the medication on an empty stomach. Evaluate the client's psychological responses between medication doses. Evaluate the client's muscle strength every hour after the medication is given.

Evaluate the client's muscle strength every hour after the medication is given rationale The onset of action of pyridostigmine is 30 to 45 minutes after administration, and the effects last up to 6 hours; the client's response will influence dosage levels. Pyridostigmine usually is administered before meals to promote mastication. Pyridostigmine should be administered with food to prevent gastric irritation. There are no psychological side effects associated with pyridostigmine.

116) A 65-year-old client is receiving amitriptyline. Which recommendation will the nurse make to the client concerning this medication? 'Obtain a complete cholesterol and lipid profile.' 'Have an eye examination to check for glaucoma.' 'Check your temperature daily for nighttime increases.' 'Watch for excessive sweating and possible weight loss.'

Have an eye exam to check for glaucoma rationale In addition to baseline laboratory tests, an older adult should have an eye examination with glaucoma testing when taking amitriptyline. It causes dilation of the pupil (mydriasis), which interferes with drainage of aqueous humor through the canal of Schlemm. Interfering with the outflow of aqueous humor will increase intraocular pressure and may cause a progressive loss of vision in clients with glaucoma. Amitriptyline does not affect cholesterol production or temperature regulation. It does not cause excessive sweating or weight loss, but it can increase appetite especially for sweets, resulting in weight gain.

346) Which nursing action is appropriate when administering imipramine? Telling the client steroids will not be prescribed Warning the client not to eat cheese Monitoring the client for increased tolerance Having the client checked for increased intraocular pressure

Having the client checked for increased intraocular pressure rationale Glaucoma is one of the side effects of imipramine, and the client should be taught about the symptoms. The prescribing of steroids and avoiding cheese are true of monoamine oxidase inhibitors (MAOIs); imipramine is not an MAOI. Tolerance is not an issue with tricyclic antidepressants such as imipramine. Test-Taking Tip: The most reliable way to ensure that you select the response to a multiple-choice question is to recall it. Depend on your learning and memory to furnish the answer to the question. To do this, read the stem and then stop! Do not look at the response options yet. Try to recall what you know and, based on this, what you would give as the answer. After you have taken a few seconds to do this, then look at all of the choices and select the one that most nearly matches the answer you recalled. It is important that you consider all the choices and not just choose the first option that seems to fit the answer you recall. Remember the distractors. The second choice may look okay, but the fourth choice may be worded in a way that makes it a slightly better choice. If you do not weigh all the choices, you are not maximizing your chances of ly answering each question.

5) An 11-year-old client reports having bedwetting issues (enuresis). Which medication would the nurse anticipate when developing a teaching plan ? Alprazolam Imipramine Lithium salts Clomipramine

Imipramine rationale Certain conditions of pediatric clients necessitate the usage of tricyclic antidepressant medications as an adjuvant. Childhood enuresis is one such condition that necessitates the administration of imipramine. Alprazolam is the medication of choice for treating anxiety disorders. Lithium salt is prescribed to treat bipolar disorders. Clomipramine is a tricyclic antidepressant medication prescribed for treating obsessive-compulsive disorder. Test-Taking Tip: Identifying content and what is being asked about that content is critical to your choosing the response. Be alert for words in the stem of the item that are the same or similar in nature to those in one or two of the options .

92) An 80-year-old client with depression requires the prescription of antidepressant medication. Which tricyclic antidepressant medication causes fewer complications in older clients? Doxepin Amoxapine Nortriptyline Trimipramine

Nortriptyline rationale Nortriptyline and desipramine are preferred for use in older adults because these antidepressant medications have less anticholinergic activity. Doxepin, amoxapine, and trimipramine have more cholinergic activity than nortriptyline and are not the preferred medications for older adult clients.

688) A client is given a prescription for bumetanide. The nurse will teach the client to watch for symptoms of which condition? Hypokalemia Hyperchloremia Hypernatremia Hypoglycemia

Rationale Bumetanide is a loop diuretic. Diuretic therapy that affects the loop of Henle increases urinary excretion of sodium, chloride, and potassium. As a result, clients are at risk for hypokalemia, hyponatremia, and hypochloremia. Additionally, hyperglycemia can occur.

1114) A client is scheduled for a bilateral adrenalectomy. Which

Rationale describes why steroids are administered to the client? To foster accumulation of glycogen in the liver To increase the inflammatory action to promote healing To facilitate urinary excretion of salt and water after surgery To compensate for sudden lack of these hormones after surgery +Rationale Adrenal steroids help an individual adjust to stress. Unless received from external sources, there is no hormone available to cope with surgical stresses after an adrenalectomy. Glucose stores (glycogen) will be used by the body to adapt to surgery. Insulin is the hormone that facilitates conversion of glucose to glycogen. Steroids do not increase inflammatory reactions. Steroids will result in fluid retention, not loss.

226) Which drug category applies to cocaine? Opioids Stimulants Barbiturates Hallucinogens

Stimulants rationale Cocaine is classified as a stimulant. It is inhaled in its powdered form or smoked as crack; its use creates experiences similar to but more intense than those experienced with amphetamines, and its withdrawal results in a deeper crash. Opioids and barbiturates are central nervous system depressants. Hallucinogens produce cerebral excitation that can yield a state similar to psychosis.

4) A client develops extrapyramidal effects after taking a neuroleptic medication, and the nurse notes extrapyramidal effects. Which medication can limit these side effects? Zolpidem Hydroxyzine Dantrolene Benztropine mesylate

benztropine mesylate rationale Benztropine mesylate, an anticholinergic, helps balance neurotransmitter activity in the central nervous system (CNS) and helps control extrapyramidal tract symptoms. Zolpidem is a sedative-hypnotic medication used for short-term insomnia. Hydroxyzine is a sedative that depresses activity in the subcortical areas in the CNS; it is used to reduce anxiety. Dantrolene, a muscle relaxant, has a direct effect on skeletal muscle by acting on the excitation-contraction coupling of muscle fibers and not at the level of the CNS, unlike most other muscle relaxation medications. STUDY TIP: Becoming a nursing student automatically increases stress levels because of the complexity of the information to be learned and applied and because of new constraints on time. One way to decrease stress associated with school is to become very organized so that assignment deadlines or tests do not come as sudden surprises. By following a consistent plan for studying and completing assignments, you can stay on top of requirements and thereby prevent added stress.

41. 41.ID: 20127755948 A client prescribed albuterol tablets reports nausea every evening with the 9:00 p.m. dose. Which action should the nurse perform to alleviate this side effect? A. Change the time of the dose. B. Hold the 9 p.m. dose. C. Administer the dose with a snack. Correct D. Offer an antiemetic with the dose.

Administering oral doses of albuterol with food helps minimize gastrointestinal discomfort such as nausea.

376) Which test would the nurse identify as specific for the diagnosis of myasthenia gravis? Electromyography Pyridostigmine test Edrophonium chloride test History of physical deterioration

Edrophonium chloride test rationale The edrophonium chloride test uses a medication that is a cholinergic and an anticholinesterase; it blocks the action of cholinesterase at the myoneural junction and inhibits the destruction of acetylcholine. Its action of increasing muscle strength is immediate for a short time. The results of electromyography will be added to the database, but they are nonspecific. Pyridostigmine is a slower-acting anticholinesterase medication that is commonly prescribed to treat myasthenia gravis; edrophonium chloride is used instead of pyridostigmine to diagnose myasthenia gravis because, when injected intravenously, it immediately increases muscle strength for a short time. The results of a history and physical are added to the database, but the data collected are not as definitive as another specific test for the diagnosis of myasthenia gravis.

1030) Which client statement indicates understanding of the side effects of nitroglycerin ointment? 'I may experience a headache.' 'Confusion is a common adverse effect.' 'A slow pulse rate in an expected side effect.' 'Increased blood pressure readings may occur initially.'

Rationale The most common side effect of nitroglycerin is a headache. Additional cardiovascular side effects are hypotension, not hypertension; tachycardia, not bradycardia; and dizziness, not confusion.

221) Which therapeutic effect would the nurse expect to identify when mannitol is administered to a client? Improved renal blood flow Decreased intracranial pressure Maintenance of circulatory volume Prevention of the development of thrombi

Rationale As an osmotic diuretic, mannitol helps reduce cerebral edema. Although there may be a transient increase in blood volume as a result of an increased osmotic pressure, which increases renal perfusion, this is not the therapeutic effect. Prevention of the development of thrombi is not the reason for giving this medication.

Question 8 A nurse is providing care to a client who takes phenytoin for seizure prevention. The latest laboratory report shows a phenytoin level of 32 mcg/mL. Which action does the nurse take next? Question 8 Answer Choices AExamine the oral cavity Examine the oral cavity BPercuss the abdomen Percuss the abdomen CCheck the skin turgor Check the skin turgor DAssess the pupillary response Assess the pupillary response Question Explanation

+Rationale A phenytoin level of 32 mcg/mL is not an expected response to therapy. The therapeutic range of phenytoin is 10 to 20 mcg/mL. Signs of phenytoin toxicity include nystagmus, ataxia, and confusion. The pupillary response will assess for symmetrical movements of the eye. Examining the oral cavity, percussing the abdomen, and checking for skin turgor do not evaluate symptoms of phenytoin toxicity.

46. 46.ID

20127755933 A client is admitted to the hospital for diagnostic testing for possible myasthenia gravis. The nurse prepares for intravenous administration of edrophonium chloride (Tensilon). What is the expected outcome for this client following administration of this pharmacologic agent? A. Progressive difficulty with swallowing. B. Decreased respiratory effort. C. Improvement in generalized fatigue. D. Decreased muscle weakness. Administration of edrophonium chloride (Tensilon), a cholinergic agent, will temporarily reduce muscle weakness, the most common complaint of newly-diagnosed clients with myasthenia gravis. This medication is used to diagnose myasthenia gravis due to its short duration of action. This drug would temporarily reverse difficulty in swallowing and respiratory effort.

Question 5 The nurse is caring for a client who was prescribed alprazolam. When educating the client about the new medication, which intended effect should the nurse include? Question 5 Answer Choices AReduce anxiety and provide a calming effect Reduce anxiety and provide a calming effect BIncrease coordination and the ability to concentrate Increase coordination and the ability to concentrate CAlleviate signs and symptoms of spasticity Alleviate signs and symptoms of spasticity DReduce symptoms of depression Reduce symptoms of depression Question Explanation

+Rationale Alprazolam is a benzodiazepine which is as an anxiolytic. The medication will not increase coordination and the ability to concentrate or alleviate symptoms associated with nerve damage, such as spasticity. Alprazolam will not reduce symptoms of depression.

Question 10 The nurse is providing discharge education to a client who is prescribed alprazolam for a panic disorder. What concept should the nurse emphasize concerning the drug action? Question 10 Answer Choices AShort-term relief can be expected Short-term relief can be expected BThe medication acts as a stimulant The medication acts as a stimulant CThe medication works by suppressing dopamine The medication works by suppressing dopamine DIf you miss a dose, double the next scheduled dose If you miss a dose, double the next scheduled dose Question Explanation

+Rationale Alprazolam is a short-acting benzodiazepine, which works quickly to control panic symptoms by enhancing the effects of the neurotransmitter Gamma-amino butyric acid (GABA). This produces a calming effect. The drug does not suppress dopamine like dopamine antagonists and some antipsychotic medications. Alprazolam will not be increased as tolerated, the lowest dose that controls the symptoms will be maintained.

Question 4 A client has been taking alprazolam for three days. For which expected effect of the medication should the nurse evaluate the client? Question 4 Answer Choices AThe client reports sleeping through the night. The client reports sleeping through the night. BThe client reports feeling less depressed. The client reports feeling less depressed. CThe client denies having auditory hallucinations. The client denies having auditory hallucinations. DThe client denies having suicide ideation. The client denies having suicide ideation. Question Explanation

+Rationale Antianxiety medications or anxiolytics, such as alprazolam, a benzodiazepine, work quickly. They produce sedative effects and reduce anxiety through effects on the limbic system, a neuronal network associated with emotionality. They also promote sleep through effects on cortical areas and on the brain's sleep-wakefulness "clock." Alprazolam is not used to treat depression or hallucinations.

Question 8 The nurse is reviewing the medical record of a client who received a new prescription for benztropine. For which condition in the client's record should the nurse clarify the prescription with the health care provider? Question 8 Answer Choices ACataracts Cataracts BSchizophrenia Schizophrenia CGlaucoma Glaucoma DParkinson's disease Parkinson's disease Question Explanation

+Rationale Benztropine is an anticholinergic medication used to treat extrapyramidal disorders caused by antipsychotic medications or Parkinson's disease. Use of benztropine or other anticholinergics is contraindicated for individuals diagnosed with glaucoma, ileus and prostatic hypertrophy. Adverse effects include tachycardia, urinary retention and increased intraocular pressure.

Question 13 The nurse is caring for a client with Parkinson's disease. Which finding indicates that the client might be experiencing an adverse side effect from the dopamine-enhancing drugs? Question 13 Answer Choices AUrinary retention Urinary retention BHallucinations Hallucinations CKidney failure Kidney failure DHypertensive urgency Hypertensive urgency Question Explanation

+Rationale Carbidopa-levodopa-entacapone is the treatment of choice for clients with Parkinson's disease. Common side effects include dyskinesia, confusion and dizziness. Serious side effects include hallucinations, paranoia and agitation. Hallucinations may be relieved by decreasing the dose of levodopa, but this may decrease the effect of the drug on the motor symptoms of Parkinson's disease.

Question 8 The nurse is preparing to administer diltiazem to a client with heart disease. Which action should the nurse take first? Question 8 Answer Choices AAssess the client's lung sounds and monitor for wheezing Assess the client's lung sounds and monitor for wheezing BAssess the client's blood pressure and apical pulse Assess the client's blood pressure and apical pulse CAssess the client's urine output and potassium level Assess the client's urine output and potassium level DAuscultate the abdomen for bowel sounds Auscultate the abdomen for bowel sounds Question Explanation

+Rationale Diltiazem is a calcium channel blocker that is used to treat hypertension, angina and tachyarrhythmias. The medication works by causing systemic vasodilation and lowering the client's heart rate. Common side effects of diltiazem include hypotension, orthostatic hypotension, bradycardia, edema and headaches. It is not necessary to auscultate the client's lung sounds prior to administering the medication. Wheezing is not considered a side effect of diltiazem. Because the medication can lead to hypotension and bradycardia, it is essential to assess the client's blood pressure and apical pulse prior to administration. It is not necessary to check the client's urine output or potassium level prior to administering the medication. Diltiazem does not affect a client's renal status or potassium level. It is not necessary to check the client's bowel sounds prior to administering the medication. Diltiazem does not affect a client's gastrointestinal system.

Question 17 The daughter of a client with Alzheimer's disease asks the nurse, "Will the medication my mother is taking cure her dementia?" What is the best response by the nurse? Question 17 Answer Choices A"It will help your mother live independently again." "It will help your mother live independently again." B"It is used to halt the progression of Alzheimer's disease." "It is used to halt the progression of Alzheimer's disease." C"It will not improve dementia but can help control emotional responses." "It will not improve dementia but can help control emotional responses." D"It will provide a steady improvement in memory." "It will provide a steady improvement in memory." Question Explanation

+Rationale Drug therapy for Alzheimer's disease such as memantine and donepezil produce modest improvements in cognition, behavior, and function, and slightly delayed disease progression. They do not reverse the dementia or halt the progression of Alzheimer's disease. At best, drugs currently in use may slow loss of memory and improve cognitive functions (e.g., memory, thought, reasoning) and emotional lability. However, these improvements are modest and last a short time and for many clients, even these modest goals are elusive.

Question 13 A nurse is teaching parents of a child recently prescribed the medication phenytoin for seizure control. Which side effect will the nurse include? Question 13 Answer Choices AHypertension Hypertension BInsomnia Insomnia CGingival hyperplasia Gingival hyperplasia DIncreased appetite Increased appetite Question Explanation

+Rationale Gingival hyperplasia (overgrowth of the gums) is a common side effect of phenytoin. Other common side effects include ataxia, central nervous system depression, drowsiness, headache, hypotension, mental confusion, nausea, vomiting, rash and nystagmus.

Question 14 The nurse is caring for a child diagnosed with seizures. While teaching the family and the child about the medication phenytoin, which concept should the nurse emphasize? Question 14 Answer Choices AOmit the medication if the child is seizure-free Omit the medication if the child is seizure-free BServe a diet that is high in iron Serve a diet that is high in iron CA rash is normal with this medication A rash is normal with this medication DMaintain good oral hygiene and dental care Maintain good oral hygiene and dental care Question Explanation

+Rationale Gingival hyperplasia may occur with this medication. It is important that good oral hygiene is maintained. The medication should never be stopped, even if the child is seizure-free. A sudden discontinuation could result in status epilepticus. A diet high in iron interferes with phenytoin absorption and will reduce the effectiveness. A blister-like rash is not normal with this medication and could indicate medication-related Stevens-Johnson syndrome, which is a serious disorder of the skin and mucous membranes.

Question 20 A newly admitted client reports taking phenytoin for several months. Which assessment should the nurse include in the admission report? Select all that apply. Report of unsteady gait, rash and diplopia Report of any seizure activity Serum phenytoin levels Report of anorexia, numbness and tingling of the extremities Question Explanation

+Rationale Serious adverse outcomes of antiseizure medications such as phenytoin are unsteady gait, slurred speech, extreme fatigue, blurred vision or feelings of suicide. Clients who are prescribed phenytoin should have their levels monitored on a routine basis. The nurse should include any seizure activity as this may demonstrate lack of a therapeutic level. Increased hunger (not anorexia), increased thirst or increased urination are additional serious side effects.

Question 16 The caregiver of a client with Alzheimer's disease asks the nurse for information about different treatment options that can help with memory or behavior problems. Which of the following responses by the nurse are ? Select all that apply. "Music therapy has been found to help some clients." "Ginkgo biloba may help with memory." "Acupuncture may be very relaxing." "Donepezil (Aricept) may help slow cognitive decline." "Garlic may help with this disease." Question Explanation

+Rationale Some complementary and integrative health therapies may help with the symptoms of Alzheimer's disease. Music, art and dance therapies can help with behavior issues. Ginkgo biloba may be used to improve memory. Acupuncture may be a frightening experience for someone with Alzheimer's disease. Garlic is not a treatment for Alzheimer's disease. Donepezil (Aricept) is used to ease the symptoms associated with Alzheimer's disease.

Question 3 During the admission process, the client reports heavy alcohol use for at least one year. What effect does the nurse anticipate the hospitalized client will experience when alcohol consumption stops? Question 3 Answer Choices ABradycardia Bradycardia BSomnolence Somnolence CWithdrawal Withdrawal DTachypnea Tachypnea Question Explanation

+Rationale The findings of alcohol withdrawal develop within 24 to 48 hours after people either stop or significantly reduce their alcohol consumption. Findings of withdrawal can range from "mild" (shaking or sweating, or perhaps nausea, headache, anxiety, tachycardia or hypertension) to severe (delirium tremens or DTs), which are characterized by rapid heartbeat, fever, hallucinations or seizures.

Question 20 A client with Parkinson's disease is prescribed benztropine (Cogentin). For which of the following should the nurse call the health care provider immediately? Question 20 Answer Choices AThe client is complaining of dizziness when standing up. The client is complaining of dizziness when standing up. BThe client is exhibiting bradykinesia and slurred speech. The client is exhibiting bradykinesia and slurred speech. CThe client's heart rate increased from 80 to 95 beats per minute. The client's heart rate increased from 80 to 95 beats per minute. DThe client has a history of primary angle-closure glaucoma. The client has a history of primary angle-closure glaucoma. Question Explanation

+Rationale The nurse must be able to recognize adverse drug effects and contraindications of medications commonly prescribed for the client with Parkinson's disease. Common clinical manifestations of Parkinson's disease include bradykinesia (slow movement), dysarthria (slurred speech) and orthostatic hypotension, caused by the loss of the neurotransmitter dopamine. The goal of pharmacotherapy is to restore the functional balance of dopamine and acetylcholine. This is achieved by giving dopaminergic drugs and cholinergic blockers. Benztropine is an anticholinergic medication used in the treatment of Parkinson's disease that blocks excess cholinergic stimulation in the brain and reduces muscular tremors and rigidity. Tachycardia is a potential adverse drug event, but a heart rate increase of 15 bpm is within acceptable limits. Due to their blocking actions of the parasympathetic nervous system, anticholinergics are contraindicated with glaucoma, where they can cause an increase in intraocular pressure (IOP), which can lead to vision loss and blindness.

Question 9 A nurse is providing education on the use of carbidopa/levodopa to a client with Parkinson's disease. What will the nurse include in the teaching? Question 9 Answer Choices AThis medication will stop the progression of your condition "This medication will stop the progression of your condition." BNotify your healthcare provider if your urine appears dark "Notify your healthcare provider if your urine appears dark." CEat plenty of whole-grain foods when taking this medication "Eat plenty of whole-grain foods when taking this medication." DAvoid eating meals that are high in protein "Avoid eating meals that are high in protein." Question Explanation

+Rationale Carbidopa/levodopa is a combination medication used in the management of Parkinson's disease. Consuming high-protein meals can impair the effects of levodopa. The nurse should instruct the client to eat protein in small portions. Carbidopa/levodopa does not halt the progression of Parkinson's disease. The medication is intended to reduce the symptoms associated with the condition. Darkening of bodily fluids can occur when taking the medication. However, the client should be informed this is not a harmful side effect. Whole grains contain pyridoxine, a vitamin that interferes with the effects of levodopa.

Question 20 The nurse is completing a health history of a client diagnosed with Alzheimer's disease. The nurse reviews a list of the client's medications and supplements routinely taken at home. Which treatment should be a cause for concern by the nurse? Question 20 Answer Choices ADonepezil Donepezil BGinkgo biloba Ginkgo biloba COmega-3 fatty acids Omega-3 fatty acids DCoconut oil Coconut oil Question Explanation

+Rationale Donepezil, rivastigmine, and galantamine are most commonly used in the treatment of Alzheimer's disease (AD). Complementary and integrative therapies used to treat AD include Gingko biloba (a plant extract) and omega-3 fatty acids. While there isn't sufficient research to support using these treatments, continued use won't necessarily be harmful. However, coconut oil, which is a source of caprylic acid, is a concern. While there has been limited research on Katasyn (an experimental drug containing caprylic acid), there is no scientific evidence that coconut oil is safe and effective or prevents cognitive decline.

Question 20 The nurse is educating a client about newly prescribed alprazolam. Which information should the nurse include in the teaching? Question 20 Answer Choices ATardive dyskinesia is common early in treatment. Tardive dyskinesia is common early in treatment. BAdministration of paroxetine may be needed to prevent adverse effects. Administration of paroxetine may be needed to prevent adverse effects. CThe use of grapefruit juice should be avoided. The use of grapefruit juice should be avoided. DHyperactivity is seen with long-term use. Hyperactivity is seen with long-term use. Question Explanation

+Rationale Grapefruit or grapefruit juice is a known food-drug interaction and may increase drug levels of alprazolam to potentially toxic concentrations. Paroxetine when given with alprazolam will increase the incidence of adverse side effects. Alprazolam is used to treat tardive dyskinesia and is not an adverse side effect of this medication. In general, side effects of benzodiazepines with long-term use include drowsiness, lethargy, and weight gain but not hyperactivity.

Question 6 The nursing is preparing to administer phenytoin IV push to a client. The client has dextrose 5% in water infusing continuously. Which action is appropriate? Question 6 Answer Choices APinch the line above the infusion port during the administration Pinch the line above the infusion port during the administration BHold the medication and collaborate with the provider prior to administration Hold the medication and collaborate with the provider prior to administration CStop the infusion and flush the port with normal saline prior to administration Stop the infusion and flush the port with normal saline prior to administration DAsk the pharmacy to mix the medication into an IV piggyback (IVPB) infusion Ask the pharmacy to mix the medication into an IV piggyback (IVPB) infusion Question Explanation

+Rationale If giving phenytoin as an infusion, it cannot be administered with D5W because it will precipitate. The D5W should be disconnected, the port flushed with normal saline solution (NSS), medication administered, and the port flushed again with NSS before the D5W is reconnected. The provider does not need to be contacted as this is best practice and aligns with hospital protocol. Administering the medication via IVPB does not reduce the risk for precipitation.

Question 21 The nurse in the neurology office is reviewing information about levetiracetam with a 30-year-old female client with a history of seizures. Which instruction about the medication should the nurse make sure to include? Question 21 Answer Choices A"Call the office immediately if you feel like hurting or killing yourself." "Call the office immediately if you feel like hurting or killing yourself." B"You should avoid becoming pregnant while taking this medication." "You should avoid becoming pregnant while taking this medication." C"You should stay away from large crowds and sick children." "You should stay away from large crowds and sick children." D"You might experience irregular menses and intermittent bleeding." "You might experience irregular menses and intermittent bleeding." Question Explanation

+Rationale Levetiracetam is an anti-convulsant medication used to prevent seizures. One of the significant side effects is behavioral changes and suicidal ideations. It is important to notify the provider's office immediately if the client experiences these thoughts. The other instructions do not apply to this particular medication.

Question 3 The nurse is preparing to administer newly prescribed intravenous phenytoin to a client. When reviewing the client's medical record, which prescription should the nurse question? Question 3 Answer Choices AContinuous infusion of dextrose 5% in 0.9% saline Continuous infusion of dextrose 5% in 0.9% saline Answer BNPH insulin 40 units before meals NPH insulin 40 units before meals CLabetalol 100 mg orally twice per day Labetalol 100 mg orally twice per day Your Answer DKetorolac 15 mg IV push as needed for pain Ketorolac 15 mg IV push as needed for pain Question Explanation

+Rationale Phenytoin is not compatible with most IV fluids, especially those with dextrose. If the nurse observes a continuous infusion of a fluid that contains dextrose, they should understand that incompatibilities are likely and should not administer the medication as prescribed. Insulin, labetalol, and ketorolac do not have potential incompatibilities.

19. 19.ID

20127756616 A client is experiencing anaphylaxis from an insect sting. Which medication should the nurse administer? A. Dopamine. B. Ephedrine. C. Epinephrine. D. Diphenhydramine. Epinephrine is an adrenergic agent that stimulates beta receptors to increase cardiac automaticity in cardiac arrest and relax bronchospasms in anaphylaxis. Epinephrine is the medication of choice in treating anaphylaxis.

Question 6 The nurse is educating a client with seizure disorder about newly prescribed phenytoin. Which statement should the nurse include in the teaching? Question 6 Answer Choices ABlood work will be required if you have a seizure while taking this medication "Blood work will be required if you have a seizure while taking this medication." BYou will need to have routine visits with a dentist when taking this medication "You will need to have routine visits with a dentist when taking this medication." CIt is normal to have a change in your gait when you first start this medication "It is normal to have a change in your gait when you first start this medication." DAvoid grapefruit juice when taking this medication "Avoid grapefruit juice when taking this medication." Question Explanation

+Rationale Phenytoin is the first-line medication for the treatment of seizures. Clients should be instructed that they will need routine lab work to ensure that they are at a therapeutic level with the medication, even if they have been seizure-free. This medication can cause gingival hyperplasia, which will require routine dental visits. The client does not need to avoid grapefruit juice with this medication. Difficulty with hand and gait coordination could indicate toxicity and should be reported to the healthcare provider.

Question 9 A nurse is providing education on the use of pregabalin to a client with a seizure disorder. Which client statement indicates further teaching is required? Question 9 Answer Choices AI will record the number of seizures I experience "I will record the number of seizures I experience." BI will hold the dose if my seizures are controlled "I will hold the dose if my seizures are controlled." CI will notify my healthcare provider if I have significant mood changes "I will notify my healthcare provider if I have significant mood changes." DI will report any weight gain to my healthcare provider "I will report any weight gain to my healthcare provider." Question Explanation

+Rationale Pregabalin is an anticonvulsant medication used to manage seizure disorders. The client should take the medication as prescribed as abrupt discontinuation can lead to seizure activity. Recording the number of seizures helps to evaluate the effectiveness of the medication. Pregabalin can cause suicidal thoughts and behaviors. The client should promptly report significant mood changes. Pregabalin can cause weight gain and peripheral edema. These side effects should be reported to the healthcare provider.

Question 15 An older adult client has been prescribed zolpidem for insomnia. The nurse should monitor the client for which side/adverse effect of this medication? Question 15 Answer Choices A Tachypnea B Hypotension C Tachycardia D Constipation Question Explanation

+Rationale Zolpidem is classified as a non-benzodiazepine and acts as a GABA agonist which can cause central nervous system depression, including drowsiness and lightheadedness. Nurses should be aware of the sedative effects and assess for hypotension, bradycardia, and bradypnea. Diarrhea, not constipation, is more commonly associated with the administration of this medication.

55. 55.ID: 20127755906 A client is receiving clonidine 0.1 mg/24 hr via transdermal patch. Which assessment finding indicates the desired effect of the medication has been achieved? A. Absence of nausea and vomiting. B. Change in peripheral edema from

3 to +1. C. Denial of anginal pain and shortness of breath. D. Blood pressure from 180/120 mmHg to 140/70 mmHg. Clonidine acts as a centrally-acting analgesic and antihypertensive agent. A reduction of the blood pressure reading of 180/120 mmHg to 140/70 mmHg indicates a reduction in hypertension. Awarded 0.0 points out of 1.0 possible points.

185) A depressed client has been prescribed a tricyclic antidepressant. Which time period indicates how long it usually takes before the client notices a significant change in the depression? 4 to 6 days 2 to 4 weeks 5 to 6 weeks 12 to 16 hours

2-4 weeks rationale It takes 2 to 4 weeks for the tricyclic antidepressant to reach a therapeutic blood level. Time spans of 4 to 6 days and 12 to 16 hours are both too short for a therapeutic blood level of the medication to be achieved. Improvement in depression should be demonstrated sooner than 5 to 6 weeks. STUDY TIP: Becoming a nursing student automatically increases stress levels because of the complexity of the information to be learned and applied and because of new constraints on time. One way to decrease stress associated with school is to become very organized so that assignment deadlines or tests do not come as sudden surprises. By following a consistent plan for studying and completing assignments, you can stay on top of requirements and thereby prevent added stress.

62. 62.ID

20127754585 A client is being treated for osteoporosis with alendronate (Fosamax), and the nurse has completed discharge teaching regarding medication administration. Which morning schedule would indicate to the nurse that the client teaching has been effective? A. Take medication, go for a 30 minute morning walk, then eat breakfast. Correct B. Take medication, rest in bed for 30 minutes, eat breakfast, go for morning walk. C. Take medication with breakfast, then take a 30 minute morning walk. D. Go for a 30 minute morning walk, eat breakfast, then take medication. Alendronate (Fosamax) is best absorbed when taken thirty minutes before eating in the morning. The client should also be advised to remain in an upright position for at least thirty minutes after taking the medication to reduce the risk of esophageal reflux and irritation.

327) A client receiving fluphenazine decanoate develops dystonia early in therapy. Which medication would the nurse anticipate administering to reverse this side effect? Nafarelin Fluoxetine Trandolapril Benztropine

Benztropine rationale Dystonia is an extrapyramidal side effect (EPS) of fluphenazine decanoate. The anticholinergic benztropine is used to reverse the signs and symptoms (e.g., oculogyric crisis, torticollis, retrocollis) of dystonia. Nafarelin is a gonadotropin that stimulates the release of luteinizing hormone and follicle-stimulating hormone. Fluoxetine is a selective serotonin reuptake inhibitor antidepressant. Trandolapril is an angiotensin-converting enzyme inhibitor antihypertensive. Test-Taking Tip: Multiple-choice questions can be challenging, because students think that they will recognize the right answer when they see it or that the right answer will somehow stand out from the other choices.

721) Which medication is often contraindicated when taking warfarin? Atenolol Ferrous sulfate Chlorpromazine Acetylsalicylic acid

Rationale Acetylsalicylic acid can cause decreased platelet aggregation, increasing the risk for undesired bleeding that may occur with administration of anticoagulants. It should not be administered unless specifically prescribed, usually by a cardiologist or other specialist, to manage serious risks of thrombosis. Ferrous sulfate does not affect warfarin; it is used for red blood cell synthesis. Atenolol is a beta-blocker that reduces blood pressure; it does not affect bleeding. Chlorpromazine is a neuroleptic; it does not affect bleeding.

405) Which sedative-hypnotics would the nurse identify as being used to treat insomnia associated with a panic disorder? Select all that apply. One, some, or all responses may be . Phenelzine Paroxetine Alprazolam Imipramine Clonazepam

Rationale Alprazolam and clonazepam are examples of benzodiazepines, a class of sedative-hypnotics used to treat clients with insomnia associated with panic disorders. Phenelzine is a monoamine oxidase inhibitor used to treat panic disorders and promote sleep. Paroxetine is a selective serotonin reuptake inhibitor used to treat panic disorders and promote sleep. Imipramine is a tricyclic antidepressant used to treat panic disorders and promote sleep.

13. 13.ID: 20127756634 The nurse admits a client with tumor-induced spinal cord compression. Which medication should the nurse anticipate to be prescribed to offer the best palliative treatment for this client? A. Morphine sulfate. B. Ibuprofen. C. Amitriptyline. D. Dexamethasone. Correct

Dexamethasone is a palliative treatment modality to manage symptoms related to compression due to tumor growth. Morphine sulphate is an opioid analgesic used in oncology to manage severe or intractable pain. Ibuprofen, a nonsteroidal antiinflammatory drug (NSAID), provides relief for mild to moderate pain, suppression of inflammation, and reduction of fever. Amitriptyline, a tricyclic antidepressant, is often prescribed for pain related to neuropathic origin and provides a reduction in opioid dosage.

86) A client is admitted to the acute medical unit for severe amphetamine intoxication. Which medications are indicated to counteract the effects of stimulant intoxication? Select all that apply. One, some, or all responses may be . Diazepam Propranolol Benztropine Bupropion Amitriptyline

Diazepam, propranolol rationale Because stimulants act by increasing both adrenaline and dopamine, seizures may occur. Diazepam can reduce the chance of seizures. Because amphetamines act by increasing adrenaline, which can stimulate the heart, propranolol, a beta-blocker, will decrease this adrenergic stimulation. Benztropine, a cholinergic blocker, is not indicated as a treatment for stimulant intoxication. Bupropion is contraindicated because it increases dopamine and adrenaline, which will exacerbate stimulant intoxication. Amitriptyline is contraindicated because it increases dopamine and adrenaline, which will exacerbate stimulant intoxication. STUDY TIP: Focus your study time on the common health problems that nurses most frequently encounter.

48) A client with a history of methamphetamine use is admitted to the medical unit. Which clinical manifestation would the nurse expect when assessing the client who is under the influence on admission? Constricted pupils Intractable diarrhea Increased heart rate Decreased respirations

Increased heart rate rationale Methamphetamine is a stimulant that causes the release of adrenaline, which activates the sympathetic nervous system and causes increased heart rate. The pupils will dilate, not constrict, because the sympathetic nervous system is activated. Clients withdrawing from opioids, not methamphetamine, experience diarrhea. The respirations will increase, not decrease, because of the activation of the sympathetic nervous system. #

860) A client receives a cardiac glycoside, a diuretic, an angiotensin-converting enzyme (ACE) inhibitor, and a vasodilator. The client's apical pulse rate is 44 beats/minute. The nurse concludes that the decreased heart rate is caused by which medication? Diuretic Vasodilator ACE inhibitor Cardiac glycoside

Rationale A cardiac glycoside such as digoxin decreases the conduction speed within the myocardium and slows the heart rate. The primary effect of a diuretic is on the kidneys, not the heart; it may reduce the blood pressure, not the heart rate. A vasodilator can cause tachycardia, not bradycardia, which is an adverse effect. ACE inhibitors act on the renin-angiotensin system and are not associated with decreased heart rates. Test-Taking Tip: Notice how the subjects of the questions are related and, through that relationship, the answers to some of the questions you skipped may be provided within other questions of the test.

110) A primary health care provider prescribes venlafaxine for a client with a diagnosis of major depressive disorder who has been taking herbal medications. Which herbal supplement is contraindicated when taking venlafaxine? Ginseng Valerian Kava-kava St. John's wort

Rationale A client who takes venlafaxine, a selective serotonin reuptake inhibitor, and St. John's wort concurrently is at risk for serotonin syndrome, a medication-induced excess of intrasynaptic serotonin. Ginseng can precipitate a hypertensive crisis in clients taking a monoamine oxidase inhibitor. Valerian (valerian root) can enhance sedation in clients taking a tricyclic antidepressant. Kava-kava can increase the risk of dystonic reactions in clients taking an antipsychotic medication.

514) The nurse teaches the parents of a child prescribed long-term phenytoin therapy about care. Which statement indicates the teaching has been effective? 'We give the medication between meals.' 'We'll call the clinic if her urine turns pink.' 'She's eating high-calorie foods, and we encourage fluids, too.' 'We'll have her massage her gums and floss her teeth frequently.'

Rationale A common side effect of phenytoin is gingival hyperplasia. Meticulous oral hygiene may reduce the risk of this side effect. Phenytoin is strongly alkaline and should be administered with meals to help prevent gastric irritation. Pink urine may be observed during medication excretion; it is expected and does not require treatment. Avoidance of overeating and overhydration may result in better seizure control.

Question 1 The nurse is assessing a client with hypertension who reports experiencing dizziness after taking prescribed diltiazem. It is most important that the nurse assesses for which client characteristic? Question 1 Answer Choices ASchedule for taking medication Schedule for taking medication Answer BAppearance of feet and ankles Appearance of feet and ankles CActivity and rest patterns Activity and rest patterns DDaily intake of potassium Daily intake of potassium Question Explanation

Rationale A critical focus is whether the client has complied with the prescribed medication schedule and dose. Although diltiazem (Cardizem, Cartia, Dilacor, Diltia, Taztia, Tiazac) can be taken either in the morning or evening, taking the medication in the evening might help with this common side effect.

976) A beta blocker is prescribed for the client with persistent ventricular tachycardia. Which response indicates that the beta blocker is working effectively? Decreased anxiety Reduced chest pain Decreased heart rate Increased blood pressure

Rationale A decreased heart rate is the expected response to a beta blocker. Beta blockers inhibit the activity of the sympathetic nervous system and of adrenergic hormones, decreasing the heart rate, conduction velocity, and workload of the heart. A beta blocker is not an anxiolytic and does not reduce anxiety. A beta blocker is not an analgesic and does not reduce chest pain. Beta blockers reduce blood pressure. Test-Taking Tip: Read the question carefully before looking at the answers: (1) Determine what the question is really asking; look for key words; (2) Read each answer thoroughly and see if it completely covers the material asked by the question; (3) Narrow the choices by immediately eliminating answers you know are in.

198) Which adverse effect will the nurse monitor for in a client who has been administered a neuromuscular blocking agent before electroconvulsive therapy? Seizures Vomiting Loss of memory Respiratory depression

Rationale A neuromuscular blocker such as succinylcholine produces respiratory depression because it inhibits contractions of respiratory muscles. This medication does not cause seizures. Because the client is not permitted anything by mouth for 8 to 10 hours before the treatment, vomiting is not a major problem. The loss of memory results from the electroconvulsive therapy, not from the neuromuscular blocking agent.

650) A client with insomnia takes a new medication to promote sleep. Two hours later, the client is wide awake and feeling more energized than before the medication was taken. Which type of response is the client demonstrating? Allergic Synergistic Paradoxical Idiosyncratic

Rationale A paradoxical response to a medication is directly opposite to the desired therapeutic response. An allergic response is an antigen-antibody reaction. A synergistic response involves medication combinations that enhance each other. Idiosyncratic responses to a medication are unpredictable and unrelated to the medication purpose. They are commonly attributed to immune responses or genetic factors.

699) Atenolol is prescribed for a client with moderate hypertension. Which information would the nurse include when teaching the client about this medication? Select all that apply. One, some, or all responses may be . Change to standing positions slowly. Take the medication before going to bed. Count the pulse before taking the medication. Mild weakness and fatigue are common side effects. It is safe to take over-the-counter (OTC) medications.

Rationale A side effect of this medication is orthostatic hypotension. The client should be advised to move to a standing position slowly to allow the body to adjust to the new position. The rate of the pulse should be taken before administering the medication; slower rates due to ventricular dysrhythmias and heart block may occur. Mild weakness and fatigue are side effects of this medication. The blood pressure decreases when the client is sleeping; the medication usually is prescribed to be administered earlier in the day. The medication should be taken with food. No OTC medication should be taken without consulting the prescribing health care provider; decreased or increased effects can occur when there is an interaction with another medication.

777) A client is scheduled for a craniotomy to remove a brain tumor. To prevent the development of cerebral edema after surgery, the nurse anticipates the use of medications from which class? Glucocorticoids Anticholinergics Anticonvulsants Antihypertensives

Rationale Glucocorticoids are used for their anti-inflammatory action, which decreases the development of cerebral edema. Anticholinergics are not used to prevent cerebral edema. Anticonvulsants prevent seizure activity, not cerebral edema. Antihypertensives control hypertension, not cerebral edema.

43) Which instruction would the nurse give an unlicensed assistive personnel (UAP) to perform while caring for a client prescribed captopril? Select all that apply. One, some, or all responses may be . Obtain blood pressure. Measure intake and output. Weigh the client every morning. Notify the nurse if the client has a dry cough. Assist the client to change positions slowly.

Rationale ACE inhibitors such as captopril are prescribed for the management of hypertension, heart failure, and diabetic nephropathy. The nurse would ask the UAP caring for a client taking captopril to perform several tasks. The UAP would obtain the client's blood pressure. The UAP would also measure the client's intake and output as well as obtain a daily weight in the morning. This data would help the nurse determine the client's fluid volume status and is an important component of heart failure management. The UAP would be aware that a dry cough is a common side effect of ACE inhibitors. Because of the blood pressure-lowering effects of this medication, the nurse would instruct the UAP to assist the client to make sure the client changes positions slowly.

190) Which symptom of levodopa toxicity will a client taking levodopa be taught as a reason to contact the primary health care provider? Nausea Dizziness Twitching Constipation

Rationale Abnormal involuntary movements (dyskinesias), such as muscle twitching, rapid eye blinking, facial grimacing, head bobbing, and an exaggerated protrusion of the tongue, are signs of toxicity; these probably result from the body's failure to readjust properly to the reduction of dopamine. Nausea is a side effect of therapy, not toxicity. Dizziness is a side effect of therapy, not toxicity. Constipation is unrelated to levodopa toxicity. Test-Taking Tip: Avoid taking a wild guess at an answer. However, should you feel insecure about a question, eliminate the alternatives that you believe are definitely in and reread the information given to make sure you understand the intent of the question. This approach increases your chances of randomly selecting the answer or getting a clearer understanding of what is being asked. Although there is no penalty for guessing, the subsequent question will be based, to an extent, on the response you give to the question at hand; that is, if you answer a question inly, the computer will adapt the next question accordingly based on your knowledge and skill performance on the examination up to that point.

949) A client with hypertension has received a prescription for metoprolol. Which information will the nurse include when teaching this client about metoprolol? Do not abruptly discontinue the medication. Consume alcoholic beverages in moderation. Report a heart rate of less than 70 beats per minute. Increase the medication dosage if chest pain occurs.

Rationale Abrupt discontinuation of metoprolol may cause rebound hypertension and an acute myocardial infarction. Alcohol is contraindicated for clients taking beta-adrenergic blockers such as metoprolol. Clients should never increase medications without medical direction. The pulse rate can go lower than 70 beats per minute as long as the client is asymptomatic.

307) A client has completed therapy and requests to discontinue treatment with benzodiazepines. Which information would the nurse provide to prevent injury? Taper dose over several months. Switch to using an antidepressant. Refrain from renewing prescription. Monitor for signs of increased anxiety.

Rationale Abruptly discontinuing the use of benzodiazepines can cause withdrawal symptoms including intense anxiety and worsening of panic disorders. Therefore, the dose will need to be tapered over several months. Switching to an antidepressant does not alleviate withdrawal symptoms when discontinuing benzodiazepines. Refraining from renewing the prescription would lead to abrupt discontinuation of the medication and would be discouraged. Monitoring for signs of increased anxiety would not actively prevent injury.

983) A health care provider prescribes milrinone for a client with congestive heart failure. Which action would the nurse perform first? Administer the loading dose over 10 minutes. Monitor the electrocardiogram (ECG) continuously for dysrhythmias during infusion. Assess the heart rate and blood pressure continuously during infusion. Have the prescription, dosage calculations, and pump settings checked by a second nurse.

Rationale Accidental overdose can cause death. Another nurse would verify accuracy of the prescription, dose, and pump settings to prevent harm to the client. Although administering the loading dose over 10 minutes is an appropriate intervention, it is not the first thing the nurse would do. Although monitoring for dysrhythmias is important because they are common with this medication and may be life threatening, it is not the first thing the nurse would do. Although taking the vital signs continuously during the infusion is important because the dose needs be slowed or discontinued if the blood pressure decreases excessively, it is not the first thing the nurse would do. Test-Taking Tip: Identify option components as or in. This may help you identify a wrong answer. Example: If you are being asked to identify a diet that is specific to a certain condition, your knowledge about that condition would help you choose the response (e.g., cholecystectomy = low-fat, high-protein, low-calorie diet).

717) The nurse provides discharge medication education to a client who has a prescription for warfarin. Which client statement indicates to the nurse that teaching was effective? 'I will avoid taking aspirin and nonsteroidal anti-inflammatory drugs [NSAIDs].' 'I will need to develop a more sedentary routine.' 'I will need to have regular complete blood counts to guide warfarin dosage.' 'Before going to the dentist, I will ask my health care provider for antibiotics.'

Rationale Acetaminophen should be used when an analgesic is required because it does not interfere with platelet aggregation. Acetylsalicylic acid (aspirin) should be avoided because it interferes with platelet aggregation. Immobility causes venous pooling and can predispose the client to deep vein thrombosis. Antibiotics are not necessary when going to the dentist; this is done when clients have cardiac problems, such as rheumatic fever or cardiac surgery. A prothrombin time (PT) or international normalized ratio (INR), not a complete blood count, needs to be done periodically.

881) Which medication would the nurse anticipate may be prescribed to produce diuresis and inhibit formation of aqueous humor for a client with glaucoma? Chlorothiazide Acetazolamide Methazolamide Aclidinium bromide

Rationale Acetazolamide is a carbonic anhydrase inhibitor that decreases inflow of aqueous humor and controls intraocular pressure in acute angle-closure glaucoma attack. Chlorothiazide has no effect on the eye. Methazolamide lowers ocular pressure but does not decrease the inflow of aqueous humor. Aclidinium bromide is a bronchodilator. Test-Taking Tip: Have confidence in your initial response to an item because it more than likely is the correct answer.

1038) A client's phenytoin level is 16 mcg/L. Which action will the nurse take? Hold the medication and notify the health care provider. Administer the next dose of the medication as prescribed. Hold the next dose and then resume administration as prescribed. Call the health care provider to obtain a prescription with an increased dose.

Rationale Administering the next dose of the medication as prescribed is within the therapeutic range of 10 to 20 mcg/L (40-80 mcmol/L); the nurse would administer the medication as prescribed. Holding the next dose and then resuming administration as prescribed is unsafe and will reduce the therapeutic blood level of the medication. Calling the health care provider to obtain a prescription with an increased dose is inappropriate because the blood level is within the therapeutic range.

1119) A client is admitted to the hospital for an adrenalectomy. When teaching the client about the prescribed medications, which advice will the nurse emphasize? Medication therapy will be given in conjunction with insulin. Once regulated, the dosage will remain the same for life. Medications will need to be held for surgery or other invasive procedures. Salt intake may have to be restricted.

Rationale Administration of adrenocortical hormones causes sodium retention; dietary intake of salt should be limited. Because pancreatic function is unimpaired, insulin therapy is not indicated. Dosages will likely need to be adjusted over time. The dosage will need to be increased for surgery and severe infections; not doing this can cause a life-threatening crisis. Test-Taking Tip: Pace yourself when taking practice quizzes. Because most nursing exams have specified time limits, you should pace yourself during the practice testing period accordingly. It is helpful to estimate the time that can be spent on each item and still complete the examination in the allotted time. You can obtain this figure by dividing the testing time by the number of items on the test. For example, a 1-hour (60-minute) testing period with 50 items averages 1.2 minutes per question. The NCLEX exam is not a timed test. Both the number of questions and the time to complete the test vary according to each candidate's performance. However, if the test taker uses the maximum of 5 hours to answer the maximum of 265 questions, each question equals 1.3 minutes.

525) A school-age child with a seizure disorder is prescribed divalproex. The nurse would include which instruction when teaching the parents about administering this medication? 'Crush the tablets and mix them with applesauce.' 'Take the child for regularly scheduled blood tests.' 'Stop the medication immediately if a rash develops.' 'Provide oral hygiene, especially gum massage and flossing.'

Rationale Adverse reactions to divalproex include thrombocytopenia, leukopenia, and lymphocytosis; blood studies must be performed on a regular basis. Tablets must be swallowed whole; they should not be broken, crushed, or chewed. If the medication is stopped suddenly, a seizure may result; a rash should be reported to the health care provider. Meticulous oral hygiene is more important for a child who is taking phenytoin.

Question 13 The nurse is teaching a client with asthma about albuterol. How should the nurse best describe the action of this medication? Question 13 Answer Choices A"The medication is given to reduce secretions that block airways." "The medication is given to reduce secretions that block airways." B"The medication will help to relax smooth muscles in the airways." "The medication will help to relax smooth muscles in the airways." C"The medication will stimulate the respiratory center in the brain." "The medication will stimulate the respiratory center in the brain." D"The medication will help to prevent pneumonia." "The medication will help to prevent pneumonia." Question Explanation

Rationale Albuterol is a bronchodilator and rescue drug of choice to treat asthma. It is a short-acting beta-adrenergic agonist that is used to prevent and treat wheezing, difficulty breathing, and chest tightness. Albuterol works by relaxing and opening the airways to make breathing easier. The medication comes as a tablet, syrup, inhaler and nebulizer. Albuterol does not reduce secretions, stimulate the respiratory center in the brain or prevent pneumonia.

Question 1 The nurse is teaching a client diagnosed with asthma about the medication albuterol. Which statement by the nurse demonstrates appropriate teaching? Question 1 Answer Choices A"Call your doctor's office if you need to use the drug more often." "Call your doctor's office if you need to use the drug more often." B"Use this medication at bedtime to promote rest." "Use this medication at bedtime to promote rest." C"Use this medication after other asthma inhalers." "Use this medication after other asthma inhalers." D"Discontinue the inhaler if you feel dizzy." "Discontinue the inhaler if you feel dizzy." Question Explanation

Rationale Albuterol is a bronchodilator used for the relief of bronchospasm. It is considered a rescue medication for a client during an asthmatic attack. If the client notices the need to use the inhaler more frequently, the health care provider (HCP) should be notified. The client may need to seek emergency medical care, as the medication is no longer effective. In addition, clients should not exceed the recommended dosage, as adverse effects may occur. Be sure the client understands how to correctly use this medication. The client may experience side effects of dizziness, headache, nausea, vomiting, rapid heart rate, anxiety, sweating, flushing and insomnia. Using albuterol at bedtime may lead to insomnia. Albuterol should be used before all other inhalers, as it dilates the bronchi or bronchioles and allows more of the other medication to reach the lower respiratory tract. It would not be appropriate to suddenly discontinue taking a bronchodilator.

536) The nurse administers albuterol to a 4-year-old child. Which intervention would assist the nurse in evaluating the effectiveness of this medication? Auscultate breath sounds. Collect a sputum sample. Conduct a neurological examination. Palpate chest excursion.

Rationale Albuterol is an adrenergic medication that stimulates beta-receptors, leading to relaxation of the smooth muscles of the airway. The lungs should be auscultated to evaluate the effectiveness of this medication. Albuterol does not affect the consistency of pulmonary secretions. Albuterol will not cause central nervous system stimulation. Albuterol does not affect intercostal contractility; chest excursion is not the appropriate assessment.

Question 5 A client with chronic obstructive pulmonary disease (COPD) is receiving aminophylline 25 mg/hour intravenously (IV). Which finding would be associated with side effects of this medication? Question 5 Answer Choices AFlushing and headache Flushing and headache BRestlessness and palpitations Restlessness and palpitations CDecreased urine volume Decreased urine volume DPruritus Pruritus Question Explanation

Rationale Aminophylline is a bronchodilator often used to treat symptom of asthma, bronchitis, and emphysema. Side effects include restlessness, palpitations, chest pain or discomfort, increased urine volume, vertigo, and vomiting. The other choices are not side effects of this drug.

597) The nurse administers albuterol to a child with asthma. Which common side effect would the nurse monitor for in the child? Flushing Dyspnea Tachycardia Hypotension

Rationale Albuterol produces sympathetic nervous system side effects such as tachycardia and hypertension. Pallor, not flushing, is a common side effect. Dyspnea is not a common side effect; this medication is given to decrease respiratory difficulty. Hypertension, not hypotension, is a common side effect. Test-Taking Tip: Study wisely, not hard. Use study strategies to save time and be able to get a good night's sleep the night before your exam. Cramming is not smart, and it is hard work that increases stress while reducing learning. When you cram, your mind is more likely to go blank during a test. When you cram, the information is in your short-term memory so you will need to relearn it before a comprehensive exam. Relearning takes more time. The stress caused by cramming may interfere with your sleep. Your brain needs sleep to function at its best.

664) A client is prescribed albuterol to relieve severe asthma. Which adverse effects will the nurse instruct the client to anticipate? Select all that apply. One, some, or all responses may be correct. Tremors Lethargy Palpitations Bronchoconstriction Decreased pulse rate

Rationale Albuterol's sympathomimetic effect causes central nervous system (CNS) stimulation, precipitating tremors, tachycardia, and palpitations. Lethargy is an adverse effect of medications that cause CNS depression, not CNS stimulation. Albuterol causes bronchodilation, not bronchoconstriction. Albuterol will cause tachycardia, not bradycardia. Test-Taking Tip: Be alert for details about what you are being asked to do. In this Question Type you are asked to select all options that apply to a given situation or client. All options likely relate to the situation, but only some of the options may relate directly to the situation.

Question 14 A client is prescribed alendronate. Which instruction should the nurse emphasize when teaching about this medication? Question 14 Answer Choices A"Take the medication with a full glass of milk two hours after meals." "Take the medication with a full glass of milk two hours after meals." B"It is recommended that you take this medication with calcium and a glass of juice." "It is recommended that you take this medication with calcium and a glass of juice." C"Be sure to take this medication on an empty stomach." "Be sure to take this medication on an empty stomach." D"You may take this medication after any meal, at the same time every day." "You may take this medication after any meal, at the same time every day." Question Explanation

Rationale Alendronate (Fosamax) is used to treat and prevent osteoporosis. It should be taken first thing in the morning with 6 to 8 ounces of plain water at least 30 minutes before other medication or food. Food and fluids (other than water) greatly decrease the absorption of this medication. The client must also be instructed to remain in the upright position for 30 minutes following the dose to facilitate passage into the stomach and minimize irritation of the esophagus.

Question 18 The nurse is caring for a client with osteoporosis who has been prescribed alendronate. When providing care, which intervention would be a priority? Question 18 Answer Choices AAdminister the alendronate 30 to 60 minutes before the client eats. Administer the alendronate 30 to 60 minutes before the client eats. BNotify the health care provider if the client reports jaw pain. Notify the health care provider if the client reports jaw pain. CEncourage the client to increase their intake of vitamin D. Encourage the client to increase their intake of vitamin D. DMonitor the client's serum calcium levels. Monitor the client's serum calcium levels. Question Explanation

Rationale Alendronate is a bisphosphonate that helps slow down bone resorption, decreasing osteoporosis. Osteonecrosis of the jaw is a rare, adverse reaction to alendronate, and jaw pain can be a symptom of this. Therefore, notifying the health care provider of the jaw pain is the priority. The other interventions are also correct for a client with osteoporosis, but are not as important as reporting the potential adverse drug effect.

Question 19 A client has been prescribed alendronate for osteoporosis. Which statements indicate that the client understands how to safely take this medication? Select all that apply. "I will notify my doctor if I experience worsening heartburn." "I will take the pill with an antacid to prevent stomach upset." "I will swallow the pill with a full glass of water." "I will stand or sit quietly for 30 minutes after taking the pill." "I will always eat breakfast before taking the pill." Question Explanation

Rationale Alendronate is a bisphosphonate used to treat osteoporosis. It can cause esophagitis or esophageal ulcers unless precautions are followed. The client must sit upright or stand for at least 30 minutes after taking the medication. The client should take the medication with a full glass of water, at least 30 minutes before eating or drinking anything or taking any other medication. Antacids will interfere with absorption and should not be taken at the same time.

1199) A thin 24-year-old woman who runs 10 miles each week asks the nurse for advice about preventing osteoporosis. Which vitamin would the nurse recommend? Vitamin E Vitamin B Vitamin D Vitamin C

Rationale All women, except those who are pregnant or lactating, should ingest between 1000 and 1300 mg of calcium daily; if the client is unable to ingest enough calcium in food, supplements of calcium and vitamin D are recommended. Vitamin C helps maintain cartilage and connective tissue integrity but does not help prevent osteoporosis. Vitamins E and B do not help prevent osteoporosis.

772) Which outcome does allopurinol produce to prevent acute gouty attacks? Promotes uric acid excretion Decreases synovial swelling Decreases uric acid production Prevents crystallization of uric acid

Rationale Allopurinol interferes with the final steps in uric acid formation by inhibiting the production of xanthine oxidase. This medication does not promote uric acid excretion. It does not affect bone density. Allopurinol has no effect on swelling of the synovial membranes. This medication prevents the synthesis of uric acid, not its crystallization.

419) Which medication would the nurse anticipate developing a teaching plan for when a client reports becoming panicked and having an irrational fear of talking in public? Buspirone Alprazolam Diazepam Lorazepam

Rationale Alprazolam (a benzodiazepine) is a short-acting anxiolytic medication used to treat those clients with panic disorders and the irrational fear of talking openly in public (agoraphobia). Buspirone, an anxiolytic medication that is different both chemically and pharmacologically from the benzodiazepines, is always administered on a scheduled basis (not on an as-needed basis) for the treatment of anxiety. Diazepam is an anxiolytic medication commonly prescribed for the treatment of anxiety but has generally been replaced by short-acting benzodiazepines. Lorazepam is an intermediate-acting anxiolytic medication used in the treatment of acutely agitated clients.

118) A primary health care provider prescribes 0.25 mg of alprazolam by mouth three times a day for a client with anxiety and physical symptoms related to work pressures. Which side effect of this medication will the nurse monitor for in this client? Drowsiness Bradycardia Agranulocytosis Tardive dyskinesia

Rationale Alprazolam, a benzodiazepine, potentiates the actions of gamma-aminobutyric acid, enhances presympathetic inhibition, and inhibits spinal polysynaptic afferent pathways. Drowsiness, dizziness, and blurred vision are common side effects. Alprazolam may cause tachycardia, not bradycardia. Agranulocytosis is usually a side effect of the antipsychotics in the phenothiazine group, not benzodiazepines. Tardive dyskinesia occurs after prolonged therapy with antipsychotic medications; alprazolam is an antianxiety medication, not an antipsychotic. Test-Taking Tip: Practicing a few relaxation techniques may prove helpful on the day of an examination. Relaxation techniques such as deep breathing, imagery, head rolling, shoulder shrugging, rotating and stretching of the neck, leg lifts, and heel lifts with feet flat on the floor can effectively reduce tension while causing little or no distraction to those around you. It is recommended that you practice one or two of these techniques intermittently to avoid becoming tense. The more anxious and tense you become, the longer it will take you to relax.

805) For which therapeutic effect will the nurse monitor the client who is prescribed alprazolam? Pain relief Decreased anxiety Reduction in dysrhythmias Reduced blood pressure

Rationale Alprazolam, an anxiolytic, promotes muscle relaxation, reduces anxiety, and facilitates rest. Possible adverse reactions to alprazolam are anger and hostility. Although drowsiness is a side effect of alprazolam caused by depression of central nervous system activity, it is not a hypnotic. Transient hypotension is a side effect of alprazolam, but this is not why it is given to an anxious client. Test-Taking Tip: Identify option components as or in. This may help you identify a wrong answer. Example: If you are being asked to identify a diet that is specific to a certain condition, your knowledge about that condition would help you choose the response (e.g., cholecystectomy = low-fat, high-protein, low-calorie diet).

Question 10 The nurse is monitoring a client who is receiving the thrombolytic agent alteplase for treatment of an acute myocardial infarction (AMI). What outcome indicates the client is receiving adequate therapy within the first few hours of treatment? Question 10 Answer Choices AReduction of ST-segment elevation on a 12-lead ECG Reduction of ST-segment elevation on a 12-lead ECG BStabilization of blood pressure Stabilization of blood pressure CAbsence of cardiac arrhythmias Absence of cardiac arrhythmias DCardiac enzymes are within normal limits Cardiac enzymes are within normal limits Question Explanation

Rationale Alteplase (a t-PA) is used in the management of AMI with ST-segment elevation (STEMI). If thrombolytic therapy was successful, a follow-up ECG will show a reduction of 50% or more in the ST segment. This indicates a return in blood flow to the injured myocardium; however, the ST segment may not return to baseline due to myocardial damage. The other responses are in: successful thrombolysis can cause a variety of cardiac arrhythmias; cardiac enzymes peak 8 hours or more after an AMI; and blood pressure may be unstable.

Question 5 A client received 40 mg of furosemide by mouth at 10 am. Which information is most important for the nurse to provide to the next nurse in the change-of-shift report? Question 5 Answer Choices AThe client lost two pounds in the last 24 hours. The client lost two pounds in the last 24 hours. BThe client is to receive another dose of furosemide at 10 pm. The client is to receive another dose of furosemide at 10 pm. CThe client's potassium level was 4.0 mEq/L prior to administration. The client's potassium level was 4.0 mEq/L prior to administration. DThe client's urine output was 1500 mL over nine hours. The client's urine output was 1500 mL over nine hours. Question Explanation

Rationale Although all of the information is important to include, a diuresis of 1,500 mL is a very large amount and could cause hypokalemia, fluid volume deficit and hypotension. Therefore, it is the most important information to provide to the nurse on the next shift.

581) A child who has nephrotic syndrome is prescribed steroid therapy. Which explanation would the nurse give the parents regarding the goal of this treatment? Prevents infection Stimulates diuresis Provides hemopoiesis Reduces blood pressure

Rationale Although the exact mechanism is unknown, steroids produce diuresis in most children with nephrotic syndrome. Steroids will not prevent infection and will mask the signs and symptoms of infection. Steroids have no effect on the production of red blood cells. Steroids do not reduce hypertension, and hypertension is not a common finding in children with nephrotic syndrome. Test-Taking Tip: Choose the best answer for questions asking for a single answer. More than one answer may be correct, but one answer may contain more information or more important information than another answer.

Question 8 The nurse is teaching a client with chronic renal failure about their medications. The client questions the purpose of taking aluminum hydroxide. How should the nurse respond? Question 8 Answer Choices A"It increases your urine output." "It increases your urine output." B"It decreases your blood's phosphate levels." "It decreases your blood's phosphate levels." C"It is taken to control gastric acid secretions." "It is taken to control gastric acid secretions." D<p>"It will reduce your blood's calcium levels."</p> <p></p> "It will reduce your blood's calcium levels." Question Explanation

Rationale Aluminum binds to phosphates that tend to accumulate in the client with chronic renal failure due to decreased filtration capacity of the kidneys. Antacids such as aluminum hydroxide are commonly used in clients with chronic renal failure to decrease serum phosphate levels. Aluminum hydroxide will not increase urine production, control gastric acid secretions or lower serum calcium levels.

742) Which mechanism of action explains how aluminum hydroxide decreases serum phosphorus? Binding with phosphorus in the intestine Promoting excretion of phosphorus Promoting excretion of excessive urinary phosphates Dissolving stones as they pass through the urinary tract

Rationale Aluminum hydroxide binds phosphorus in the intestine, preventing its absorption; this decreases serum phosphorus. Promoting excretion of phosphorus, promoting excretion of excessive urinary phosphates, and dissolving stones as they pass through the urinary tract are not actions of this medication. Test-Taking Tip: Do not select answers that contain exceptions to the general rule, controversial material, or degrading responses.

846) A client who had a myocardial infarction has runs of ventricular tachycardia. Which medication will the nurse prepare to administer? Digoxin Furosemide Amiodarone Norepinephrine

Rationale Amiodarone decreases the irritability of the ventricles by prolonging the duration of the action potential and refractory period. It is used in the treatment of ventricular dysrhythmias such as ventricular tachycardia. Digoxin slows and strengthens ventricular contractions; it will not rapidly ectopic beats. Furosemide, a diuretic, does not affect ectopic foci. Norepinephrine is a sympathomimetic and is not the medication of choice for ventricular irritability. Test-Taking Tip: You have at least a 25% chance of selecting the response in multiple-choice items. If you are uncertain about a question, eliminate the choices that you believe are wrong and then call on your knowledge, skills, and abilities to choose from the remaining responses.

946) A client's cardiac monitor indicates multiple multifocal premature ventricular complexes (PVCs). Which medication is indicated for treatment of ventricular dysrhythmias? Amiodarone Epinephrine Methyldopa Hydrochlorothiazide

Rationale Amiodarone has an antiarrhythmic action that stabilizes cell membranes of the heart, reducing cardiac excitability; it is used for acute ventricular dysrhythmias. Methyldopa is used to treat hypertension, not PVCs. Epinephrine increases the contractibility of the heart; the effect is opposite of that which is needed. Hydrochlorothiazide is a diuretic used for hypertension, not for ing multiple PVCs.

676) Which client response must the nurse monitor to determine the effectiveness of amiodarone? Absence of ischemic chest pain Decrease in cardiac dysrhythmias Improvement in fasting lipid profile Maintenance of blood pressure control

Rationale Amiodarone is a class III antidysrhythmic used for treating ventricular and supraventricular tachycardia and for conversion of atrial fibrillation. Results of fasting lipid profile are expected with antilipidemics. A degree of blood pressure control is expected with antihypertensives. Incidence of ischemic chest pain is expected with antianginal agents, such as nitrates. Test-Taking Tip: Practicing a few relaxation techniques may prove helpful on the day of an examination. Relaxation techniques such as deep breathing, imagery, head rolling, shoulder shrugging, rotating and stretching of the neck, leg lifts, and heel lifts with feet flat on the floor can effectively reduce tension while causing little or no distraction to those around you. It is recommended that you practice one or two of these techniques intermittently to avoid becoming tense. The more anxious and tense you become, the longer it will take you to relax.

1066) The cardiac monitor reveals several runs of ventricular tachycardia. Which medication is used to treat this dysrhythmia? Atropine Epinephrine Amiodarone Sodium bicarbonate

Rationale Amiodarone suppresses ventricular activity; therefore it is used for treatment of premature ventricular complexes (PVCs). It works directly on the heart tissue and slows the nerve impulses in the heart. Atropine blocks vagal stimulation; it increases the heart rate and is used for bradycardia, not PVCs. Epinephrine increases myocardial contractility and heart rate; therefore it is contraindicated in the treatment of PVCs. Sodium bicarbonate increases the serum pH level; therefore it combats metabolic acidosis.

830) A primary health care provider prescribes atenolol 20 mg by mouth four times a day. Which information is important for the nurse to include in the discharge teaching plan for this client? Drink alcoholic beverages in moderation. Avoid abruptly discontinuing the medication. Increase the medication if chest pain develops. Report a pulse rate less than 70 beats/minute.

Rationale An abrupt discontinuation of atenolol may cause an acute myocardial infarction. Alcohol is contraindicated for clients taking atenolol because it can cause additive hypotension. Clients should never increase medications without a health care provider's direction. The pulse rate can go much lower as long as the client feels well and is not dizzy.

612) The clinic nurse receives a call from the mother of an infant prescribed digoxin. The mother reports she forgot whether she gave the morning dose of digoxin. Which response by the nurse is most appropriate? 'Give the next dose immediately.' 'Wait 2 hours before giving the medication.' 'Skip this dose and give it at the next prescribed time.' 'Take the baby's pulse and give the medication if it's more than 90 beats/min.'

Rationale An additional dose may cause overdosage, leading to toxicity; it is better to skip the dose. Giving the dose without waiting may cause an overdose, which could result in toxicity. Even waiting 2 hours may cause an overdose, leading to toxicity. Taking the pulse is not a reliable method for determining a missed dose; 90 to 110 beats/min is within the expected range for this age.

707) The spouse of a client with an intracranial hemorrhage asks the nurse, 'Why aren't they administering an anticoagulant?' How will the nurse respond? 'It is not advisable because bleeding will increase.' 'If necessary, it will be started to enhance circulation.' 'If necessary, it will be started to prevent pulmonary thrombosis.' 'It is inadvisable because it masks the effects of the hemorrhage.'

Rationale An anticoagulant should not be administered to a client who is bleeding because it will interfere with clotting and will increase hemorrhage. Anticoagulants are unsafe and will not be used to enhance the circulation or prevent pulmonary thrombosis. The response 'It is inadvisable because it masks the effects of the hemorrhage' is not the reason why it is contraindicated; if given, it will increase, not mask, the effects of the hemorrhage.

610) A child undergoing prolonged steroid therapy takes on a cushingoid appearance. The nurse would expect to find which of these manifestations during further assessment? Select all that apply. One, some, or all responses may be correct. Truncal obesity Thin extremities Increased linear growth Loss of hair on the body Decreased blood pressure

Rationale An increase in appetite results in deposition of fat on the abdomen and trunk. Muscle wasting results in thin extremities. Increased excretion of calcium causes retardation of linear growth and a resulting short stature. Because of the excess production of androgens, virilization and hirsutism occur. Increased salt and water retention cause hypertension and hypernatremia.

406) Which medications would the nurse identify as being used as preanesthetic agents? Select all that apply. One, some, or all responses may be correct. Barbiturates Benzodiazepines Antiepileptic agents Atypical antipsychotics Mood stabilizing agents

Rationale Barbiturates and benzodiazepines are sedative-hypnotics that may also be used to decrease effects of anxiety in presurgical clients. Antiepileptic agents are used to treat seizure activity and manage bipolar disorder. Atypical antipsychotics are mainly used to treat bipolar disorder and psychosis. Mood stabilizing agents are used to treat bipolar disorder.

1380) A client comes in for a pregnancy test. She tells the nurse that pregnancy may have occurred because she missed her contraceptive pills for 1 week when she had the flu. Which response by the nurse is appropriate? 'That's the trouble with using contraceptive pills. People frequently forget to take them.' 'You may be correct. The effect of contraceptive pills depends on them being taken on a regular schedule.' 'Let's find out whether you really are pregnant. If you are, you may want to consider having an abortion.' 'Contraceptive pills are unpredictable. You could have become pregnant even if you had taken them regularly.'

Rationale An oral contraceptive program requires the client to take one tablet daily from the fifth day of the cycle and continue taking tablets for 20 or 21 days. Interrupting the monthly dosage program may permit release of luteinizing hormone, resulting in ovulation and possibly pregnancy. Stating that people often forget to take oral contraceptive pills is judgmental; contraceptive practice is the client's choice. It is premature to discuss abortion. Oral contraceptives that are taken on an exact schedule have a very high rate of success.

951) Which instructions about the use of nitroglycerin to prevent angina will the nurse provide to a client? 'At the point when pain first occurs, place two tablets under the tongue.' 'Place one tablet under the tongue before activity, and swallow another if pain occurs.' 'Before physical activity, place one tablet under the tongue, and repeat the dose in 5 minutes if pain occurs.' 'Place one tablet under the tongue when pain occurs and use an additional tablet after the attack to prevent recurrence.'

Rationale Anginal pain, which can be anticipated during certain activities, may be prevented by dilating the coronary arteries immediately before engaging in the activity. Generally, one tablet is administered at a time; doubling the dosage may produce severe hypotension and headache. The sublingual form of nitroglycerin is absorbed directly through the mucous membranes and should not be swallowed. When the pain is relieved, rest generally will prevent its recurrence by reducing oxygen consumption of the myocardium.

672) How would the nurse determine if a client is experiencing the therapeutic effect of valsartan? Check a lipid profile. Assess an apical pulse. Measure urinary output. Check the blood pressure.

Rationale Angiotensin II receptor blockers (ARBs) are antihypertensive medications that lower the blood pressure. ARBs do not directly affect lipid profile, apical pulse, or urinary output. Test-Taking Tip: You have at least a 25% chance of selecting the response in multiple-choice items. If you are uncertain about a question, eliminate the choices that you believe are wrong and then call on your knowledge, skills, and abilities to choose from the remaining responses.

336) An antianxiety medication is prescribed for an extremely anxious client. The client states, 'I'm afraid to take this medication because I heard it's addictive.' Which response by the nurse is most appropriate? 'This medication rarely causes dependence when the dosage is controlled.' 'You may require increases in your dosage; however, it rarely causes dependence.' 'It usually results in psychological but not physiological dependence.' 'The medication has the potential for physiological and psychological dependence.'

Rationale Antianxiety medications have the potential for physiological and psychological dependence; the nurse would teach the client about both the advantages and disadvantages of taking this medication. Physiological or psychological dependence may develop even when the dosage is controlled. Tolerance does develop and can lead to dependence. STUDY TIP: Rest is essential to the body and brain for good performance; think of it as recharging the battery. A run-down battery provides only substandard performance. For most students, it is better to spend 7 hours sleeping and 3 hours studying than to cut sleep to 6 hours and study 4 hours. The improvement in the rested mind's efficiency will balance out the difference in the time spent studying. Knowing your natural body rhythms is necessary when it comes to determining the amount of sleep needed for personal learning efficiency.

1376) A biphasic antiovulatory medication of combined progestin and estrogen is prescribed for a client. Which instruction would the nurse include when teaching about this oral contraceptive? 'Report irregular vaginal bleeding.' 'Restrict sexual activity temporarily.' 'Have regular bimonthly Pap smears.' 'Increase dietary intake of calcium.'

Rationale Antiovulatory medications suppress menstruation. Breakthrough bleeding is not expected with biphasic medications. The medication is given for 21 days, and menstrual flow does not occur during this time. Sexual activity is not restricted when one is taking oral contraceptives. There is no indication for increased frequency of Pap smears; one a year is sufficient. Increased calcium intake is not relevant to the administration of oral contraceptives.

1333) Which points would the nurse include when counseling a woman on hormone therapy? Select all that apply. One, some, or all responses may be correct. The client should use appropriate sun protection. The client should monitor any deviations in body weight. The client should take the medication on an empty stomach. The client should discontinue the medication if adverse effects occur. The client should alternate the time of day the medication is taken.

Rationale Appropriate sun protection should be used because hormones make people sensitive to sunlight. The client's body weight should be monitored during hormonal therapy because abnormal bleeding can lead to weight loss and serious complications. Hormonal medications should be taken with food to reduce gastrointestinal upset. The client should report any side effects to the primary health care provider and seek his or her advice. Hormonal therapy should never be discontinued without the knowledge of the primary health care provider. Oral medications should be administered at the same time every day to maintain the appropriate concentration of serum medication levels.

Question 10 The nurse is discharging a client who is at risk for venous thromboembolism (VTE). The client is prescribed enoxaparin. Which instruction should the nurse provide to this client? Question 10 Answer Choices A"Notify your health care provider if your stools appear tarry or black." "Notify your health care provider if your stools appear tarry or black." B"You must have your partial thromboplastin time (PTT) checked weekly." "You must have your partial thromboplastin time (PTT) checked weekly." C"You should massage the injection site for better absorption." "You should massage the injection site for better absorption." D"An intravenous (IV) catheter will be placed to administer the medication." "An intravenous (IV) catheter will be placed to administer the medication." Question Explanation

Rationale As with any anticoagulant, enoxaparin carries the risk of bleeding. Clients should be instructed to report the presence of tarry stools, bleeding gums, hematuria, ecchymosis or petechiae to their HCP. PTT monitoring is not required for this medication. This type of heparin is administered subcutaneously, not intravenously. Massaging the site will cause bruising and decrease effectiveness of the drug.

Question 13 A client is being discharged with a prescription for warfarin. The client asks "May I take aspirin with this medication? It helps my arthritis." Which response by the nurse is appropriate to address the client's concern? Question 13 Answer Choices A"Use about half the recommended dose of aspirin." "Use about half the recommended dose of aspirin." B"When you take the aspirin, do not take the warfarin that day." "When you take the aspirin, do not take the warfarin that day." C"Avoid aspirin because it can increase the bleeding effects of warfarin." "Avoid aspirin because it can increase the bleeding effects of warfarin." D"Take the warfarin in the morning and the aspirin at night." "Take the warfarin in the morning and the aspirin at night." Question Explanation

Rationale Aspirin is a salicylate, which inhibits platelet aggregation. When used in conjunction with warfarin, the risk of bleeding increases. Therefore, aspirin and warfarin should not be taken together. It is inappropriate to tell the client to not take the prescribed medication, warfarin.

719) Tissue plasminogen activator (t-PA) is to be administered to a client in the emergency department. Before beginning the infusion, which assessment is the nurse's priority? Vital signs Electrocardiogram (ECG) monitoring Signs of bleeding Level of chest pain

Rationale Assessment for bleeding is a priority because it is a contraindication for administration of thrombolytic agents; administration in the presence of bleeding can cause life-threatening hemorrhage. All the other options are important, but none pose a life-threatening contraindication to tissue plasminogen activator (t-PA) administration.

1179) Which intervention would the nurse implement for a client admitted for an exacerbation of asthma? Determine the client's emotional state. Give prescribed medications to promote bronchiolar dilation. Provide education about the effect of a family history. Encourage the client to use an incentive spirometer routinely.

Rationale Asthma involves spasms of the bronchi and bronchioles as well as increased production of mucus; this decreases the size of the lumina, interfering with inhalation and exhalation. Bronchiolar dilation will reduce airway resistance and improve the client's breathing. Although identifying and addressing a client's emotional state is important, maintaining airway and breathing are the priority. In addition, emotional stress is only one of many precipitating factors, such as allergens, temperature changes, odors, and chemicals. Although recent studies indicate a genetic correlation along with other factors that may predispose a person to the development of asthma, exploring this issue is not the priority. The use of an incentive spirometer is not helpful because of mucosal edema, bronchoconstriction, and secretions, all of which cause airway obstruction.

345) Which intervention would the nurse implement for a client admitted for an exacerbation of asthma? Determine the client's emotional state. Give prescribed medications to promote bronchiolar dilation. Provide education about the effect of a family history. Encourage the client to use an incentive spirometer routinely.

Rationale Asthma involves spasms of the bronchi and bronchioles as well as increased production of mucus; this decreases the size of the lumina, interfering with inhalation and exhalation. Bronchiolar dilation will reduce airway resistance and improve the client's breathing. Although identifying and addressing a client's emotional state is important, maintaining airway and breathing are the priority. In addition, emotional stress is only one of many precipitating factors, such as allergens, temperature changes, odors, and chemicals. Although recent studies indicate a genetic correlation along with other factors that may predispose a person to the development of asthma, exploring this issue is not the priority. The use of an incentive spirometer is not helpful because of mucosal edema, bronchoconstriction, and secretions, all of which cause airway obstruction.

818) The health care provider prescribes atenolol for a client with angina. Which potential side effect will the nurse mention when instructing the client about this medication? Headache Tachycardia Constipation Hypotension

Rationale Atenolol competitively blocks stimulation of beta-adrenergic receptors within vascular smooth muscles, which lowers the blood pressure. This medication does not cause headaches; this medication may be used to relieve vascular headaches. This medication may cause bradycardia, not tachycardia. This medication may cause diarrhea, not constipation.

Question 7 The nurse is providing discharge education to a client who will be starting daily atenolol for the treatment of hypertension. Which side effect is most important for the client to notify their health care provider about? Question 7 Answer Choices ADecreased libido Decreased libido BSlow, irregular heart rate Slow, irregular heart rate CDizziness in the morning Dizziness in the morning DDecreased exercise tolerance Decreased exercise tolerance Question Explanation

Rationale Atenolol is a Beta-1 selective adrenergic blocking agent or a "beta blocker." These medications are commonly used to treat hypertension or chronic angina. Due to their selectivity, they are the preferred medications for clients who have the comorbidities of Chronic Obstructive Pulmonary Disease (COPD). Common adverse effects often relate to the therapeutic action of the drug and include impotence, decreased libido, dizziness, decreased exercise tolerance, slowed heart rate, arrhythmias and heart failure. The client should be taught to assess their heart rate and to notify the health care provider of any changes to the heart rate or rhythm.

924) A client who has type 1 diabetes and chronic bronchitis is prescribed atenolol for the management of angina pectoris. Which clinical manifestation will alert the nurse to the fact that the client may be developing a life-threatening response to the medication? Paroxysmal nocturnal dyspnea Supraventricular tachycardia Malignant hypertension Hyperglycemia

Rationale Atenolol is associated with the adverse reactions of bradycardia, heart failure, and pulmonary edema; these are the most serious responses to atenolol and are often manifested by episodes of paroxysmal nocturnal dyspnea and orthopnea. A decreased, not increased, pulse rate is associated with atenolol so supraventricular tachycardia is not a response. Atenolol decreases, not increases, blood pressure so malignant hypertension is not a response. It also will not cause an increase in blood glucose. It may increase the hypoglycemic response to insulin, causing hypoglycemia. In addition, the medication may mask the clinical manifestations of hypoglycemia.

693) Which action describes a therapeutic effect of atenolol? Heart rate decreases Cardiac output increases Bronchospasm is relieved Pulse oximetry improves

Rationale Atenolol, a beta-blocker, slows the rate of sinoatrial (SA) node discharge and atrioventricular (AV) node conduction, thus decreasing the heart rate; it prevents angina by decreasing the cardiac workload and myocardial oxygen consumption. Cardiac output is not increased and may be decreased. Atenolol may promote bronchospasm, not relieve it. Atenolol does not directly affect gas exchange in the lungs to promote improving oxygenation.

120) A client has been robbed, beaten, and sexually assaulted. The primary health care provider prescribes 0.25 mg of alprazolam for agitation. Which event would alert the nurse to administer this medication? The client's crying increases. The client requests something to calm her. The nurse determines a need to reduce her anxiety. The primary health care provider is getting ready to perform a vaginal examination.

Rationale Because a sexual assault is a threat to the sense of control over one's life, some control should be given back to the client as soon as possible. Crying is a typical way to express emotions; the client should be told that medication is available if desired. The nurse determining a need to reduce the client's anxiety or administering the medication when the primary health care provider is getting ready to do a vaginal examination takes control away from the client. She may view these actions as an additional assault on the body, which increases feelings of vulnerability and anxiety and does not restore control.

1080) Which assessment will the nurse conduct before administering digoxin to a client? Apical heart rate Radial pulse Difference between carotid and radial pulses Difference between apical and radial pulses

Rationale Because digoxin slows the heart rate, the apical pulse should be counted for 1 minute before administration. If the apical rate is below a preset parameter (usually 60 beats/minute), digoxin should be withheld because its administration may further decrease the heart rate. Some protocols permit waiting for 1 hour and retaking the apical rate; the result determines if it is administered or if the health care provider is notified. Obtaining the radial pulse on the left side is not as accurate as an apical pulse; the client also may have an atrial dysrhythmia, which cannot be detected through a radial rate alone. Obtaining the radial pulse in both right and left arms is not as accurate as an apical pulse; the client also may have an atrial dysrhythmia, which cannot be detected through a radial rate alone. Obtaining the difference between apical and radial pulses is a pulse deficit, not a pulse rate. Test-Taking Tip: Identifying content and what is being asked about that content is critical to your choosing the response. Be alert for words in the stem of the item that are the same or similar in nature to those in one or two of the options . Example: If the item relates to and identifies stroke rehabilitation as its focus and only one of the options contains the word stroke in relation to rehabilitation, you are safe in identifying this choice as the response.

833) A client is receiving metoprolol. Which potential effect will the nurse teach the client to expect? Dizziness with strenuous activity Acceleration of the heart rate after eating a heavy meal Flushing sensations after taking the medication Pounding of the heart

Rationale Because metoprolol competes with catecholamines at beta-adrenergic receptor sites, the expected increase in the heart's rate and contractility in response to exercise does not occur. This, combined with the medication's hypotensive effect, may lead to dizziness. Metoprolol decreases the heart rate. Flushing sensations and pounding of the heart do not represent side effects of metoprolol.

907) A client is admitted to the hospital for an adrenalectomy. Before the client's replacement steroid therapy is regulated fully, the nurse will monitor the client for which complication? Hypotension Hypokalemia Hypernatremia Hyperglycemia

Rationale Because of instability of the vascular system and the lability of circulating adrenal hormones after an adrenalectomy, hypotension frequently occurs until the hormonal level is controlled by replacement therapy. Hyperglycemia is a sign of excessive adrenal hormones; after an adrenalectomy, adrenal hormones are not secreted. Sodium retention is a sign of hyperadrenalism; it does not occur after the adrenals are removed. Potassium excretion is a response to excessive adrenal hormones; after an adrenalectomy, adrenal hormones are decreased until replacement therapy is regulated. Test-Taking Tip: Practicing a few relaxation techniques may prove helpful on the day of an examination. Relaxation techniques such as deep breathing, imagery, head rolling, shoulder shrugging, rotating and stretching of the neck, leg lifts, and heel lifts with feet flat on the floor can effectively reduce tension while causing little or no distraction to those around you. It is recommended that you practice one or two of these techniques intermittently to avoid becoming tense. The more anxious and tense you become, the longer it will take you to relax.

1153) A client has a tonic-clonic seizure caused by an overdose of aspirin. Which action would the nurse take next? Check reflexes every 2 hours. Insert a urinary retention catheter. Monitor vital signs every 15 minutes. Prepare a setup for a central venous pressure (CVP) line.

Rationale Because of the lethal toxicity of an aspirin overdose, hypotensive crisis and cardiac irregularities can occur. The central nervous system is not involved at the reflex level at this time. Inserting a urinary retention catheter is not the priority at this time. CVP readings are not indicated in this situation.

1499) Which nursing assessment is important for a school-age child undergoing long-term steroid therapy? Monitoring pulse for irregularities Testing of stools for occult blood Inspection of urine for mucous threads Check of oral mucous membranes for ulcers

Rationale Because steroids decrease production of prostaglandins that have a role in protecting the stomach, gastrointestinal bleeding may occur; stools should be checked for frank and occult blood. Steroids do not cause pulse irregularities, mucus in the urine, or ulceration of mucous membranes.

Question 20 The nurse inly administers carvedilol (Coreg) to a client with an order for benztropine (Cogentin). What is the priority nursing intervention after making this medication error? Question 20 Answer Choices AComplete an incident report Complete an incident report BNotify the nurse manager Notify the nurse manager CMonitor the client's blood pressure Monitor the client's blood pressure DNotify the health care provider Notify the health care provider Question Explanation

Rationale Because the nurse mistakenly administered a beta blocker medication, the priority intervention is to monitor the client for any adverse physiological response to the given drug. Carvedilol blocks alpha1 and beta receptors in blood vessels, causing dilation and a decrease in blood pressure.

632) Which angiotensin-converting enzyme inhibitors (ACE inhibitors) are appropriate for a client with liver dysfunction? Select all that apply. One, some, or all responses may be . Ramipril Enalapril Quinapril Captopril Lisinopril

Rationale Captopril and Lisinopril are the best choices for someone with liver dysfunction because they are the only two ACE inhibitors that are not inactive in the administered form and then are metabolized to the active form once they are in the body, usually by the liver. Ramipril, Enalapril, Quinapril, and Benazepril are not good choices for the client with liver dysfunction.

Question 16 A nurse is preparing to administer morning medications to a client with heart failure. The morning lab values are: sodium 142 mEq/L (142 mmol/L), potassium 2.9 mEq/L (2.9 mmol/L), digoxin level 1.4 ng/mL. Which of the following medications should the nurse not administer until after speaking with the health care provider? Question 16 Answer Choices ASpironolactone Spironolactone BCarvedilol (Coreg) Carvedilol (Coreg) CDigoxin (Lanoxin) Digoxin (Lanoxin) DFerrous sulfate Ferrous sulfate Question Explanation

Rationale Because the potassium levels are low (normal is 3.5 to 5 mEq/L or 3.5 to 5 mmol/L), the nurse should not give the digoxin; hypokalemia can predispose a person to digoxin toxicity. The other medications can be administered. Although carvedilol can increase plasma digoxin concentration, the digoxin level is normal. Spironolactone is a potassium-sparing diuretic and because the potassium level is low, this too can be given. Ferrous sulfate does not affect the given lab values.

765) A beclomethasone inhaler would be prescribed for which purpose? Prevents atelectasis Decreases inflammation Relaxes smooth muscle in the airways Reduces bacteria in the respiratory tract

Rationale Beclomethasone reduces the inflammatory response in bronchial walls by suppression of polymorphonuclear leukocytes and fibroblasts and the reversal of capillary permeability. Beclomethasone does not prevent atelectasis. Beclomethasone does not cause smooth muscle relaxation in the airways. Beclomethasone is not an antibiotic.

473) Which nursing assessment would performed by a nurse before administering intravenous (IV) infusion of potassium chloride (KCl) 40 mEq in 100 mL of 5% dextrose and water to be infused over 2 hours? Select all that apply. One, some, or all responses may be . Urinary output Deep tendon reflexes Last bowel movement Arterial blood gas results Last serum potassium level Patency of the intravenous access

Rationale Before administering IV potassium, the urinary output must be normal. If the urine output is low, a potassium infusion may damage renal cells. The last serum potassium level should also be checked to ensure potassium replacement is appropriate. A patent IV access is essential because potassium is very irritating and painful to subcutaneous tissue. The infusion of KCL 40 mEq in 100 mL of 5% dextrose and water has no direct effect on bowel movement patterns, arterial blood gases, or deep tendon reflexes. These items are not required to be assessed before the administration of this medication.

1046) Which concern will the nurse keep in mind when a client has been taking a benzodiazepine? Rebound insomnia may occur if the medication is discontinued abruptly. Lifelong treatment is often required. Higher doses are needed to accommodate physiological changes during pregnancy. These medications have both analgesic and antidepressant properties.

Rationale Benzodiazepine often leads to tolerance and withdrawal; therefore it can cause rebound insomnia when discontinued abruptly. Benzodiazepine is not indicated for lifelong treatment; addiction is a concern. Benzodiazepine should be discontinued if the client becomes pregnant. Benzodiazepines not have analgesic, antidepressant, or antipsychotic properties.

88) Which statement about benzodiazepines requires ion? They are indicated for ethanol withdrawal. These medications increase the activity of gamma-aminobutyric acid. Benzodiazepines are the first-line medications used in chronic anxiety disorders. These medications depress activity in the brainstem.

Rationale Benzodiazepines act by decreasing the activity of gamma-aminobutyric acid, which is an inhibitory neurotransmitter. Apart from their indication in the treatment of depression, benzodiazepines are also prescribed for ethanol withdrawal, insomnia, and muscle spasms. Benzodiazepines are the first-line medications of choice in acute and chronic anxiety disorders. Benzodiazepines act by depressing activity in the brainstem and the limbic system.

265) Which finding would the nurse report to the health care provider when caring for a client prescribed temazepam at bedtime? Anxiety Drowsiness Sleep driving Morning headache

Rationale Benzodiazepines are frequently used at bedtime for treating insomnia. However, some clients taking the medications may experience the side effect of performing tasks without remembering the events. When these side effects occur at night, dangerous sleep-related behaviors, such as sleep driving or preparing and consuming meals, can occur. If a client reports these side effects, the nurse will need to report the finding to the health care provider immediately to ensure the client's safety. Anxiety, drowsiness, and morning headaches are common findings during treatment with benzodiazepines and do not need to be reported to the health care provider. Test-Taking Tip: Read every word of each question and option before responding to the item. Glossing over the questions just to get through the examination quickly can cause you to misread or misinterpret the real intent of the question.

368) Which class of medication would the nurse anticipate preparing to administer to a client with alcohol dependence admitted to a detoxification unit? Opiate Antipsychotic Antidepressant Benzodiazepine

Rationale Benzodiazepines are indicated for the treatment of alcohol withdrawal because they help prevent seizures and calm the vital signs. Opiates are not indicated for the treatment of alcohol detoxification. Antipsychotics and antidepressants are contraindicated because they can lower the seizure threshold. Test-Taking Tip: After choosing an answer, go back and reread the question stem along with your chosen answer. Does it fit ly? The choice that grammatically fits the stem and contains the information is the best choice.

54) A client is prescribed the benzodiazepine alprazolam for the management of panic attacks. Which action by the patient makes the nurse confident that the medication information discussed has been understood? The client removes the pepperoni from a pizza. The client asks for an extra bottle of flavored water to drink with dinner. The client requests a prescription for oral contraceptives before being discharged. The client states that chewable antacids may be taken to relieve heartburn.

Rationale Benzodiazepines increase the risk of congenital anomalies and so should not be taken by pregnant women. Refraining from eating pepperoni is appropriate for people taking monoamine oxidase inhibitors because tyramine needs to be strictly avoided. Appropriate hydration is critical for those taking lithium. Antacids can affect both absorption and metabolism of benzodiazepines and should be avoided.

102) Which manifestation is an adverse effect of intravenous lorazepam? Select all that apply. One, some, or all responses may be . Amnesia Drowsiness Sleep driving Blurred vision Respiratory depression

Rationale Benzodiazepines such as lorazepam have a range of side effects, many of which are related to central nervous system depression. Anterograde amnesia, drowsiness, sleep driving, blurred vision, and respiratory depression are all potential adverse effects of lorazepam.

1185) A client is prescribed cholestyramine for the treatment of type II hyperlipoproteinemia. Which vitamin would the nurse anticipate may become deficient because of this therapy? Niacin (vitamin B 3) Calciferol (vitamin D) Ascorbic acid (vitamin C) Cyanocobalamin (vitamin B 12)

Rationale Bile acid sequestrants (also known as bile acid-binding resins) bind with bile acids to form an insoluble compound that is then excreted in the feces. These medications decrease the absorption of fat-soluble vitamins (A, D, E, K). Vitamins B 3, C, and B 12 are water-soluble vitamins and are not affected by the administration of this medication. Test-Taking Tip: Calm yourself by closing your eyes, putting down your pencil (or computer mouse), and relaxing. Deep-breathe for a few minutes (or as needed, if you feel especially tense) to relax your body and to relieve tension.

324) Which vitamin would the nurse anticipate may become deficient in a client prescribed cholestyramine for the treatment of type II hyperlipoproteinemia? Niacin (vitamin B 3) Calciferol (vitamin D) Ascorbic acid (vitamin C) Cyanocobalamin (vitamin B 12)

Rationale Bile acid sequestrants (also known as bile acid-binding resins) bind with bile acids to form an insoluble compound that is then excreted in the feces. These medications decrease the absorption of fat-soluble vitamins (A, D, E, K). Vitamins B 3, C, and B 12 are water-soluble vitamins and are not affected by the administration of this medication. Test-Taking Tip: Calm yourself by closing your eyes, putting down your pencil (or computer mouse), and relaxing. Deep-breathe for a few minutes (or as needed, if you feel especially tense) to relax your body and to relieve tension.

1541) Which action by a client taking alendronate requires correction? Select all that apply. One, some, or all responses may be correct. Taking medication twice a week Taking medication before rising Taking medication with breakfast Taking medication before bedtime Taking medication with apple juice

Rationale Bisphosphonates such as alendronate have specific dosing instructions. The medication is given once a week. Clients must remain in an upright position for 30 minutes after taking the medication. Clients should refrain from eating right before they are given the medication and should take it with a full glass of water because of the risk for esophagitis. Alendronate must be administered in the morning. Clients should take the medication with water.

530) A 6-year-old child is receiving an intravenous solution of 10% glucose and mannitol to reduce cerebral edema. Which complication would the nurse monitor the child for? Overhydration Seizure activity Acute heart failure Hypovolemic shock

Rationale Both hypertonic glucose and mannitol cause diuresis; the child should be monitored for excessive fluid loss. Hypertonic glucose and mannitol will cause fluid loss, not gain. Seizure activity is not anticipated as a result of this infusion. An increased fluid volume can lead to heart failure; however, hypertonic glucose and mannitol cause fluid loss, not gain.

Question 6 A client is prescribed digoxin 0.25 mg by mouth daily. The health care provider has written a new order to give metoprolol tartrate 25 mg twice a day by mouth. In assessing the client prior to administering the medications, which finding should the nurse report to the health care provider? Question 6 Answer Choices AUrine output of 50 mL/hour Urine output of 50 mL/hour BRespiratory rate of 16 Respiratory rate of 16 CBlood pressure of 94/60 Blood pressure of 94/60 DHeart rate of 76 BPM Heart rate of 76 BPM Question Explanation

Rationale Both medications decrease the heart rate. Metoprolol (Lopressor)affects blood pressure. Therefore, the heart rate and blood pressure must be within normal range (HR 60 to 100 BPM and systolic BP greater than 100 mm Hg) in order to safely administer both medications.

386) Which nursing intervention is important when caring for clients receiving intravenous (IV) digoxin? Select all that apply. One, some, or all responses may be . Monitor the heart rate closely. Check the blood levels of digoxin. Administer the dose over 1 minute. Monitor the serum potassium level. Give the medication with other infusing medications.

Rationale Bradycardia or other dysrhythmias may occur; therefore the heart rate and rhythm should be monitored. Electrocardiogram (ECG) monitoring should be continuous. The digoxin level is checked before administration to avoid toxicity. A low serum potassium level when digoxin is administered can contribute to toxicity. Digoxin should be given over a 5-minute period through a Y-tube or three-way stopcock. There are many syringe, Y-site, and additive incompatibilities; the manufacturer recommends that digoxin not be administered with other medications.

1425) A client is prescribed an oral contraceptive. As part of teaching, the nurse plans to inform the client of the possibility of which adverse effect? Cervicitis Ovarian cysts Fibrocystic breasts Breakthrough bleeding

Rationale Breakthrough bleeding, or midcycle bleeding, commonly occurs when women start using oral contraceptives. If it persists, the dosage should be changed. There is no evidence that cervicitis, ovarian cysts, or fibrocystic breasts are related to the use of oral contraceptives. Test-Taking Tip: Do not panic while taking an exam! Panic will only increase your anxiety. Stop for a moment, close your eyes, take a few deep breaths, and resume review of the question.

Question 2 The nurse is providing discharge education to a client with moderate persistent asthma. The nurse should instruct the client to administer which medication first? Question 2 Answer Choices ABronchodilator Bronchodilator BGlucocorticoid Glucocorticoid CAnticholinergic Anticholinergic DMast cell stabilizer Mast cell stabilizer Question Explanation

Rationale Bronchodilators, such as albuterol, are beta-agonist drugs that relieve bronchospasm by relaxing the smooth muscle of the airway. These medications should be inhaled first to open the airways, which will allow the other medications to move more deeply into the lungs and increase their effectiveness.

1448) Which medication is derived from a natural source and may be prescribed for the treatment of osteoporosis? Calcitonin Raloxifene Clomiphene Bisphosphonates

Rationale Calcitonin is derived from natural sources such as fish; this medication may be prescribed to prevent osteoporosis. Raloxifene is prescribed to prevent postmenopausal osteoporosis. Clomiphene is prescribed to induce ovulation. Bisphosphonates are prescribed to treat osteoporosis; this medication is not derived from natural sources.

1203) Which supplement would the nurse instruct a client taking oral contraceptives to increase? Calcium Vitamin C Vitamin E Potassium

Rationale Oral contraceptives can affect the metabolism of certain vitamins, particularly vitamin C, and supplementation may be required. It is unnecessary to increase the intake of calcium when one is taking oral contraceptives. There is no clinical evidence linking oral contraceptives with a deficiency of vitamin E. There is no interrelationship between oral contraceptives and dietary intake of potassium.

Question 13 The nurse is reviewing prescribed medications with a client. Which information should the nurse reinforce about captopril? Question 13 Answer Choices ATake the medication with meals. Take the medication with meals. BAvoid using salt substitutes. Avoid using salt substitutes. CRestrict fluids to 1000 mL/day. Restrict fluids to 1000 mL/day. DAvoid green leafy vegetables. Avoid green leafy vegetables. Question Explanation

Rationale Captopril is an angiotensin converting enzyme (ACE) inhibitor. It reduces aldosterone secretion, thereby reducing sodium and water retention. Captopril is used to treat hypertension and heart failure. Because it can cause an accumulation of serum potassium (i.e., hyperkalemia), clients should avoid the use of salt substitutes, which often contain potassium instead of sodium chloride. The other information does not apply to captopril.

673) Captopril is prescribed for a client. Which effect would the nurse anticipate? Increased urine output Decreased anxiety Improved sleep Decreased blood pressure

Rationale Captopril is an angiotensin-converting enzyme (ACE) inhibitor antihypertensive. It does not have diuretic, sedative, or hypnotic properties. Diuretics promote fluid excretion. Sedatives reduce muscle tension and anxiety. Hypnotics promote sleep. Test-Taking Tip: Answer every question. A question without an answer is always a wrong answer, so go ahead and guess.

1497) Which medication may be useful in managing hypertension in a child with acute glomerulonephritis? Digoxin Diazepam Captopril Phenytoin

Rationale Captopril, an angiotensin-converting enzyme inhibitor antihypertensive, blocks the conversion of angiotensin I to the constrictor angiotensin II. Digoxin is not an antihypertensive; it increases the contractility and output of the heart. Diazepam is not an antihypertensive; it relaxes skeletal muscle. Phenytoin is not an antihypertensive; it is an anticonvulsant.

700) Carbidopa-levodopa is prescribed for a client with Parkinson's disease. Which instruction will the nurse include when teaching the client about this medication? 'Take this medication between meals.' 'Blood levels of the medication should be monitored weekly.' 'It can cause happy feelings followed by feelings of depression.' 'You may experience dizziness when moving from sitting to standing.'

Rationale Carbidopa-levodopa is a metabolic precursor of dopamine; it reduces sympathetic outflow by limiting vasoconstriction, which may result in orthostatic hypotension. Carbidopa-levodopa should be administered with food to minimize gastric irritation. Although periodic tests to evaluate hepatic, renal, and cardiovascular status are required for prolonged therapy, these tests aren't required on a weekly basis. Carbidopa-levodopa may produce either happiness or depression, but no established pattern of such responses exists. STUDY TIP: Becoming a nursing student automatically increases stress levels because of the complexity of the information to be learned and applied and because of new constraints on time. One way to decrease stress associated with school is to become very organized, so that assignment deadlines or tests do not come as sudden surprises. By following a consistent plan for studying and completing assignments, you can stay on top of requirements and thereby prevent added stress.

703) The nurse administers carbidopa-levodopa to a client with Parkinson's disease. Which activity describes the mechanism of action of this medication? Increase in acetylcholine production Regeneration of injured thalamic cells Improvement in myelination of neurons Replacement of a neurotransmitter in the brain

Rationale Carbidopa-levodopa is used because levodopa is the precursor of dopamine. It is converted to dopamine in the brain cells, where it is stored until needed by axon terminals; it functions as a neurotransmitter. Regeneration of injured thalamic cells is not an action of this medication; neurons do not regenerate. Increase in acetylcholine production and improvement in myelination of neurons are not actions of this medication.

1154) Which would the nurse include when teaching a client with Parkinson disease about carbidopa-levodopa? Multivitamins should be taken daily. A high-protein diet should be followed. The medication should be taken with meals. Alcohol consumption should be in moderation.

Rationale Carbidopa-levodopa should be taken with meals to reduce the nausea and vomiting that commonly are caused by this medication. Multivitamins are contraindicated; vitamins may contain pyridoxine (vitamin B 6), which diminishes the effects of levodopa. A high-protein diet is contraindicated. Sinemet contains levodopa, an amino acid that may increase blood urea nitrogen levels. Also, some proteins contain pyridoxine, which increases the peripheral metabolism of levodopa, decreasing the amount of levodopa crossing the blood-brain barrier. Moderate amounts of alcohol will antagonize the medication's effects; a rare, occasional drink is not harmful.

819) The nurse has administered sublingual nitroglycerin. Which outcome would the nurse use to determine the effectiveness of sublingual nitroglycerin? Relief of anginal pain Improved cardiac output Decreased blood pressure Ease in respiratory effort

Rationale Cardiac nitrates relax smooth muscles of the coronary arteries; they dilate and deliver more blood to heart muscle, relieving ischemic pain. Although cardiac output may improve because of improved oxygenation of the myocardium, improved cardiac output is not a basis for evaluating the effectiveness of sublingual nitroglycerin. Although dilation of blood vessels and a subsequent drop in blood pressure is a reason why intravenous (IV) nitroglycerin may be administered, decreased blood pressure is not the basis for evaluating the effectiveness of sublingual nitroglycerin, which is indicated for pain relief. Although superficial vessels dilate, lowering the blood pressure and creating a flushed appearance, dilation of superficial blood vessels is not the basis for evaluating the medication's effectiveness.

890) Which instruction will the nurse include in a teaching plan for a client taking a calcium channel blocker such as nifedipine? Select all that apply. One, some, or all responses may be . Reduce calcium intake. Report peripheral edema. Expect temporary hair loss. Avoid drinking grapefruit juice. Change to a standing position slowly.

Rationale Changing positions slowly helps reduce orthostatic hypotension. Peripheral edema may occur as a result of heart failure and must be reported. Grapefruit juice affects the metabolism of calcium channel blockers and should be avoided. Reducing calcium intake is unnecessary because calcium levels are not affected. Hair loss does not occur. STUDY TIP: When forming a study group, carefully select members for your group. Choose students who have abilities and motivation similar to your own. Look for students who have a different learning style than you. Exchange names, email addresses, and phone numbers. Plan a schedule for when and how often you will meet. Plan an agenda for each meeting. You may exchange lecture notes and discuss content for clarity or quiz one another on the material. You could also create your own practice tests or make flash cards that review key vocabulary terms.

671) Which advice would the nurse include in a teaching plan to reduce the side effects of diltiazem? Lie down after meals. Avoid dairy products in diet. Take the medication with an antacid. Change slowly from sitting to standing.

Rationale Changing positions slowly will help prevent the side effect of orthostatic hypotension. Diltiazem decreases esophageal tone, so lying down after meals can lead to acid reflux. Avoiding dairy products and taking the medication with an antacid are not necessary. Test-Taking Tip: If you are unable to answer a multiple-choice question immediately, eliminate the alternatives that you know are incorrect and proceed from that point. The same goes for a multiple-response question that requires you to choose two or more of the given alternatives. If a fill-in-the-blank question poses a problem, read the situation and essential information carefully and then formulate your response.

935) The nurse provides medication discharge instructions to a client who received a prescription for digoxin. Which statement by the client leads the nurse to conclude that the teaching was effective? 'I will avoid foods high in potassium.' 'I must increase my intake of vitamin K.' 'I should adjust the dosage according to my activities.' 'It will be important to check my pulse rate daily.'

Rationale Checking the pulse rate daily is necessary for monitoring cardiac function; digoxin slows and strengthens the heart rate. Digoxin should be withheld, and the health care provider notified, if the pulse rate falls below a predetermined rate (e.g., 60 beats per minute). Hypokalemia increases the potential for digoxin toxicity; potassium intake may need to be increased, not decreased. An increase in the intake of foods rich in vitamin K is unnecessary; digoxin does not affect vitamin K or vitamin K clotting factors. Adjusting the dosage according to activities is not an appropriate decision for the client; the health care provider should make this decision.

576) A child with nephrotic syndrome has been receiving prednisone for 1 week. Which information in the child's record indicates to the nurse that the medication has been effective? Select all that apply. One, some, or all responses may be correct. Weight loss Lower blood pH Decreased lethargy Increased urine output Decreased blood pressure

Rationale Children with nephrotic syndrome are grossly edematous. Those who have the steroid-sensitive form of nephrotic syndrome respond to corticosteroids with diuresis within 7 to 21 days after therapy is started, and the edematous weight is lost. Once the child feels better, lethargy decreases, and the activity level increases. Steroid therapy does not affect the blood pH. There is no increase in the blood pressure of a child with nephrotic syndrome and no change in blood pressure when the child improves.

91) Which vitamin deficiency may occur if cholestyramine, an anion exchange resin, to treat a client's persistent diarrhea is needed long-term? Retinol (Vitamin A) Riboflavin (Vitamin B 2) Thiamine (Vitamin B 12) Pyridoxine (Vitamin B 6)

Rationale Cholestyramine is a fat-binding agent; it binds with and interferes with all the fat-soluble vitamins (A, D, E, and K). Riboflavin is not a fat-soluble vitamin and is unaffected. Thiamine is not a fat-soluble vitamin and is unaffected. Vitamin B 6 is not a fat-soluble vitamin and is unaffected. Test-Taking Tip: Note the number of questions and the total time allotted for the test to calculate the times at which you should be halfway and three-quarters finished with the test. Look at the clock only every 10 minutes or so.

Question 7 A client who is 34 weeks gestation is diagnosed with a pulmonary embolism. Which of these medications should the nurse plan to administer? Question 7 Answer Choices AOral low-dose aspirin Oral low-dose aspirin BOral warfarin Oral warfarin CIntravenous heparin Intravenous heparin DSubcutaneous enoxaparin Subcutaneous enoxaparin Question Explanation

Rationale Clients diagnosed with pulmonary embolism (PE), whether pregnant or not, are initially treated with intravenous unfractionated heparin. Alternatively, low molecular weight heparin such as enoxaparin can be used to treat women who are pregnant. Warfarin should never be given during pregnancy due to its teratogenic effects. Although aspirin has anticoagulant properties, low-dose aspirin therapy (81 mg) is more often used prophylactically, not for the treatment of a PE.

Question 2 A client with angina has been instructed about the use of sublingual nitroglycerin. Which statement by the client indicates the need for additional teaching? Question 2 Answer Choices A"I'll call the health care provider if pain continues after three tablets five minutes apart." "I'll call the health care provider if pain continues after three tablets five minutes apart." B"I will rest briefly right after taking one tablet." "I will rest briefly right after taking one tablet." C"I understand that the medication should be kept in the dark bottle." "I understand that the medication should be kept in the dark bottle." D"I can swallow two or three tablets at once if I have severe pain." "I can swallow two or three tablets at once if I have severe pain." Question Explanation

Rationale Clients must understand that just one sublingual tablet should be taken at a time. Clients must also understand that they should rest when experiencing angina. Two or three tablets should not be used at once, even in the setting of severe pain, as this can lead to significant hypotension. The client should notify their primary healthcare provider should they not have a relief of symptoms with nitroglycerin use.

1544) Which food would the nurse instruct a client taking diltiazem to avoid? Select all that apply. One, some, or all responses may be . Alcohol Grapefruit juice Cheddar cheese Summer sausage Dark green vegetables

Rationale Clients taking calcium-channel blockers such as diltiazem would be instructed to avoid drinking grapefruit juice or eating grapefruit because it can interfere with metabolism of the medication. Clients taking acetaminophen would be instructed to avoid alcohol. Aged cheese and meat, such as sausage, should be avoided in clients taking monoamine oxidase inhibitors (MAOIs). Clients taking anticoagulants, such as warfarin, should avoid dark green vegetables.

729) The health care provider prescribes finasteride for a client with benign prostatic hyperplasia. Which information would the nurse provide to the client? Male pattern baldness can occur. Results can be expected in 4 to 6 weeks. The medication relaxes the muscles in the bladder neck. A condom should be worn during intercourse with a pregnant female.

Rationale Contact with the semen of a client taking finasteride can adversely affect a developing male fetus in a pregnant woman. Finasteride helps prevent male pattern baldness. Results may take 6 to 12 months. Finasteride is used to shrink an enlarged prostate. Other medications, such as tamsulosin, relax the muscles in the prostate and bladder neck, making it easier to urinate.

Question 18 Question 18 A client has been taking rosuvastatin for six weeks as part of a treatment plan to reduce hyperlipidemia. The clinic nurse is reviewing and reinforcing information about the medication with the client. Which statements by the client indicates an understanding about the medication? Select all that apply. "I will need to call my doctor if I have any muscle weakness or pain, especially in my legs." "I will need to come back to have my liver and kidney labs checked." "I need to be careful when I get up because this medication can make my blood pressure drop." "I add some nuts and fresh fruit to my oatmeal in the morning and I can't remember when I last ate a steak." "This medication has to be taken first thing in the morning, before I eat breakfast." Question Explanation

Rationale Clients taking rosuvastatin need to be monitored for alteration in liver function. An adverse effect of rosuvastatin is muscle pain and weakness (rhabdomyolysis). Left untreated, rhabdomyolysis can lead to renal impairment. The medication does not affect blood pressure or cause orthostatic hypotension. The client should be taught to follow a low-cholesterol diet, which includes increasing intake of whole grains and limiting intake of foods high in saturated fats, trans fats and dietary cholesterol. The medication is ordered once a day. The client can take it at any time of day, preferably at the same time of day each day, before or after eating.

1542) Which action would the nurse perform when administering fluticasone propionate to a client with asthma? Select all that apply. One, some, or all responses may be correct. Assessing heart rate and rhythm Monitoring liver function blood tests Rinsing the oral cavity with water after use Obtaining blood glucose levels before meals Giving stool softeners to prevent constipation

Rationale Clients using inhaled glucocorticoids are at an increased risk for oral candidiasis. The nurse would instruct the client to rinse the mouth with water after using the inhaler. The nurse would monitor heart rate and pattern in clients taking beta 2 agonists such as albuterol, which can lead to tachydysrhythmias. Liver function is monitored in clients taking leukotrienes such as zileuton. Blood glucose monitoring is necessary for clients taking oral and intravenous glucocorticoids. Stool softeners are given to clients taking tiotropium because of the medication's anticholinergic side effect of constipation.

1044) A client has refused prescribed cortisone. The nurse continues to administer the cortisone while evading the client's questions. When the client later discovers that cortisone continued to be administered, the client decides to sue the nurse. Which elements must be considered in a legal action? Select all that apply. One, some, or all responses may be correct. Clients have a right to refuse treatment. Nurses are required to answer clients truthfully. The health care provider should have been notified. The client had insufficient knowledge to make such a decision. Legally prescribed medications are administered despite a client's objections.

Rationale Clients who are mentally competent have the right to refuse treatment; this right takes precedence over the health care provider's prescription and the nurse must respect this right. A client's questions must always be answered truthfully. The nurse would explore the client's reasons for refusal and then notify the health care provider to plan an alternative treatment. A client's insufficient knowledge is not an acceptable reason for using deliberate deception to administer a treatment that the client has refused.

917) A client who is on long-term corticosteroid therapy after an adrenalectomy is admitted to the surgical intensive care unit after being involved in a motor vehicle crash. Which statement is an important concern for client safety? The dosage of steroids will have to be tapered down slowly to prevent acute adrenal crisis. Steroid therapy will need to be increased to avert a life-threatening crisis. Osteoporosis secondary to long-term corticosteroids increases fracture risk. The client is at risk for Cushing syndrome if taking long-term corticosteroid therapy.

Rationale Clients with adrenocorticoid insufficiency who are receiving steroid therapy require increased amounts of medication during periods of stress because they are unable to produce the excess needed by the body. With severe stress, a failure to ensure adequate corticosteroid levels can be life-threatening. Increased stress requires an increase, not a decrease, in glucocorticoids. Although osteoporosis may have contributed to fractures secondary to trauma, this does not present a current risk. Cushing syndrome is a problem with excess corticosteroid therapy, but after an adrenalectomy, the corticosteroid is given in amounts sufficient to replace what the body cannot produce.

750) Which goal is the priority for a client with asthma who has a prescription for an inhaled bronchodilator? Is able to obtain pulse oximeter readings Demonstrates use of a metered-dose inhaler Knows the health care provider's office hours Can identify triggers that may cause wheezing

Rationale Clients with asthma use metered-dose inhalers to administer medications prophylactically or during times of an asthma attack; this is an important skill to have. Home management typically includes self-monitoring of the peak expiratory flow rate rather than pulse oximetry. Although knowing the health care provider's office hours is important, it is not the priority; during a persistent asthma attack that does not respond to planned interventions, the client should go to the emergency department of the local hospital or call 911 for assistance. Although it is important to be able to identify triggers that may cause wheezing, knowing these cannot prevent all wheezing; therefore, being able to abort wheezing with a bronchodilator is the greater priority.

463) Which statement by a client who had an endarterectomy that is prescribed clopidogrel would cause the nurse to conclude that teaching was effective? "Clopidogrel will limit inflammation around my incision." "Taking this medication will help prevent further clogging of my arteries." "The medication will lower the slight fever I have had since surgery." "I will take this medication to reduce the discomfort I feel at the surgical incision."

Rationale Clopidogrel interferes with platelet aggregation, impeding the formation of thrombi. Clopidogrel is a platelet aggregation inhibitor, not an anti-inflammatory, antipyretic, or analgesic.

1196) Several minutes after a client receives an infusion of oxytocin for induction of labor, the uterine monitor indicates contractions lasting 100 seconds with a frequency of 130 seconds. Which action would the nurse take next? Discontinuing the infusion Checking the fetal heart rate Slowing the oxytocin flow rate Turning the client on her left side

Rationale Contractions lasting too long and occurring too frequently can lead to fetal hypoxia; stopping the oxytocin infusion should stop the contractions, thereby increasing oxygen flow to the fetus. The fetal heart rate should be monitored, but this is not the priority. Oxytocin will continue to promote uterine contractions; this is unsafe because the prolonged, frequent contractions decrease oxygen flow to the fetus. Turning the client on her left side will promote placental perfusion, but it is not the priority at this time.

898) A client who had a femoropopliteal bypass graft is receiving clopidogrel postoperatively. Which instruction will the nurse teach the client related to the medication? Eliminate starches and red meats from the diet. Eat more roughage if constipation occurs. Report any occurrence of multiple bruises. Take the medication on an empty stomach.

Rationale Clopidogrel is a platelet aggregation inhibitor that decreases the probability of clots forming where the graft was placed, but it also increases bleeding tendencies when the dosage is excessive. Clopidogrel does not interact with starches or red meats, which are permitted in the diet. Diarrhea, not constipation, is more likely to occur with clopidogrel. Clopidogrel should be taken with food to decrease the side effects of gastric discomfort, diarrhea, and gastrointestinal bleeding. Test-Taking Tip: Prepare for exams when and where you are most alert and able to concentrate. If you are most alert at night, study at night. If you are most alert at 2 AM, study in the early morning hours. Study where you can focus your attention and avoid distractions. This may be in the library or in a quiet corner of your home. The key point is to keep on doing which is working for you. If you are distracted or falling asleep, you may want to change when and where you are studying.

685) For the client taking clopidogrel, the nurse will monitor for which adverse effect? Nausea Epistaxis Chest pain Elevated temperature

Rationale Clopidogrel is a platelet aggregation inhibitor; therefore bleeding can occur as an adverse effect. The high vascularity of the nose, combined with its susceptibility to trauma (e.g., sneezing, nose blowing), makes it a frequent site of hemorrhage. Nausea, chest pain, and elevated temperature are not associated with anticoagulant therapy.

Question 8 A client at risk for a stroke has been prescribed clopidogrel. Which information is most important for the nurse to reinforce with the client? Question 8 Answer Choices A"You must take the medication on an empty stomach." "You must take the medication on an empty stomach." B"If you miss a dose, take a double dose the next day." "If you miss a dose, take a double dose the next day." C"You must stop the medication a week before your surgery." "You must stop the medication a week before your surgery." D"You must have your lab tests checked weekly." "You must have your lab tests checked weekly." Question Explanation

Rationale Clopidogrel is an oral antiplatelet drug with similar effects to aspirin. The drug is taken for secondary prevention of myocardial infarction, ischemic stroke and other vascular events. Clopidogrel prevents platelet aggregation. Like all other antiplatelet drugs, clopidogrel poses a risk of serious bleeding. Clopidogrel should be discontinued 5 to 7 days before elective surgery. The drug's effects begin two hours after the first dose and plateau after 3 to 7 days of treatment. Platelet function and bleeding time return to baseline 7 to 10 days after the last dose. It can be taken with or without food. No weekly lab tests are required with clopidogrel. Clients should not be instructed to double up when missing a dose.

1031) A client with a seizure disorder is receiving phenytoin and phenobarbital. Which client statement indicates that the instructions regarding the medications are understood? 'I will not have any seizures with these medications.' 'These medicines must be continued to prevent falls and injury.' 'Stopping the medications can cause continuous seizures and I may die.' 'By my staying on the medicines I will prevent postseizure confusion.'

Rationale Combination therapy suggests that this client has seizures that are difficult to control. Sudden withdrawal of any antiepileptic medication can cause onset of frequent seizures or even status epilepticus. Death can occur if seizures are continuous due to lack of adequate oxygenation and cardiac irregularities. It is important to take medication as prescribed to lessen the frequency of seizures; there is no guarantee that seizures will stop. Medication may or may not eliminate the seizures; stress may precipitate a seizure. Antiepileptic medications are not prescribed to prevent falls and injury and the added central nervous system (CNS) depression increases fall risk. Although seizures may occur while the client is taking the medications, the medications do not stop postseizure confusion.

1435) A woman questions the nurse about the effectiveness of oral contraceptives. Which important factor about the effectiveness of oral contraceptives would the nurse include in the response? User motivation Simplicity of use Reliability record Identified risk factors

Rationale Conception will not be prevented unless the user is motivated to use the method correctly and consistently. No matter how simple, the method must be used consistently. Reliability record is not relevant if the method is not used correctly and consistently by the woman. Risk factors have little influence on the effectiveness of the contraceptive method.

1103) Which complication will occur as the result of administering sildenafil to a client who takes isosorbide dinitrate? Constipation Protracted vomiting Respiratory distress Severe hypotension

Rationale Concurrent use of sildenafil and a nitrate, which causes vasodilation, may result in severe, potentially fatal hypotension. Protracted vomiting and respiratory distress are not medication interaction effects associated with concurrent use of sildenafil and a nitrate. Sildenafil may cause diarrhea; adding a nitrate will not cause constipation. STUDY TIP: Becoming a nursing student automatically increases stress levels because of the complexity of the information to be learned and applied and because of new constraints on time. One way to decrease stress associated with school is to become very organized so that assignment deadlines or tests do not come as sudden surprises. By following a consistent plan for studying and completing assignments, you can stay on top of requirements and thereby prevent added stress.

630) Which lifestyle advice does the nurse give to a client when oral digoxin therapy is initiated? Select all that apply. One, some, or all responses may be . Bran can decrease digoxin absorption. Digoxin should not be taken with hawthorn supplements. Ginseng may cause a dangerous increase in digoxin levels in the blood. St. John's Wort can increase digoxin levels in the blood. Medications that lower serum potassium or magnesium can cause digoxin toxicity.

Rationale Consuming large amounts of bran can decrease the absorption of digoxin. Hawthorn may potentiate the effects of digoxin and should be avoided. Ginseng might increase levels of digoxin. St. John's Wort can reduce levels of digoxin in the blood. Test-Taking Tip: The following are crucial requisites for doing well on the NCLEX exam: (1) A sound understanding of the subject; (2) The ability to follow explicitly the directions given at the beginning of the test; (3) The ability to comprehend what is read; (4) The patience to read each question and set of options carefully before deciding how to answer the question; (5) The ability to use the computer ly to record answers; (6) The determination to do well; (7) A degree of confidence.

704) A health care provider prescribes dexamethasone for a client with head trauma. The nurse recognizes that it reduces swelling in the brain by which process? Acts as a hyperosmotic diuretic Increases resistance to infection Reduces the inflammatory response of tissues Decreases the formation of cerebrospinal fluid

Rationale Corticosteroids act to decrease inflammation, which decreases edema. Dexamethasone is an anti-inflammatory agent, not a diuretic. Resistance to infection is decreased, not increased, with a corticosteroid. The client's problem is not with increased cerebrospinal fluid.

209) Which symptom would the nurse expect to decrease in response to corticosteroid therapy prescribed for a client with multiple sclerosis? Emotional lability Muscular contractions Pain in the extremities Visual impairment

Rationale Corticosteroids decrease the inflammatory process around the optic nerve, thus improving vision; visual impairment is the most common physiological manifestation of multiple sclerosis. Steroids are associated with increased emotional lability. Steroids are not effective in easing muscle contractions. Pain in the extremities is not common unless spasms are present; steroids do not relieve spasms.

792) A client receiving corticosteroid therapy states, 'I have difficulty controlling my temper, which is so unlike me, and I don't know why this is happening.' How will the nurse respond? Tell the client it is nothing to worry about. Reassure that everyone does this at times. Instruct the client to attempt to avoid situations that cause irritation. Inquire about mood swings.

Rationale Corticosteroids increase the excitability of the central nervous system, which can cause labile emotions manifested as euphoria and excitability or depression. Telling the client it is nothing to worry about or that it is normal denies the value of the client's statement and offers false reassurance. The client has already stated the problem and does not know why this is happening. Instructing the client to attempt to avoid situations that cause irritation is impractical because the mood swings may occur without an overt cause.

Question 11 A client is started on long-term corticosteroid therapy for an autoimmune disorder. Which statement by the client indicates the need for more teaching by the nurse? Question 11 Answer Choices A"For 1 week each month I will stop taking the medication." "For 1 week each month I will stop taking the medication." B"I will keep a record of my weight each week." "I will keep a record of my weight each week." C"The medication needs to be taken with food." "The medication needs to be taken with food." D"I will be sure to eat foods that are high in potassium." "I will be sure to eat foods that are high in potassium." Question Explanation

Rationale Corticosteroids should never be stopped abruptly, they should always be weaned. To suddenly stop this medication may result in a sudden drop in the blood pressure from a loss in fluid volume associated with adrenal crisis. Clients should be warned not to abruptly stop taking the medication. Corticosteroids can lower the amount of potassium in the body so the client should eat more potassium rich foods. Weight gain is an expected effect of corticosteroid therapy. Clients should regularly keep track of their weight. Generally, corticosteroid medications are taken with breakfast.

912) When a client is receiving dexamethasone for adrenocortical insufficiency, which action would the nurse take to monitor for an adverse effect of the medication? Auscultate for bowel sounds. Assess deep tendon reflexes. Culture respiratory secretions. Measure blood glucose levels.

Rationale Corticosteroids, such as dexamethasone, have a hyperglycemic effect, and blood glucose levels should be monitored routinely. Assessing bowel sounds is unnecessary; corticosteroids are not known to precipitate cessation of gastrointestinal activity. Although corticosteroids may increase the risk of developing an infection, routine culturing of respiratory secretions is unnecessary. Culturing respiratory secretions becomes necessary when the client exhibits adaptations of a respiratory infection. Monitoring deep tendon reflexes is required when administering magnesium sulfate, not dexamethasone.

Question 4 A nurse is reviewing the INR results for caring for a client who had a cerebral vascular accident and is receiving prescribed warfarin. The nurse notes the INR is 5.2. Which finding requires priority follow-up? Question 4 Answer Choices AGum bleeding Gum bleeding BGeneralized weakness Generalized weakness CPharyngitis Pharyngitis DAnorexia Anorexia Question Explanation

Rationale Coumadin is an anticoagulant. The normal range of the INR for a client who is taking warfarin is 1 to 2.5 times normal, or 2-3. This INR level is elevated indicating the blood is taking longer to clot and presents a risk of internal bleeding. Generalized weakness post CVA is a normal finding. A sore throat (pharyngitis) and loss of appetite (anorexia) do not pose a serious risk at this time.

1338) A registered nurse teaches a nursing student about caring for a client prescribed estradiol to treat low estrogen levels. Which statement by the student indicates to the nurse a need for additional learning? 'I should apply the emulsion once a day on the thighs.' 'I should avoid covering the medication with clothing after it is dried.' 'I should educate the client about the pharmacokinetic effects of estradiol.' 'I should advise the client to avoid applying sunscreen at the same time as the medication intake.'

Rationale Covering the medication with clothing after it is dried helps prevent the transfer of the medication to other individuals. The nurse would instruct the client to apply the emulsion once a day on the thighs. The nurse would educate the client about the pharmacokinetic properties of the medication to ensure the medication's safe and effective administration. The nurse would advise the client to not apply sunscreen products at the same time because this action may reduce the absorption of estradiol. Test-Taking Tip: Identifying content and what is being asked about that content is critical to your choosing the correct response. Be alert for words in the stem of the item that are the same or similar in nature to those in one or two of the options.

709) Diazepam is administered to the client with status epilepticus. In addition to decreasing central neuronal activity, which effect would the nurse anticipate? Relaxing of peripheral muscles Decreased heart rate Dilation of airways Hypertension

Rationale Diazepam is a benzodiazepine indicated for treatment of anxiety, muscle spasms, and seizures. Peripheral muscles may relax as a result of the antispasmodic effects. Diazepam does not slow the heart rate. Diazepam does not dilate the bronchial airways and may cause bronchoconstriction, though this is uncommon. Diazepam may cause hypotension, not hypertension.

1546) The client with congestive heart failure is receiving furosemide 80 mg once daily. Which data collection assessment would be performed to evaluate medication effectiveness? Select all that apply. One, some, or all responses may be . Daily weight Intake and output Monitor for edema Daily pulse oximetry Auscultate breath sounds

Rationale Daily weight at the same time, on the same scale, and in the same clothing is important as it is an indication of fluid gains or losses. The nurse would also record daily intake and output and report intake exceeding output. The nurse would monitor for peripheral edema and document the findings. It is important to obtain and record vital signs and daily pulse oximetry as improving results relate to effectiveness of furosemide. The nurse would also auscultate breath sounds, look for jugular venous distension, and report abnormal data. Test-Taking Tip: If you are unable to answer a multiple-choice question immediately, eliminate the alternatives that you know are in and proceed from that point. The same goes for a multiple-response question that requires you to choose two or more of the given alternatives. If a fill-in-the-blank question poses a problem, read the situation and essential information carefully and then formulate your response.

42) Which data collection assessment would be performed to evaluate the effectiveness of furosemide administered to a client with congestive heart failure? Select all that apply. One, some, or all responses may be . Daily weight Intake and output Monitor for edema Daily pulse oximetry Auscultate breath sounds

Rationale Daily weight at the same time, on the same scale, and in the same clothing is important as it is an indication of fluid gains or losses. The nurse would also record daily intake and output and report intake exceeding output. The nurse would monitor for peripheral edema and document the findings. It is important to obtain and record vital signs and daily pulse oximetry as improving results relate to effectiveness of furosemide. The nurse would also auscultate breath sounds, look for jugular venous distension, and report abnormal data. Test-Taking Tip: If you are unable to answer a multiple-choice question immediately, eliminate the alternatives that you know are in and proceed from that point. The same goes for a multiple-response question that requires you to choose two or more of the given alternatives. If a fill-in-the-blank question poses a problem, read the situation and essential information carefully and then formulate your response.

783) A client is admitted to the emergency department after experiencing a seizure. Which action would the nurse take first? Ask the emergency provider for a prophylactic anticonvulsant. Obtain a history of seizure type and incidence. Ask the client to remove any dentures and eyeglasses. Observe the client for increased restlessness and agitation.

Rationale Data collection is an essential first step for a client with a seizure disorder; it should always include a history of the seizures (e.g., type and incidence). Because different seizure medicines are used to control different seizure types, it is important to determine the type before treating. Although dentures and eyeglasses may be removed during a seizure, the client's normal routines should be respected. Increased restlessness may be evidence of the prodromal phase of a seizure in some individuals, but signs and symptoms vary so widely that the client's history should be obtained.

734) Which assessment would be brought to the health care provider's attention before administration of intravenous potassium chloride? Progressively worsening muscle weakness Poor tissue turgor with tenting Urinary output of 200 mL during the previous 8 hours Oral fluid intake of 300 mL during the previous 12 hours

Rationale Decreased urinary output may result in the retention of potassium, causing hyperkalemia. Progressively worsening muscle weakness is a manifestation of hypokalemia, which is the reason for prescribing the potassium. Reporting poor tissue turgor with tenting is unnecessary; this may indicate dehydration, which is probably the +Rationale for the fluid prescribed. Reporting an oral fluid intake of 300 mL during the previous 12 hours is unnecessary; this can precipitate dehydration or can compound an existing dehydration, which can be treated with appropriate hydration.

644) Which complication is an adverse effect of cortisone therapy? Hypoglycemia Severe anorexia Anaphylactic shock Behavioral changes

Rationale Development of mood swings and psychosis is possible during long-term therapy with glucocorticoids because of fluid and electrolyte alterations. Hypoglycemia, severe anorexia, and anaphylactic shock are not responses to long-term glucocorticoid therapy.

749) A client is receiving dexamethasone to treat acute exacerbation of asthma. For which side effect would the nurse monitor the client? Hyperkalemia Liver dysfunction Orthostatic hypotension Increased blood glucose

Rationale Dexamethasone increases gluconeogenesis, which may cause hyperglycemia. Hypokalemia, not hyperkalemia, is a side effect. Liver dysfunction is not a side effect. Hypertension, not hypotension, is a side effect. Test-Taking Tip: Answer the question that is asked. Read the situation and the question carefully, looking for key words or phrases. Do not read anything into the question or apply what you did in a similar situation during one of your clinical experiences. Think of each question as being an ideal, yet realistic, situation.

1063) Dexamethasone has been prescribed for a client after a craniotomy for a brain tumor. Which physiological response is responsible for this medication's therapeutic effect? Reduced cell growth Reduced cerebral edema Increased renal reabsorption Increased response to sedation

Rationale Dexamethasone is a corticosteroid with anti-inflammatory effects, which will reduce cerebral edema. Dexamethasone will not keep the tumor from growing; it will reduce fluid content and therefore cell size, not the number of cells. Dexamethasone does not promote fluid reabsorption, which is undesirable because it increases fluid retention and therefore cerebral edema. Dexamethasone does not promote sedation; sedation is not desired because it may mask the client's adaptations to the craniotomy. Test-Taking Tip: Identifying content and what is being asked about that content is critical to your choosing the correct response. Be alert for words in the stem of the item that are the same or similar in nature to those in one or two of the options . Example: If the item relates to and identifies stroke rehabilitation as its focus and only one of the options contains the word stroke in relation to rehabilitation, you are safe in identifying this choice as the correct response.

1503) An 18-month-old toddler is being treated with intravenous diazepam every 4 hours for generalized tetanus. Which response to the medication would the nurse anticipate? Control of hypertonicity and prevention of seizures Control of laryngospasms and neck and jaw rigidity Prevention of excess oxygen and caloric expenditure Prevention of restlessness and resistance to assisted ventilation

Rationale Diazepam is commonly used to manage generalized muscular spasms. Laryngospasm and nuchal rigidity are responses to the exotoxin and are treated with tetanus immune globulin. Diazepam is not administered to decrease the metabolic rate. Pancuronium bromide, an acetylcholine antagonist, is given to children who do not respond to sedatives and muscle relaxants and resist ventilatory assistance.

Question 14 The nurse is caring for a client diagnosed with heart failure who will begin treatment with digoxin. Which therapeutic effect would the nurse expect to find after administering this medication? Question 14 Answer Choices ADecreased chest pain with decreased blood pressure Decreased chest pain with decreased blood pressure BIncreased heart rate with increased respirations Increased heart rate with increased respirations CImproved respiratory status with increased urinary output Improved respiratory status with increased urinary output DDiaphoresis with decreased urinary output Diaphoresis with decreased urinary output Question Explanation

Rationale Digoxin (Lanoxin), a cardiac glycoside, is used in clients with heart failure to slow and strengthen the heartbeat. As cardiac output is improved, renal perfusion is improved and urinary output increases. The other findings are related to adverse, not therapeutic, effects related to digoxin or are not typically seen at all with digoxin.

1537) A child being treated with cardiac medications developed vomiting, bradycardia, anorexia, and dysrhythmias. The nurse understands which medication toxicity is responsible for these symptoms? Digoxin Nesiritide Dobutamine Spironolactone

Rationale Digoxin helps improve pumping efficacy of the heart, but an overdose can cause toxicity leading to nausea, vomiting, bradycardia, anorexia, and dysrhythmias. The side effects of nesiritide may include effects such as headache, insomnia, and hypotension. Dobutamine does not cause nausea or vomiting but may cause hypertension and hypotension. Spironolactone may cause edema. Test-Taking Tip: You have at least a 25% chance of selecting the response in multiple choice items. If you are uncertain about a question, eliminate the choices you believe are wrong and then call on your knowledge, skills, and abilities to choose from the remaining responses.

1062) A client with heart failure is to receive digoxin. Which therapeutic effect is associated with this medication? Reduces edema Increases cardiac conduction Increases rate of ventricular contractions Slows and strengthens cardiac contractions

Rationale Digoxin improves cardiac function by increasing the strength of myocardial contractions (positive inotropic effect) and, by altering the electrophysiological properties of the heart, slows the heart rate (negative chronotropic effect). Digoxin increases the strength of the contractions but decreases the heart rate. Although a reduction in edema may result from the increased blood supply to the kidneys, it is not the reason for administering digoxin. Digoxin decreases, not increases, cardiac impulses through the conduction system of the heart. Test-Taking Tip: A psychological technique used to boost your test-taking confidence is to look into a mirror whenever you pass one and say out loud, 'I know the material, and I'll do well on the test.' Try it; many students have found that it works because it reduces test anxiety.

922) A client is given a loading dose of digoxin and placed on a maintenance dose of digoxin 0.25 mg by mouth daily. Which responses would the nurse expect the client to exhibit when a therapeutic effect of digoxin is achieved? Resolution of heart failure Decreased anginal episodes Conversion of atrial fibrillation Decreased blood pressure

Rationale Digoxin improves cardiac output to improve heart failure. Digoxin is not an antianginal medication; if it decreases angina as a result of controlling heart failure, it is a secondary effect. Digoxin may be given to control a rapid ventricular response to atrial fibrillation, but it does not convert the rhythm. Digoxin has a negligible effect on blood pressure; therefore it is not an antihypertensive medication.

886) A client with left ventricular heart failure and supraventricular tachycardia is prescribed digoxin 0.25 mg daily. Which changes would the nurse expect to find if this medication is therapeutically effective? Select all that apply. One, some, or all responses may be . Diuresis Tachycardia Decreased edema Decreased pulse rate Reduced heart murmur Jugular vein distention

Rationale Digoxin increases kidney perfusion, which results in urine formation and diuresis. The urine output increases because of improved cardiac output and kidney perfusion, resulting in a reduction in edema. Because of digoxin's inotropic and chronotropic effects, the heart rate will decrease. Digoxin increases the force of contractions (inotropic effect) and decreases the heart rate (chronotropic effect). Digoxin does not affect a heart murmur. Jugular vein distention is a specific sign of right ventricular heart failure; it is treated with diuretics to reduce the intravascular volume and venous pressure.

Question 11 The nurse is preparing to administer digoxin to a client with recurring atrial fibrillation. Which laboratory value should be of highest concern for the nurse? Question 11 Answer Choices AHemoglobin 9.4 g/dL Hemoglobin 9.4 g/dL BSerum potassium 3.1 mEq/L Serum potassium 3.1 mEq/L CSerum creatinine 1.9 mg/dL Serum creatinine 1.9 mg/dL DB-type natriuretic peptide 140 pg/mL B-type natriuretic peptide 140 pg/mL Question Explanation

Rationale Digoxin is a cardiac glycoside used to treat atrial dysrhythmias and heart failure. Because digoxin competes with potassium ions, digoxin should not be given when the client's potassium level is below normal range. Giving digoxin to a client with hypokalemia can cause digoxin toxicity and life-threatening cardiac dysrhythmias. Although all of the lab values are outside of normal range, the low potassium level (normal range 3.5-5.0 mEq/L) should be of highest concern for the client at this time. The nurse should hold the digoxin and notify the health care provider.

842) A client has been given a prescription for furosemide 40 mg every day in conjunction with digoxin. Which concern would prompt the nurse to ask the health care provider about potassium supplements? Digoxin causes significant potassium depletion. The liver destroys potassium as digoxin is detoxified. Lasix requires adequate serum potassium to promote diuresis. Digoxin toxicity occurs rapidly in the presence of hypokalemia.

Rationale Furosemide promotes potassium excretion, and low potassium (hypokalemia) increases cardiac excitability. Digoxin is more likely to cause dysrhythmias when potassium is low. Digoxin does not affect potassium excretion. Furosemide causes potassium excretion. Potassium is excreted by the kidneys, not destroyed by the liver. Furosemide causes diuresis and consequent potassium loss regardless of the serum potassium level.

Question 5 A client is prescribed furosemide and digoxin for heart failure. The nurse should monitor the client for which potential adverse drug effect? Question 5 Answer Choices APulmonary hypertension Pulmonary hypertension BAcute arterial occlusion Acute arterial occlusion CAcute kidney injury Acute kidney injury DCardiac dysrhythmias Cardiac dysrhythmias Question Explanation

Rationale Digoxin is a cardiac glycoside, or positive inotrope that increases myocardial contractility. By increasing contractile force, digoxin can increase cardiac output in clients with heart failure (HF). Furosemide is a potassium-wasting (loop) diuretic, prescribed to prevent fluid overload in clients with HF. Clients who take furosemide are at risk for developing hypokalemia. Potassium ions compete with digoxin and a low potassium level can cause digoxin toxicity, leading to lethal cardiac dysrhythmias. Therefore, it is imperative that potassium levels be kept within normal range (3.5 to 5 mEq/L) while taking digoxin.

Question 2 The client diagnosed with heart failure is prescribed oral digoxin. What is the priority nursing assessment for this medication? Question 2 Answer Choices AMonitor serum electrolytes and creatinine Monitor serum electrolytes and creatinine BMeasure apical pulse prior to administration Measure apical pulse prior to administration CMaintain accurate intake and output ratios Maintain accurate intake and output ratios DMonitor blood pressure every 4 hours Monitor blood pressure every 4 hours Question Explanation

Rationale Digoxin is an antiarrhythmic and an inotropic drug. It works to increase cardiac output and slow the heart rate. The priority assessment is to measure the apical pulse for one minute prior to administering the drug. The nurse will withhold the dose and notify the healthcare provider if the apical rate is less than 60 beats per minute. Intake and output ratios and daily weights should be monitored for a client in heart failure, but this is not the priority assessment. Impaired renal function may contribute to drug toxicity, which is why the nurse will monitor serum electrolytes, creatinine and BUN; the nurse should also monitor serum digoxin levels.

Question 4 The nurse is reviewing medication instructions with a client who is taking digoxin. The nurse should reinforce to the client to report which of the following side effects? Question 4 Answer Choices ARash, dyspnea, edema Rash, dyspnea, edema BNausea, vomiting, fatigue Nausea, vomiting, fatigue CHunger, dizziness, diaphoresis Hunger, dizziness, diaphoresis DPolyuria, thirst, dry skin Polyuria, thirst, dry skin Question Explanation

Rationale Digoxin is considered an antidysrhythmic and inotrope, that is used to treat atrial dysrhythmias and congestive heart failure. The medication produces a positive inotropic effect, prolongs the refractory period and slows conduction through the sinoatrial (SA) and atrioventricular (AV) nodes. Overall, digoxin increases cardiac output and slows the heart rate. The effects of digoxin produce many side effects and clients who take digoxin are at risk for digoxin toxicity. Because digoxin improves cardiac output, side effects of the medication would not include dyspnea or edema. Rashes are also not considered a side effect of digoxin. Common manifestations of digoxin toxicity include nausea, vomiting and fatigue. Hunger, dizziness and diaphoresis, together, are not considered side effects of digoxin. Although dizziness could occur with another side effect of digoxin, such as bradycardia. Polyuria, thirst and dry skin are not considered side effects of digoxin.

Question 1 A nurse is educating a client about digoxin toxicity. Which statement made by the client indicates that more teaching is needed? Question 1 Answer Choices A"High levels of digoxin can cause vision changes." "High levels of digoxin can cause vision changes." Your Answer B"I must report a strong pulse of 62 beats per minute to the health care provider." "I must report a strong pulse of 62 beats per minute to the health care provider." Answer C"I should report nausea and vomiting lasting more than a few days." "I should report nausea and vomiting lasting more than a few days." D"I will let the health care provider know if my pulse feels uneven or misses beats." "I will let the health care provider know if my pulse feels uneven or misses beats." Question Explanation

Rationale Digoxin is used to increase the strength of heart contraction. The expected effect of digoxin use is a slower, strong pulse. The client should be instructed to check their pulse prior to taking this medication and to note the rate and if the rhythm is irregular. If the heart rate is less than 60 or greater than 100 the client should not take the medication. Therefore, a strong pulse of 62 is a therapeutic effect of this medication and would not warrant a call to health care provider. This needs to be clarified with the client. The other statements indicate understanding of the medication. Digoxin toxicity would cause irregular pulse, loss of appetite, nausea, vomiting and vision changes. The client should be alert to these clinical manifestations and call the health care provider if they experience any of these changes.

849) Digoxin is prescribed for a client. Which therapeutic effect of digoxin would the nurse expect? Decreased cardiac output Decreased stroke volume of the heart Increased contractile force of the myocardium Increased electrical conduction through the atrioventricular (AV) node

Rationale Digoxin produces a positive inotropic effect that increases the strength of myocardial contractions and thus cardiac output. The positive inotropic effect of digoxin increases, not decreases, cardiac output. Digoxin increases the strength of myocardial contractions (positive inotropic effect) and slows the heart rate (negative chronotropic effect); these effects increase the stroke volume of the heart. Digoxin decreases the refractory period of the AV node and decreases conduction through the sinoatrial (SA) and AV nodes. Test-Taking Tip: Do not select answers that contain exceptions to the general rule, controversial material, or responses that appear to be degrading.

682) When teaching a client about digoxin, which symptom will the nurse include as a reason to withhold the digoxin? Fatigue Yellow vision Persistent hiccups Increased urinary output

Rationale Digoxin toxicity is a common and dangerous effect. Visual disturbances, most notably yellow vision, may be evidence of digoxin toxicity. Fatigue is not a toxic effect of digoxin. Persistent hiccups are not related to digoxin toxicity. An increased urinary output is not a sign of digoxin toxicity; it may be a sign of a therapeutic response to the medication and an improved cardiac output.

1502) Which vaccine is used to prevent human papilloma virus infection? Varivax RotaTeq Gardasil Hepatitis A vaccine

Rationale Gardasil is a quadrivalent vaccine used to prevent genital cancers and warts caused by human papilloma virus. Varivax is associated with protection from the varicella virus; this vaccine is sometimes given in combination with the MMR vaccine. The RotaTeq vaccine is used to vaccinate against a rotavirus infection. The hepatitis A vaccine is used to protect against the hepatitis A virus.

Question 4 The nurse is preparing to administer digoxin to a client admitted for acute decompensated heart failure. Which action is the priority before giving this drug? Question 4 Answer Choices AMonitor oxygen saturation on room air Monitor oxygen saturation on room air BAssess the client's weight and compare to the baseline Assess the client's weight and compare to the baseline CAuscultate the lungs for crackles in the bases Auscultate the lungs for crackles in the bases DAssess the apical pulse for a full minute Assess the apical pulse for a full minute Question Explanation

Rationale Digoxin, a cardiac glycoside, is used to slow the heart rate and increase the force of contraction. The priority for the nurse is to count the client's apical pulse for one full minute, even if the heart rhythm is regular. Typically, when the pulse is less than 60, digoxin should not be given. The other actions are also appropriate assessments for a client with heart failure. However, they are not the priority when administering digoxin.

Question 13 A client recently diagnosed with heart failure has been prescribed digoxin and furosemide. Which of the following foods should the nurse teach the client to eat at least one serving a day? Question 13 Answer Choices ABlueberries Blueberries BWheat cereal Wheat cereal CTomato juice Tomato juice DPear nectar Pear nectar Question Explanation

Rationale Digoxin, an antiarrhythmic, and furosemide, a diuretic, are commonly prescribed for clients with heart failure. A common side effect for furosemide is depletion of potassium. Of the food choices, tomato juice is the highest in potassium. To reduce the risk of potassium depletion, the client should be encouraged to drink at least 1/2 cup of tomato juice every day which is about 400 mg of potassium. The other choices are low in potassium which would be recommended for clients diagnosed with chronic renal failure.

982) A client with supraventricular tachycardia (SVT) has a heart rate of 170 beats/minute. After treatment with diltiazem, which assessment indicates to the nurse that the diltiazem is effective? Increased urine output Blood pressure of 90/60 mm Hg Heart rate of 98 beats/minute No longer complaining of heart palpations

Rationale Diltiazem hydrochloride's purpose is to slow down the heart rate. SVT has a heart rate of 150 to 250 beats/minute. A heart rate of 110 beats/minute indicates that the diltiazem hydrochloride is having the desired effect. Hypotension is a side effect of diltiazem hydrochloride, not a desired effect. Heart palpations are experienced by some with various dysrhythmias. A decreased sensation of heart palpations is a positive finding but is not present in all clients. Increased urine output may occur over a period of time because of the increased ventricular filling time but would not occur until after the heart rate had stabilized. Test-Taking Tip: If you are unable to answer a multiple-choice question immediately, eliminate the alternatives that you know are in and proceed from that point. The same goes for a multiple-response question that requires you to choose two or more of the given alternatives. If a fill-in-the-blank question poses a problem, read the situation and essential information carefully and then formulate your response.

Question 19 The nurse is evaluating the plan of care for a client with benign prostatic hyperplasia (BPH). For which prescribed medication should the nurse notify the health care provider (HCP)? Question 19 Answer Choices ADiphenhydramine Diphenhydramine BFinasteride Finasteride CTerazosin Terazosin DMetoprolol Metoprolol Question Explanation

Rationale Diphenhydramine is a first generation histamine1 receptor antagonist or antihistamine, commonly used for relief from symptoms of mild to moderate allergic disorders. H1 blockers have anticholinergic effects or atropine-like responses and can cause urinary hesitancy or retention. A client with BPH is already at risk for urinary retention and should not receive an antihistamine such as diphenhydramine without clarification from the HCP first. Metoprolol is a beta blocker, which does not affect the bladder. Finasteride and terazosin are drugs commonly used to treat BPH.

726) A health care provider prescribes a diuretic for a client with hypertension. Which mechanism of action explains how diuretics reduce blood pressure? They facilitate vasodilation. They promotes smooth muscle relaxation. They reduce the circulating blood volume. They block the sympathetic nervous system.

Rationale Diuretics decrease blood volume by blocking sodium reabsorption in the renal tubules, thus promoting fluid loss and reducing arterial pressure. Direct relaxation of arteriolar smooth muscle is accomplished by vasodilators, not diuretics. Vasodilators, not diuretics, act on vascular smooth muscle. Medications that act on the nervous system, not diuretics, inhibit sympathetic vasoconstriction.

887) A client is receiving hydrochlorothiazide. Which physiological alteration will the nurse monitor to best determine the effectiveness of the client's hydrochlorothiazide therapy? Blood pressure Decreasing edema Serum potassium level Urine specific gravity

Rationale Diuretics promote urinary excretion, which reduces the volume of fluid in the intravascular compartment, thus lowering blood pressure. The measure of blood pressure is the best determination of effectiveness because it is a direct measure of the desired outcome. A reduction in edema reflects effectiveness; however, multiple physiological processes, including venous competence, gravity, and disuse, maintain a significant degree of edema even when the diuretic is optimally effective. A lowered potassium level would indirectly indicate that the medication is working; however, this does not provide a good measure of effectiveness. Although specific gravity decreases with increased urinary output, and thus would demonstrate that the medication is working, it is not a direct measure of the desired outcome. A measure of the reduction in intravascular pressure is preferable.

485) Which is an appropriate nursing action when caring for a client taking benazepril for hypertension? Assess for dizziness. Assess for dark, tarry stools. Administer the medication after meals. Monitor the electroencephalogram (EEG).

Rationale Dizziness may occur during the first few weeks of therapy until the client adapts physiologically to the medication. Dark, tarry stools are not a side effect of benazepril. Administering the medication after meals is unnecessary; however, if nausea occurs, the medication may be taken with food or at bedtime. The blood pressure should be monitored before and after administration. An EEG is unnecessary. Cardiac monitoring may be instituted because of possible dysrhythmias.

193) Donepezil is prescribed for a client who has mild dementia of the Alzheimer type. Which information would the nurse include when discussing this medication with the client and family? Fluids should be limited to 4 large glasses per day. Constipation should be reported to the primary health care provider immediately. Blood tests that reflect liver function will be performed routinely. The client's medication dosage may be self-adjusted according to the client's response.

Rationale Donepezil may affect the liver because alanine aminotransferase (ALT) is found predominantly in the liver; most ALT increases indicate hepatocellular disease. Clients taking this medication should have regular liver function tests and report light stools and jaundice to the primary health care provider. Fluids should not be limited, because a side effect of donepezil is constipation. A side effect of constipation is expected; therefore, fluids, high-fiber foods, and exercise should be recommended to help keep the stools soft. The client should not increase or decrease the dosage abruptly; donepezil should be taken exactly as prescribed. Test-Taking Tip: As you answer each question, write a few words about why you think that answer is ; in other words, justify why you selected that answer. If an answer you provide is a guess, mark the question to identify it. This will permit you to recognize areas that need further review. It will also help you see how your 'guessing' can be. Remember: On the licensure examination, you must answer each question before moving on to the next question.

366) The nurse teaches a client's family about the administration of donepezil for treatment of dementia of the Alzheimer type. Which side effect identified by the caregiver indicates to the nurse that further teaching is needed? Nausea Dizziness Headache Constipation

Rationale Donepezil, a cholinesterase inhibitor, may cause nausea, vomiting, increased salivation, diarrhea, and involuntary defecation related to the increase in gastrointestinal secretions and activity caused by parasympathetic nervous stimulation; it does not cause constipation. Common side effects of donepezil include anorexia, nausea, and vomiting that result from stimulation of the parasympathetic nervous system. Dizziness and headache are common side effects of donepezil that result from central nervous system cholinergic effects.

561) The mother of an infant recently prescribed phenobarbital for seizures calls the pediatric clinic and states that the infant is lethargic and sleeps for long periods. Which response by the nurse is most appropriate? 'There's a medication that will prevent this problem.' 'This means that your baby's dosage needs to be adjusted.' 'This is a temporary response to the medication; it usually stops after a few weeks.' 'Many infants experience the same problem, but your baby needs the medication.'

Rationale Drowsiness is frequently a side effect of barbiturate therapy because it depresses the central nervous system; the infant will adapt to this over time. Stimulants are not routinely administered because they counteract the desired effect of seizure reduction. The dosage does not need adjustment; this response demonstrates little understanding of barbiturate therapy. The mother's concern is with her own baby; the medication's side effects should be explained.

865) A client is prescribed oral disopyramide to manage a ventricular dysrhythmia. Which side effects will the nurse include when teaching the client about this medication? Select all that apply. One, some, or all responses may be . Dry mouth Rhinorrhea Constipation Hyperglycemia Stress incontinence

Rationale Dry mouth occurs because of its anticholinergic properties. Constipation is a side effect of this nonnitrate antidysrhythmic because of its anticholinergic properties. A thin, watery discharge from nose (rhinorrhea) does not occur with this medication because of its anticholinergic properties. Hypoglycemia, not hyperglycemia, may occur. Urinary hesitancy and retention, rather than stress incontinence, occur. STUDY TIP: Develop a realistic plan of study. Do not set rigid, unrealistic goals.

311) Which action would the nurse implement for a client who has overdosed on barbiturates? Select all that apply. One, some, or all responses may be . Apply ice packs. Maintain airway. Administer naloxone. Give activated charcoal. Give intravenous fluids.

Rationale Due to the adverse effect of respiratory depression, the nurse will administer oxygen and maintain the airway. Any unabsorbed medication can be removed by giving the client activated charcoal. To eliminate any barbiturate that has been absorbed, the nurse would administer intravenous fluids to alkalinize the urine. Naloxone is given to reverse the effects of opioid overdoses. The nurse will provide warm blankets, not ice packs, to clients who have overdosed.

848) Amlodipine is prescribed for a client with hypertension. Which response to the medication will the nurse instruct the client to report to the health care provider? Blurred vision Dizziness on rising Difficulty breathing Excessive urination

Rationale Dyspnea may indicate development of pulmonary edema, which is a life-threatening condition. Blurred vision may occur in some people, but it is not life-threatening. Dizziness on rising and excessive urination are common side effects of this medication that are not life-threatening. Test-Taking Tip: You have at least a 25% chance of selecting the response in multiple-choice items. If you are uncertain about a question, eliminate the choices that you believe are wrong and then call on your knowledge, skills, and abilities to choose from the remaining responses.

1184) A client is receiving furosemide to relieve edema. The nurse will monitor the client for which responses? Select all that apply. One, some, or all responses may be . Weight loss Negative nitrogen balance Increased urine specific gravity Excessive loss of potassium ions Pronounced retention of sodium ions

Rationale Each liter of fluid weighs 2.2 pounds (1 kilogram). Assessing weight loss is an objective measure of the effectiveness of the medication. Furosemide is a potent diuretic that is used to provide rapid diuresis in clients with pulmonary edema; it acts in the loop of Henle and causes depletion of electrolytes, such as potassium and sodium. A negative nitrogen balance does not affect protein metabolism. With increased fluid loss, the specific gravity is likely to be lowered. Furosemide inhibits the reabsorption of sodium.

326) Which response would a nurse monitor for when a client is receiving furosemide to relieve edema? Select all that apply. One, some, or all responses may be . Weight loss Negative nitrogen balance Increased urine specific gravity Excessive loss of potassium ions Pronounced retention of sodium ions

Rationale Each liter of fluid weighs 2.2 pounds (1 kilogram). Assessing weight loss is an objective measure of the effectiveness of the medication. Furosemide is a potent diuretic that is used to provide rapid diuresis in clients with pulmonary edema; it acts in the loop of Henle and causes depletion of electrolytes, such as potassium and sodium. A negative nitrogen balance would not be monitored. Furosemide does not affect protein metabolism. With increased fluid loss, the specific gravity is typically decreased.

1241) A client is taking an estrogen-progestin oral contraceptive. Which adverse effects from the contraceptive would the nurse teach the client to report to the primary health care provider? Select all that apply. One, some, or all responses may be correct. Dizziness Chest pain Bloating Nausea Calf tenderness Breast tenderness

Rationale Early side effects of oral contraceptives include bloating, nausea, and breast tenderness. Although they may be bothersome enough to lead to discontinuation of the contraceptive, these side effects usually subside in several months. Dizziness is not a common side effect and should be reported to the provider. Contraceptives have been associated with thrombophlebitis; clinical manifestations of thrombophlebitis include calf tenderness and redness and heat over the affected area. If the clot travels, it could present as a pulmonary embolism, so chest pain should be reported as well. STUDY TIP: An adverse effect is any unexpected or unintended response to a therapeutic dose of a drug. An adverse effect usually requires changing dosage, stopping the drug, or administering an antidote to terminate drug action. Adverse effects, sometimes called side effects, fall into two groups: predictable effects occur as the result of known, dose-related pharmacological effects, whereas unpredictable effects are unrelated to the drug's characteristics. Reactions in this second group are often the result of something distinctive to the individual, such as drug allergies, hypersensitivities, or idiosyncratic reactions. Adverse effects can range in severity from those that are merely annoying to those that are life threatening.

225) Which information will the nurse include when teaching a client about potassium chloride effervescent tablets? Chew the tablet completely. Take the medication with food. Take the medication at bedtime. Use warm water to dissolve the tablet.

Rationale Eating food when taking the medication will decrease gastrointestinal irritation. Side effects of this medication include abdominal cramps, diarrhea, and ulceration of the small intestine. Chewing the tablet completely will cause oral mucosal irritation and is not the way the medication should be administered. Taking the medication at bedtime increases the possibility of mucosal irritation because the gastrointestinal tract is empty during the night. The tablet should be dissolved in cold water or juice to make it more palatable. STUDY TIP: Enhance your time-management abilities by designing a study program that best suits your needs and current daily routines by considering issues such as the following: (1) amount of time needed; (2) amount of time available; (3) best time to study; (4) time for emergencies and relaxation.

1197) The nurse explains to a group of nursing students how magnesium sulfate treats severe preeclampsia. Which response by a student indicates effective teaching? 'It blocks neuromuscular transmissions.' 'It provides analgesia for the client in labor.' 'It increases the quantity of acetylcholine.' 'It improves the quality of uterine contractions.'

Rationale Eclamptic seizures may be prevented with the administration of intravenous magnesium sulfate, which is a central nervous system depressant that blocks neuromuscular transmissions. Although magnesium sulfate is a neuromuscular sedative that relaxes smooth muscle and decreases blood pressure, it is not considered an analgesic and is not given for that purpose. Magnesium sulfate decreases, not increases, the quantity of acetylcholine. Improved quality of uterine contractions is not associated with magnesium sulfate administration. STUDY TIP: Determine whether you are a 'lark' or an 'owl.' Larks, day people, do best getting up early and studying during daylight hours. Owls, night people, are more alert after dark and can remain up late at night studying, catching up on needed sleep during daylight hours. It is better to work with natural biorhythms than to try to conform to an arbitrary schedule. You will absorb material more quickly and retain it better if you use your most alert periods of each day for study. Of course, it is necessary to work around class and clinical schedules. Owls should attempt to register in afternoon or evening lectures and clinical sections; larks do better with morning lectures and day clinical sections.

926) A health care provider prescribes enalapril for a client. Which nursing action is important? Assess the client for hypokalemia. Monitor for adverse effects on renal function. Monitor the client's blood pressure during therapy. Assess the client for hypoglycemia.

Rationale Enalapril is an antihypertensive. A lowering of the client's blood pressure reflects a therapeutic response and needs to be monitored regularly. The client may be at risk for hyperkalemia, not hypokalemia. Enalapril has renal protective effects rather than adverse renal effects. Although an antihypertensive of choice for some clients with diabetes, it does not affect glucose levels.

652) The nurse prepares discharge instructions for a client who will take enalapril for hypertension. Which instruction would the nurse include in the client's teaching? 'Change to a standing position slowly.' 'This may color your urine green.' 'The medication may cause a sore throat for the first few days.' 'Schedule blood tests weekly for the first 2 months.'

Rationale Enalapril is classified as an angiotensin-converting enzyme (ACE) inhibitor. Like many antihypertensives, it can cause orthostatic hypotension. Clients should be advised to change positions slowly to minimize this effect. This medication does not alter the color of urine or cause a sore throat the first few days of treatment. Presently, there are no guidelines that suggest blood tests are required weekly for the first 2 months.

Question 16 Question 16 The nurse is caring for a client who is taking leuprolide for endometriosis. The nurse should monitor the client for which side effects? Select all that apply. Amenorrhea Hot flashes Vaginal dryness Emotional lability Increased fertility Anorexia Question Explanation

Rationale Endometriosis is a benign gynecologic condition in which endometrial tissue grows outside of the uterus. It can be controlled, but not cured, by drug therapy. Drugs commonly used include oral contraceptives and GnRH agonists such leuprolide and nafarelin. GnRH drugs result in amenorrhea (absence of menstruation) and other symptoms that mimic menopause such as hot flashes, vaginal dryness and emotional lability. Anorexia (lack of appetite) is not a side effect usually seen with leuprolide. Leuprolide does not increase fertility.

730) After the nurse provides education about hydrochlorothiazide, the client will agree to notify the health care provider regarding the development of which symptom? Insomnia Nasal congestion Increased thirst Generalized weakness

Rationale Generalized weakness is a symptom of significant hypokalemia, which may be a sequela of diuretic therapy. Insomnia is not known to be related to hypokalemia or hydrochlorothiazide therapy. Although a stuffy nose is unrelated to hydrochlorothiazide therapy, it can occur with other antihypertensive medications. Increased thirst is associated with hypernatremia. Because this medication increases excretion of water and sodium in addition to potassium and chloride, hyponatremia, not hypernatremia, may occur.

839) A health care provider prescribes enoxaparin 30 mg subcutaneously daily. Which measure would the nurse take when administering this medication? Push over 2 minutes. Administer in the abdomen. Massage site after administration. Remove air pocket from prepackaged syringe before administration.

Rationale Enoxaparin specifically targets blood clots throughout the body and carries a lower risk of hemorrhage than that associated with the medications heparin and warfarin. Enoxaparin is administered once a day through a subcutaneous injection site around the naval. Enoxaparin should be injected into the fatty tissue only, which is why the abdomen is the recommended injection site. Avoid administering in a muscle. Manufacturer recommendations indicate the air pocket from prepackaged syringes not be removed before administration. Rubbing the site is contraindicated, because it can cause bruising. There are no recommendations to push this subcutaneous medication over 2 minutes. Test-Taking Tip: Avoid taking a wild guess at an answer. However, should you feel insecure about a question, eliminate the alternatives that you believe are definitely incorrect, and reread the information given to make sure you understand the intent of the question. This approach increases your chances of randomly selecting the or getting a clearer understanding of what is being asked. Although there is no penalty for guessing, the subsequent question will be based, to an extent, on the response you give to the question at hand; that is, if you answer a question incorrectly, the computer will adapt the next question accordingly based on your knowledge and skill performance on the examination up to that point.

980) Enoxaparin 40 mg subcutaneously daily is prescribed for a client who had abdominal surgery. The nurse explains that the medication is given for which purpose? To control postoperative fever To provide a constant source of mild analgesia To limit the postsurgical inflammatory response To provide prophylaxis against postoperative thrombus formation

Rationale Enoxaparin, a low-molecular-weight heparin, prevents the conversion of fibrinogen to fibrin and of prothrombin to thrombin by enhancing the inhibitory effects of antithrombin III. Enoxaparin is not an antipyretic. Enoxaparin is not an analgesic. Enoxaparin is not an anti-inflammatory medication. Test-Taking Tip: Multiple-choice questions can be challenging, because students think that they will recognize the right answer when they see it or that the right answer will somehow stand out from the other choices. This is a dangerous misconception. The more carefully the question is constructed, the more each of the choices will seem like the correct response.

1334) A 25-year-old woman on estrogen therapy has a history of smoking. Which complication would the nurse anticipate in the client? Osteoporosis Hypermenorrhea Endometrial cancer Pulmonary embolism

Rationale Estrogen therapy increases the risk of pulmonary embolism in clients who have a history of smoking because the medication affects blood circulation and hemostasis. Osteoporosis may be caused by reduced bone density observed in postmenopausal woman. Hypermenorrhea (excessive menstrual bleeding) is treated with estrogen therapy. Endometrial cancer is a complication of estrogen therapy seen in postmenopausal woman. Test-Taking Tip: Identifying content and what is being asked about that content is critical to your choosing the correct response. Be alert for words in the stem of the item that are the same or similar in nature to those in one or two of the options.

778) Long-term corticosteroid therapy has been initiated for a client with myasthenia gravis who experiences inadequate symptomatic control with pyridostigmine bromide. Which action is important for the nurse to take? Request a high-sodium diet. Establish protective isolation. Decrease the client's total daily fluid intake. Monitor the client for an exacerbation of symptoms.

Rationale Exacerbation of myasthenia gravis may occur temporarily at the beginning of steroid therapy, causing respiratory embarrassment and dysphagia. Increasing sodium intake is contraindicated because corticosteroids increase sodium retention. Although clients should avoid contact with persons who have upper respiratory infections, protective isolation (neutropenic precautions) is not required. Decreasing total daily fluid intake is unnecessary; adequate fluid intake should be maintained.

856) A client with chronic liver disease reports, 'My gums have been bleeding spontaneously.' The nurse identifies small hemorrhagic lesions on the client's face. The nurse concludes that the client needs which additional supplement? Bile salts Folic acid Vitamin A Vitamin K

Rationale Fat-soluble vitamin K is essential for synthesis of prothrombin by the liver; a lack results in hypoprothrombinemia, inadequate coagulation, and hemorrhage. Although cirrhosis may interfere with production of bile, which contains the bilirubin needed for optimum absorption of vitamin K, the best and quickest manner to counteract the bleeding is to provide vitamin K intramuscularly. Folic acid is a coenzyme with vitamins B 12 and C in the formation of nucleic acids and heme; thus a deficiency may lead to anemia, not bleeding. Vitamin A deficiency contributes to the development of polyneuritis and beriberi, not hemorrhage.

872) Which instructions will the nurse include in the teaching plan for a client with hyperlipidemia who is being discharged with a prescription for cholestyramine? 'Increase your intake of fiber and fluid.' 'Take the medication before you go to bed.' 'Check your pulse before taking the medication.' 'Contact your health care provider if your skin turns yellow.'

Rationale Fiber and fluids help prevent the most common adverse effect of constipation and its complication, fecal impaction. The medication should be taken with meals. The pulse is not affected. Cholestyramine binds bile in the intestine; therefore it reduces the incidence of jaundice.

Question 3 A nurse is teaching a client with asthma about the correct use of the fluticasone inhaler. Which statement, if made by the client, would indicate that the teaching was effective? Question 3 Answer Choices A"The inhaler can be used when I feel short of breath." "The inhaler can be used when I feel short of breath." B"If I forget a dose, I will double the next dose." "If I forget a dose, I will double the next dose." C"I should rinse my mouth after using the inhaler." "I should rinse my mouth after using the inhaler." D"I should not use a spacer with my inhaler." "I should not use a spacer with my inhaler." Question Explanation

Rationale Fluticasone is an inhaled corticosteroid used to prevent asthma attacks. After using the inhaler, the client should rinse away any residue in the mouth to reduce the risk of an oral fungal infection. Fluticasone is not a bronchodilator and should not be used as needed for shortness of breath. The client should not double the dose of this medication and should use a spacer with this inhaler.

Question 2 The nurse is reinforcing the correct use of a metered-dose inhaler (MDI) for a client newly-diagnosed with asthma. The client asks, "how will I know the canister is empty?" What is the best response by the nurse? Question 2 Answer Choices A"Contact your pharmacy to find out when to obtain a refill." "Contact your pharmacy to find out when to obtain a refill." B"Drop the canister in water to observe if it floats." "Drop the canister in water to observe if it floats." C"Count the number of doses as the inhaler is used." "Count the number of doses as the inhaler is used." D"Shake the canister and listen for any fluid movement." "Shake the canister and listen for any fluid movement." Question Explanation

Rationale Floating an MDI in water, or shaking it to listen for fluid movement to determine how much medication is left, is not recommended. MDIs that count down the number of remaining doses are available, however, these mechanisms are not always accurate. Therefore, it is best to calculate how long the inhaler will last by dividing the number of doses in the container by the number of doses the client takes per day. For example, a client who needs to take two puffs of albuterol, four times a day, will take a total of eight puffs per day. The MDI contains a total of 200 puffs. Divide 200/8 = 25 days. The inhaler in this example will last 25 days. To ensure that the client does not run out of medication, the client should obtain a refill at least 7 to 10 days before it runs out. The pharmacy would not be able to determine if the canister is empty.

115) Which response to fludrocortisone will the nurse teach a client with adrenal insufficiency to report? Select all that apply. One, some, or all responses may be correct. Edema Rapid weight gain Fatigue in the afternoon Unpredictable changes in mood Increased frequency of urination

Rationale Fludrocortisone has a strong effect on sodium retention by the kidneys, which leads to fluid retention, causing edema and weight gain. Fatigue may occur with adrenal insufficiency and is not related to cortisone therapy. Unpredictable changes in mood commonly occur but are not as serious a threat as fluid retention. Fluid retention, and thus decreased urination, may occur. Test-Taking Tip: The computerized NCLEX exam is an individualized testing experience in which the computer chooses your next question based on the ability and competency you have demonstrated on previous questions. The minimum number of questions will be 75 and the maximum 265. You must answer each question before the computer will present the next question, and you cannot go back to any previously answered questions. Remember that you do not have to answer all of the questions correctly to pass.

364) Which medication acts as an antidote to benzodiazepine? Zolpidem Temazepam Suvorexant Flumazenil

Rationale Flumazenil is a benzodiazepine receptor antagonist and is used to reverse conscious sedation induced by benzodiazepines. Zolpidem is a nonbenzodiapepine, sedative used to treat insomnia. Temazepam and is a benzodiazepine. Suvorexant is in a class of medications called orexin receptor antagonists that works by blocking the action of a certain natural substance in the brain that causes wakefulness.

157) Which medication would the nurse expect to administer to actively reverse the overdose sedative effects of benzodiazepines? Lithium Flumazenil Methadone Chlorpromazine

Rationale Flumazenil is the medication of choice in the management of overdose when a benzodiazepine is the only agent ingested by a client not at risk for seizure activity. Flumazenil competitively inhibits activity at benzodiazepine recognition sites on gamma-aminobutyric acid-benzodiazepine receptor complexes. Lithium is used in the treatment of mood disorders. Methadone is used for narcotic addiction withdrawal. Chlorpromazine is contraindicated in the presence of central nervous system depressants.

1513) Sildenafil is prescribed for a man with erectile dysfunction. Which side effects of this medication would the nurse mention in teaching? Select all that apply. One, some, or all responses may be correct. Flushing Headache Dyspepsia Constipation Hypertension

Rationale Flushing is a common central nervous system response to sildenafil. Headache is a common central nervous system response to sildenafil. Dyspepsia is a common gastrointestinal response to sildenafil. Diarrhea, not constipation, is a common gastrointestinal response to sildenafil. Hypotension, not hypertension, is a cardiovascular response to sildenafil. It should not be taken with antihypertensives and nitrates because medication interactions can precipitate cardiovascular collapse. Test-Taking Tip: A psychological technique used to boost your test-taking confidence is to look into a mirror whenever you pass one and say out loud, 'I know the material, and I'll do well on the test.' Try it; many students have found that it works because it reduces 'test anxiety.'

37) Sildenafil is prescribed for a man with erectile dysfunction. Which side effects of this medication would the nurse mention in teaching? Select all that apply. One, some, or all responses may be correct. Flushing Headache Dyspepsia Constipation Hypertension

Rationale Flushing is a common central nervous system response to sildenafil. Headache is a common central nervous system response to sildenafil. Dyspepsia is a common gastrointestinal response to sildenafil. Diarrhea, not constipation, is a common gastrointestinal response to sildenafil. Hypotension, not hypertension, is a cardiovascular response to sildenafil. It should not be taken with antihypertensives and nitrates because medication interactions can precipitate cardiovascular collapse. Test-Taking Tip: A psychological technique used to boost your test-taking confidence is to look into a mirror whenever you pass one and say out loud, 'I know the material, and I'll do well on the test.' Try it; many students have found that it works because it reduces 'test anxiety.'

524) A child is prescribed fluticasone after an acute asthma attack. Which instruction would the nurse give the family about the administration of this medication? 'Fluticasone needs to be taken with food or milk.' 'Fluticasone is primarily used to treat acute asthma attacks.' 'The child should suck on hard candy to help relieve dry mouth.' 'Watch for white patches in the mouth and report to the health care provider.'

Rationale Fluticasone is a steroid commonly administered by way of inhalation for long-term control of asthma symptoms. Oral thrush is a side effect that manifests as white patches. Fluticasone is administered via inhalation so food or milk is not needed before administration. Dry mouth is not a side effect of fluticasone.

21) A client who has a long history of medication and alcohol abuse mentions taking ginkgo biloba. Which condition is this client taking ginkgo biloba to treat? Insomnia Depression Memory impairment Anxiety and nervousness

Rationale Ginkgo biloba is an herb used to treat age-related memory impairment and dementia. It has not been shown to be effective in treating insomnia, depression, or anxiety. Test-Taking Tip: Do not read too much into the question or worry that it is a "trick." If you have nursing experience, ask yourself how a classmate who is inexperienced would answer this question from only the information provided in the textbooks or given in the lectures.

1180) The nurse teaches the client about appropriate foods to consume when taking warfarin. The nurse evaluates that the client needs further teaching when the client makes which statement? 'Eggs provide a good source of iron, which is needed to prevent anemia.' 'Yellow vegetables are high in vitamin A and should be included in the diet.' 'Dark green leafy vegetables are high in vitamin K, so I should eat them more often.' 'Milk and other high-calcium dairy products are necessary to counteract bone density loss.'

Rationale Foods high in vitamin K should be limited to the usual amounts eaten by the client when establishing the prothrombin time/international normalized ratio because vitamin K is part of the body's blood-clotting mechanism and will counter the effects of warfarin if eaten in excess. Foods containing protein and iron are permitted because they are unrelated to blood clotting. Foods containing vitamin A are permitted because vitamin A is unrelated to blood clotting. Foods containing calcium are permitted because calcium is unrelated to blood clotting.

337) Which statement about appropriate foods to consume when taking warfarin would indicate that the client needs further teaching? "Eggs provide a good source of iron, which is needed to prevent anemia." "Yellow vegetables are high in vitamin A and should be included in the diet." "Dark green leafy vegetables are high in vitamin K, so I should eat them more often." "Milk and other high-calcium dairy products are necessary to counteract bone density loss."

Rationale Foods high in vitamin K should be limited to the usual amounts eaten by the client when establishing the prothrombin time/international normalized ratio because vitamin K is part of the body's blood-clotting mechanism and will counter the effects of warfarin if eaten in excess. Foods containing protein and iron are permitted because they are unrelated to blood clotting. Foods containing vitamin A are permitted because vitamin A is unrelated to blood clotting. Foods containing calcium are permitted because calcium is unrelated to blood clotting.

Question 3 The nurse is reviewing the prothrombin time results for a client who is taking warfarin. The nurse notes the value is 20 seconds. What is an appropriate nursing action? Question 3 Answer Choices ARecognize that this is a therapeutic level. Recognize that this is a therapeutic level. BAssess for bleeding gums or IV sites. Assess for bleeding gums or IV sites. CNotify the primary health care provider immediately. Notify the primary health care provider immediately. DObserve the client for hematoma development. Observe the client for hematoma development. Question Explanation

Rationale For the client on warfarin therapy, this prothrombin level is within the therapeutic range. Therapeutic levels for warfarin are usually one and a half to two times the normal level.

1068) The nurse is preparing to discharge a client who presented to the emergency room for an acute asthma attack. The nurse notes that upon discharge the health care provider has prescribed theophylline 300 mg orally to be taken daily at 9:00 AM. The nurse will teach the client to take the medication on which schedule? One hour before or 2 hours after eating At bedtime At the specific time prescribed Daily until symptoms are gone

Rationale For theophylline to be effective, therapeutic serum levels must be maintained by taking the medication at the prescribed time. If the medication is not taken at the prescribed time, the level may drop below the therapeutic range. The medication will not be effective if it drops below the therapeutic range. Theophylline should be given after a meal and with a full glass of water to decrease gastric irritability. Giving it 2 hours after a meal (on an empty stomach) can result in gastric discomfort. It should not be taken at night, because it can cause central nervous system stimulation resulting in insomnia, restlessness, irritability, etc. Theophylline is used for long-term medication therapy. STUDY TIP: Begin studying by setting goals. Make sure they are realistic. A goal of scoring 100% on all exams is not realistic, but scoring an 85% may be a better goal.

1519) An adolescent is prescribed phenylephrine nasal spray. The nurse would determine teaching has been effective when the adolescent identifies which complication that may occur if the spray is used more frequently or longer than recommended? Tinnitus Nasal polyps Bleeding tendencies Increased nasal congestion

Rationale Frequent and continued use of phenylephrine can cause rebound congestion of mucous membranes. Tinnitus is not a side effect of phenylephrine nasal spray; however, hypotension, tachycardia, and tingling of the extremities may occur. Nasal polyps may be associated with allergies but are unrelated to phenylephrine nasal spray. Bleeding tendencies are unrelated to the use of phenylephrine nasal spray.

Question 7 The nurse is caring for a client who has been taking furosemide for the past week. Which manifestation would indicate that the client may be experiencing a negative side effect? Question 7 Answer Choices AEdema of the ankles Edema of the ankles BGastric irritability Gastric irritability CWeight gain of five pounds Weight gain of five pounds DDecreased appetite Decreased appetite Question Explanation

Rationale Furosemide (Lasix) causes a loss of potassium if a supplement is not taken. Findings of hypokalemia include anorexia, fatigue, nausea, decreased gastrointestinal motility, muscle weakness and dysrhythmias.

1101) A health care provider prescribes furosemide for a client with hypervolemia. The nurse recalls that furosemide exerts its effects in which part of the renal system? Distal tubule Collecting duct Glomerulus of the nephron Loop of Henle

Rationale Furosemide acts in the ascending limb of the loop of Henle in the kidney. Thiazides act in the distal tubule in the kidney. Potassium-sparing diuretics act in the collecting duct in the kidney. Plasma expanders, not diuretics, act in the glomerulus of the nephron in the kidney.

343) Which diuretic would the nurse anticipate administering to a client admitted with acute pulmonary edema? Furosemide Chlorothiazide Spironolactone Acetazolamide

Rationale Furosemide acts on the loop of Henle by increasing the excretion of chloride and sodium; is available for intravenous administration; and is more effective than chlorothiazide, spironolactone, and acetazolamide. Although it is used in the treatment of edema and hypertension, chlorothiazide is not as efficacious as furosemide. Spironolactone is a potassium-sparing diuretic; it is less efficacious than thiazide diuretics. Acetazolamide is used in the treatment of glaucoma to lower intraocular pressure. Test-Taking Tip: Being prepared reduces your stress or tension level and helps you maintain a positive attitude.

1033) Furosemide has been prescribed as part of the medical regimen for a client with hypertension. Which client statement indicates a need for medication education? 'This can decrease my vitamin K level.' 'I will take the medication in the morning.' 'I will contact my health care provider if I notice muscle weakness.' 'I plan to take the medication even when my blood pressure is normal.'

Rationale Furosemide can produce hypokalemia, not vitamin K deficiency. A well-balanced diet should provide all the necessary vitamins and nutrients. Further teaching is necessary. The morning is the desirable time to take furosemide; early administration prevents nocturia. The client's statement to call the health care provider at signs of muscle weakness is appropriate because muscle weakness may indicate hypokalemia. The client's response to take the medicine even when the blood pressure is normal demonstrates an understanding that the medication should be taken as prescribed, independent of how the client feels, because hypertension is often asymptomatic.

523) Hypertension develops in a school-age child with acute glomerulonephritis. Which medication would the nurse anticipate providing teaching for? Digoxin Furosemide Diazepam Phenytoin

Rationale Furosemide inhibits the reabsorption of sodium and chloride from the loop of Henle and distal tubule, increasing urine output and thereby decreasing blood pressure. Digoxin increases the contractility and output of the heart; it is not an antihypertensive. Diazepam is inappropriate; it relaxes skeletal muscle, not the smooth muscle of the arterioles. Phenytoin is an anticonvulsant; it does not reduce blood pressure.

Question 7 The client is discharged from the hospital with a new prescription for furosemide. During a follow-up visit one week later, the nurse notes the following findings. Which finding is most important to report to the health care provider? Question 7 Answer Choices AConstipation Constipation BMuscle cramps Muscle cramps COccasional lightheadedness Occasional lightheadedness DIncreased urine production Increased urine production Question Explanation

Rationale Furosemide is a loop (potassium-wasting) diuretic. It can cause dehydration and hypokalemia, which can result in muscle cramps. This is the most important finding. Dizziness or lightheadedness may occur as the body adjusts to the medication. The nurse should reinforce to the client that they should get up slowly when rising from a sitting or lying position. The client should tell the HCP if these findings persist or become worse. Increased urine production is an expected action of the medication. Some people experience constipation when taking this medication, but it is not as important to report that finding as the possibility of hypokalemia.

690) A client was prescribed furosemide. The nurse would instruct the client to include which food in the diet? Liver Apples Cabbage Bananas

Rationale Furosemide is a loop diuretic that increases potassium excretion by preventing renal absorption. Bananas have a significant amount of potassium. Bananas: 450 mg; cabbage: 243 mg; liver: 73.6 mg; apples: 100 to 120 mg.

615) A school-age child is admitted with hypertensive acute glomerulonephritis. Which medication would the nurse anticipate being prescribed initially in addition to hydralazine? Digoxin Alprazolam Phenytoin Furosemide

Rationale Furosemide is a loop diuretic that is recommended for the treatment of acute glomerulonephritis; it promotes the excretion of fluid and thus limits fluid retention. Digoxin is not used because there is no cardiac involvement. An anxiolytic is unnecessary. Phenytoin may be used only if hypertensive encephalopathy causes seizures. Test-Taking Tip: Being emotionally prepared for an examination is key to your success. Proper use of resources over an extended period of time ensures your understanding and increases your confidence about your nursing knowledge. Your lifelong dream of becoming a nurse is now within your reach! You are excited, yet anxious. This feeling is normal. A little anxiety can be good because it increases awareness of reality; but excessive anxiety has the opposite effect, acting as a barrier and keeping you from reaching your goal. Your attitude about yourself and your goals will help keep you focused, adding to your strength and inner conviction to achieve success.

Question 7 The nurse is providing discharge instructions to an older adult client with heart failure. The client asks, "What is the purpose for taking the furosemide?" How should the nurse respond? Question 7 Answer Choices AIt will help with decreasing fluid buildup in your lungs. It will help with decreasing fluid buildup in your lungs. BIt will help with reducing the risk for an irregular heart rhythm. It will help with reducing the risk for an irregular heart rhythm. CIt will protect your kidneys from chronic damage. It will protect your kidneys from chronic damage. DIt will reverse the damage to your heart muscle. It will reverse the damage to your heart muscle. Question Explanation

Rationale Furosemide is a loop diuretic. Diuretics are the first-line drug of choice in older adults with heart failure (HF) and fluid overload. These drugs enhance the renal excretion of sodium and water by reducing circulating blood volume, decreasing preload, and reducing systemic and pulmonary congestion, i.e., decreased fluid buildup in the lungs. The other actions do not pertain to furosemide.

740) A client is receiving furosemide to relieve edema. The nurse will monitor the client for which adverse effect? Hypernatremia Elevated blood urea nitrogen Hypokalemia Increase in the urine specific gravity

Rationale Furosemide is a potent diuretic used to provide rapid diuresis; it acts in the loop of Henle and causes depletion of electrolytes, such as potassium and sodium. Furosemide inhibits the reabsorption, not retention, of sodium. Furosemide does not affect protein metabolism and will not elevate blood urea nitrogen. Because furosemide increases water excretion relative to solutes, the specific gravity of the fluid more likely will be low.

731) Which client statements indicate that the teaching about furosemide is understood? Select all that apply. One, some, or all responses may be . 'It may take 2 or 3 days for this medication to take effect.' 'I should wear dark glasses when outdoors during the day.' 'I should avoid lying flat in bed.' 'I need to change my position slowly.' 'I should eat more food that is high in potassium.'

Rationale Furosemide may cause hypovolemia, which can result in orthostatic hypotension with sudden changes in position. With loop diuretics, such as furosemide, there is an increase in potassium excretion, so increased potassium intake is needed. The response to furosemide is rapid with diuresis beginning within an hour of administration. Furosemide does not cause photophobia, so dark glasses are not necessary. Lying flat in bed is unnecessary.

573) The nurse is teaching the parents of a child prescribed a high dose of oral prednisone for asthma. Which information is critical for the nurse to include when teaching about this medication? It protects against infection. It should be stopped gradually. An early growth spurt may occur. A moon-shaped face will develop.

Rationale Gradual weaning from prednisone is necessary to prevent adrenal insufficiency or adrenal crisis. Prednisone depresses the immune system, thereby increasing susceptibility to infection. The medication usually suppresses growth. A moon face may occur, but it is not a critical, life-threatening side effect. Test-Taking Tip: Come to your test prep with a positive attitude about yourself, your nursing knowledge, and your test-taking abilities. A positive attitude is achieved through self-confidence gained by effective study. This means (a) answering questions (assessment), (b) organizing study time (planning), (c) reading and further study (implementation), and (d) answering questions (evaluation).

1401) The nurse would instruct a client to stop taking an oral contraceptive and notify the health care provider immediately for the presence of which clinical findings? Select all that apply. One, some, or all responses may be correct. One, some, or all responses may be correct. Nausea Headaches Weight loss Visual disturbances Increased menstrual flow

Rationale Headaches, either sudden or persistent, may indicate hypertension or a cardiovascular event. Visual disorders, such as partial or complete loss of vision or double vision, may indicate neuro-ocular lesions, which are associated with the use of some oral contraceptives. Nausea is a side effect that is not life threatening; the dose may need to be adjusted or the product changed to an alternative. Weight gain, not loss, may occur because of fluid retention. Menorrhagia is less likely to occur.

1205) Oral contraceptives are prescribed for a client who smokes heavily. Which side effect would the nurse warn the client might occur? Blood clots Cervical cancer Ovarian cancer Risk of coronary heart disease later in life

Rationale Heavy smoking is a major risk factor for an increased risk of thrombosis or blood clots. Cervical cancer is associated with human papillomavirus infection, not oral contraceptive use. Oral contraceptives have a protective effect against ovarian cancer. Although there is an increased risk of coronary heart disease while taking an oral contraceptive, this risk abates when it is no longer taken and does not carry over into later life.

675) A client who has atrial fibrillation with rapid ventricular response is started on a continuous heparin infusion. Which clinical finding enables the nurse to conclude that the heparin therapy is effective? Atrial fibrillation converts to a sinus rhythm. The heart rate is stabilized at 70 to 90 beats per minute. The international normalized ratio (INR) is within normal range. An activated partial thromboplastin time (aPTT) is twice the usual value.

Rationale Heparin is an anticoagulant administered to clients with atrial fibrillation to prevent formation of mural thrombi. The desired anticoagulant effect is achieved when the activated partial thromboplastin time is 1.5 to 2 times normal. Medications other than heparin are administered to convert the rhythm and control the rate. The INR is not used to determine heparin effectiveness. Test-Taking Tip: Key words or phrases in the stem of the question such as first, primary, early, or best are important. Similarly, words such as only, always, never, and all in the alternatives are frequently evidence of a wrong response. As in life, no real absolutes exist in nursing; however, every rule has its exceptions, so answer with care.

1221) A client develops a deep vein thrombophlebitis in her leg 3 weeks after giving birth and is admitted for anticoagulant therapy. The nurse would anticipate developing a teaching plan for which anticoagulant? Heparin Warfarin Clopidogrel Enoxaparin

Rationale Heparin is the medication of choice during the acute phase of a deep vein thrombosis; it prevents conversion of fibrinogen to fibrin and of prothrombin to thrombin. Warfarin, a long-acting oral anticoagulant, is started after the acute stage has subsided; it is continued for 2 to 3 months. Clopidogrel is a platelet aggregate inhibitor and is used to reduce the risk of a brain attack. A low-molecular-weight heparin (e.g., enoxaparin) is not administered during the acute stage; it may be administered later to prevent future deep vein thromboses.

Question 10 A client is prescribed heparin therapy for a deep vein thrombosis (DVT). Which laboratory value should the nurse monitor closely? Question 10 Answer Choices AD-dimer D-dimer BPlatelet count Platelet count CActivated partial thromboplastin time Activated partial thromboplastin time DBleeding time Bleeding time Question Explanation

Rationale Heparin is used to prevent further clots from being formed and to prevent the present clot from enlarging. The activated partial thromboplastin time (APTT) test measures the time it takes blood to clot and is used to monitor the effectiveness of heparin therapy. The therapeutic range is about 1 1/2 to 2 or 2 1/2 times the normal values. D-dimer is used to evaluate blood clot formation. Platelet counts are used to evaluate abnormal bleeding times. Bleeding time refers to the time it takes for a pinprick to stop bleeding (normally, about 2 1/2 minutes).

639) A client is diagnosed with acute gouty arthritis. Which medication would the health care provider prescribe to treat the acute attack of gout and prevent future attacks? Ibuprofen Colchicine Probenecid Allopurinol

Rationale High-dose colchicine decreases the inflammatory response to alleviate an acute gout attack; low-dose colchicine is used prophylactically to prevent future attacks. Ibuprofen is a nonsteroidal anti-inflammatory agent that may be used to treat an acute gout attack, but it does not possess activity to prevent future attacks. Probenecid increases uric acid excretion by the kidneys, making it useful for prophylaxis, but it does not possess the anti-inflammatory property needed to treat an acute gout attack. Allopurinol decreases uric acid formation, making it useful for prophylaxis, but it does not possess the anti-inflammatory properties needed to treat an acute gout attack.

95) Which medication would the health care provider prescribe to treat the acute attack of gout and prevent future attacks? Ibuprofen Colchicine Probenecid Allopurinol

Rationale High-dose colchicine decreases the inflammatory response to alleviate an acute gout attack; low-dose colchicine is used prophylactically to prevent future attacks. Ibuprofen is a nonsteroidal anti-inflammatory agent that may be used to treat an acute gout attack, but it does not possess activity to prevent future attacks. Probenecid increases uric acid excretion by the kidneys, making it useful for prophylaxis, but it does not possess the anti-inflammatory property needed to treat an acute gout attack. Allopurinol decreases uric acid formation, making it useful for prophylaxis, but it does not possess the anti-inflammatory properties needed to treat an acute gout attack.

972) Which effect explains the purpose for gradual dosage reduction of glucocorticoids such as dexamethasone? Builds glycogen stores in the muscles Produces antibodies by the immune system Allows the increased intracranial pressure to return to normal Promotes return of cortisone production by the adrenal glands

Rationale Hormone therapy must be withdrawn slowly to allow the adrenal glands to adjust and resume production of their hormone. Building glycogen stores in the muscles, producing antibodies by the immune system, and allowing the increased intracranial pressure to return to normal are not reasons for the gradual withdrawal of dexamethasone.

1102) Which mechanism of action explains how hydrochlorothiazide increases urine output? Increases the excretion of sodium Increases the glomerular filtration rate Decreases the reabsorption of potassium Increases renal perfusion

Rationale Hydrochlorothiazide inhibits sodium reabsorption in the nephrons, causing increased excretion of sodium, which increases urine excretion. The glomerular filtration rate is not affected. The loss of potassium is a side effect, not the mechanism of action. Renal perfusion is not affected.

911) Hydrocortisone is prescribed for a client with Addison's disease. Which response is a therapeutic effect of this medication? Supports a better response to stress Promotes a decrease in blood pressure Decreases episodes of shortness of breath Controls an excessive loss of potassium

Rationale Hydrocortisone is a glucocorticoid that has anti-inflammatory action and aids in metabolism of carbohydrates, fats, and proteins, causing elevation of the blood glucose level. Thus it enables the body to adapt to stress. It may promote fluid retention that results in hypertension and edema. Shortness of breath (dyspnea) is caused by hypovolemia and decreased oxygen supply; neither is affected by hydrocortisone. It may cause potassium depletion. Test-Taking Tip: Have confidence in your initial response to an item because it more than likely is the .

961) The nurse is caring for a client who is scheduled for a bilateral adrenalectomy. Which medication would the nurse expect to be prescribed for this client? Methimazole Regular insulin Pituitary extract Hydrocortisone

Rationale Hydrocortisone is a glucocorticoid. A client undergoing bilateral adrenalectomy must be given adrenocortical hormones so that adjustment to the sudden lack of these hormones that occurs with this surgery can take place Methimazole is used to treat a client with hyperthyroidism, not a client with a bilateral adrenalectomy. Because the surgery involves the adrenal glands, not the pituitary gland, secretion of pituitary hormones will not be affected. Regular insulin is not necessary. Insulin is produced by the pancreas, and its function is not altered by this surgery.

603) The nurse is administering hydroxyzine to a client. The nurse would monitor the client for which side effect of this medication? Ataxia Drowsiness Vertigo Slurred speech

Rationale Hydroxyzine suppresses activity in key regions of the subcortical area of the central nervous system; it also has antihistaminic and anticholinergic effects. Ataxia, vertigo and slurred speech are not associated with hydroxyzine. Test-Taking Tip: Do not worry if you select the same numbered answer repeatedly, because there usually is no pattern to the answers.

978) Valsartan, an angiotensin II receptor antagonist, is prescribed for a client. The nurse will monitor the client for which adverse effect? Constipation Hyperkalemia Hypertension Change in visual acuity

Rationale Hyperkalemia may occur with valsartan. Angiotensin II receptor antagonists, such as valsartan, block vasoconstrictor and aldosterone-producing effects of angiotensin II at receptor sites to decrease blood pressure. Hypotension, not hypertension, may occur. Diarrhea, not constipation, may occur with valsartan. Valsartan does not cause altered visual acuity.

634) Which medications may be used to severe hyperkalemia resulting from intravenous (IV) administration? Select all that apply. One, some, or all responses may be . Calcium chloride Sodium chloride Calcium gluconate Sodium bicarbonate Dextrose solution with insulin

Rationale Hyperkalemia resulting from IV administration might be treated with calcium chloride, calcium gluconate, sodium bicarbonate, and dextrose solution with insulin. These substances lead to the rapid shifting of intracellular potassium ions, thereby reducing potassium concentration. Sodium chloride is primarily used to prevent or treat sodium losses. Test-Taking Tip: Do not read information into questions and avoid speculating. Reading into questions creates errors in judgment.

711) The client with a seizure disorder receives intravenous (IV) phenytoin. The nurse will monitor closely for which condition? Cardiac dysrhythmias Hypoglycemia Polycythemia Paradoxical excitation

Rationale IV phenytoin was once used to treat dysrhythmias until better medications were developed. It depresses both atrial and ventricular conduction, and so it can cause significant dysrhythmias. It can also cause hyperglycemia (not hypoglycemia) and pancytopenia (not polycythemia). Paradoxical excitation is not a known issue; it has a depressant effect resulting in drowsiness.

920) A client with the diagnosis of primary hypertension is started on a regimen of hydrochlorothiazide. Which information will the nurse include when providing instructions regarding this medication? A common side effect is decreased sexual libido. One dose should be omitted if dizziness occurs when standing up. The client should adjust the dosage daily based on the client's blood pressure. An antihypertensive medication will likely be required for the remainder of life.

Rationale If medication is necessary to control primary hypertension, usually it is a lifetime requirement. The client will not adjust the dosage without the health care provider's direction. Impotence may occur with some antihypertensive medications but not with hydrochlorothiazide. The medication will not be stopped; orthostatic hypotension can be controlled by a slow change of body position.

858) A client who has been diagnosed with a myocardial infarction receives digoxin, fluoxetine, morphine, and docusate sodium. Which medication would the nurse identify as a risk factor for straining due to constipation? Digoxin Morphine Docusate Fluoxetine

Rationale Morphine is an opioid. Opioids decrease peristalsis, which may precipitate constipation; straining at stool should be avoided to prevent the Valsalva maneuver, which increases demands on the heart. Digoxin is unrelated to intestinal peristalsis and the potential for constipation. Docusate sodium is a stool softener, which would relieve, not cause, constipation. A side effect of fluoxetine is diarrhea, not constipation. Test-Taking Tip: Calm yourself by closing your eyes, putting down your pencil (or computer mouse), and relaxing. Deep-breathe for a few minutes (or as needed, if you feel especially tense) to relax your body and to relieve tension.

372) Which administration guidelines would the nurse follow when administering midazolam to an older client? The increments should be smaller, and the rate of injection should be slower. The medication should be given as a rapid intravenous push. It is important to monitor for spikes in blood pressure elevation during administration. During the procedure, the medication should be given as needed for pain management.

Rationale In an older client, the peak effect may be delayed; increments should be smaller, and the rate of injection should be slower. When used for sedation/anxiolysis/amnesia for a procedure, the dosage must be individualized and titrated. Midazolam should always be titrated slowly; administer over at least 2 minutes, and allow an additional 2 or more minutes to fully evaluate the sedative effect. Titration to effect with multiple small doses is essential for safe administration. Central nervous system depression is the most serious side effect. A sudden rise in blood pressure shortly after administration has not been evidenced. Midazolam is given to induce sedation/anxiolysis/amnesia for a procedure.

923) When a client's cells are deprived of oxygen during a cardiac arrest, which medication s for deleterious effects of anaerobic energy production? Regular insulin Calcium gluconate Potassium chloride Sodium bicarbonate

Rationale In the absence of oxygen, the body derives its energy anaerobically; this results in a buildup of lactic acid. Sodium bicarbonate, an alkaline medication, will help neutralize the acid, raising the pH. Insulin is used to treat diabetes; it lowers blood sugar by facilitating transport of glucose across cell membranes. Calcium gluconate is used to treat hypocalcemia. Although potassium is essential for cardiac function, it will not acidosis. With acidosis, serum hydrogen ions will exchange with intracellular potassium, leading to a temporary hyperkalemic state; therefore potassium chloride is contraindicated until acidosis is ed.

962) Which information from the client's history would the nurse identify as a risk factor for developing osteoporosis? Takes estrogen therapy Receives long-term steroid therapy Has a history of hypoparathyroidism Engages in strenuous physical activity

Rationale Increased levels of steroids will accelerate bone demineralization. Hyperparathyroidism, not hypoparathyroidism, accelerates bone demineralization. Weight-bearing that occurs with strenuous activity promotes bone integrity by preventing bone demineralization. Estrogen promotes deposition of calcium into bone which may prevent, not cause, osteoporosis.

995) Which statements accurately describe the different medications available for the treatment of osteoporosis? Select all that apply. One, some, or all responses may be correct. Denosumab acts by stimulating bone formation. Individuals with an allergy to salmon can't take calcitonin. Bisphosphonates inhibit osteoclast-mediated bone resorption. Denosumab is a subcutaneous injection given once every 6 months. Teriparatide is used for the prevention of postmenopausal osteoporosis.

Rationale Individuals with an allergy to salmon cannot take calcitonin, which is salmon-based. Bisphosphonates work by inhibiting osteoclasts, which are responsible for breaking down and reabsorbing minerals such as calcium from the bone. Denosumab is a subcutaneous injection that is given once every 6 months; the client takes calcium and vitamin D daily. It is not true that denosumab acts by stimulating bone formation; it is a monoclonal antibody that blocks osteoclast activation and thus decreases the breakdown of bone. Teriparatide is not used for the prevention of postmenopausal osteoporosis; instead, it is generally reserved for the subset of clients at the highest risk—those who have suffered a previous fracture. It stimulates osteoblast function and increases gastrointestinal calcium absorption and renal tubular reabsorption of calcium.

1453) Which birth control agent requires administration once every 3 months? Progestin-only pills Intramuscular progestin Combination biphasic forms Combination monophasic forms

Rationale Intramuscular progestin requires the administration of a single injection every 3 months. Progestin-only oral contraceptive pills should be taken on a daily basis because of a higher incidence of ovulatory cycles. Combination biphasic and monophasic forms are administered as either a 21- or 28-day course.

1481) A pediatric client is prescribed an intravenous infusion of methylprednisolone. Which clinical manifestation requires immediate intervention during administration of the initial dose? Polyuria Tinnitus Drowsiness Hypotension

Rationale Intravenous administration of a steroid can cause a rapid increase in the blood glucose level. One early sign of hyperglycemia is increased urine output. Blood glucose should be checked frequently, and insulin administered as needed. Tinnitus is associated with some antibiotics and with aspirin, not steroids. Drowsiness is associated with sedatives, not steroids. Hypertension, not hypotension, is associated with steroid administration.

125) Which topic will the nurse include in the discharge teaching of a client who has had a total gastrectomy? Daily use of a stool softener Injections of vitamin B 12 for life Monthly injections of iron dextran Replacement of pancreatic enzymes

Rationale Intrinsic factor is lost with removal of the stomach, and vitamin B 12 is needed to maintain the hemoglobin level and prevent pernicious anemia. Adequate diet, fluid intake, and exercise should prevent constipation. Iron-deficiency anemia is not expected. Secretion of pancreatic enzymes should not be affected because this surgery does not alter this function.

953) The health care provider prescribes isosorbide dinitrate 10 mg for a client with chronic angina pectoris. The client asks the nurse why the isosorbide dinitrate is prescribed. How will the nurse respond? 'It prevents excessive blood clotting.' 'It suppresses irritability in the ventricles.' 'It decreases cardiac oxygen demand.' 'The inotropic action increases the force of contraction of the heart.'

Rationale Isosorbide dinitrate dilates peripheral veins and arteries thus decreasing preload and decreasing oxygen demand. Preventing blood from clotting is the action of anticoagulants. Suppressing irritability in the ventricles is the action of antidysrhythmics. Increasing the force of contraction of the heart is the action of cardiac glycosides.

331) Which adverse response would a nurse assesses for when carbidopa-levodopa is prescribed for a client with Parkinson disease? Select all that apply. One, some, or all responses may be . Nausea Lethargy Bradycardia Polycythemia Emotional changes

Rationale Nausea and vomiting may occur; this reflects a central emetic reaction to levodopa. Changes in affect, mood, and behavior are related to the toxic effects of carbidopa-levodopa. Insomnia, tremors, and agitation are side effects that may occur, not lethargy. Tachycardia and palpitations, not bradycardia, occur. Anemia and leukopenia, not polycythemia, are adverse reactions.

884) A client with an intravenous (IV) infusion containing 40 mEq of potassium reports a stinging pain at the IV site. Which actions will the nurse take? Select all that apply. One, some, or all responses may be . Restart the IV in a different vein. Assist the client through guided imagery. Assess the IV site. Ask the health care provider for pain medication. Verify that the potassium is adequately diluted and not infusing too rapidly.

Rationale It is important to first make sure that the IV catheter is patent and that there is no infiltration. The potassium dosage is large and can be very irritating to veins if it isn't sufficient diluted or if it infuses too rapidly. A 40-mEq dose should be diluted in at least 1 L of IV solution. Although imagery may help distract the client from discomfort, this response provides no information as to why the stinging sensation is occurring. Asking the provider for an analgesic doesn't address the underlying problem. STUDY TIP: A word of warning: do not expect to achieve the maximum benefits of this review tool by cramming a few days before the examination. It doesn't work! Instead, organize planned study sessions in an environment that you find relaxing, free of stress, and supportive of the learning process.

Question 20 The home care nurse is reviewing the medical record of a new client with a history of chronic obstructive pulmonary disease, atrial fibrillation and gout. After reviewing the client's medication list, for which medications should the nurse arrange to monitor blood levels? Select all that apply. Beclomethasone Digoxin Theophylline Allopurinol Montelukast Question Explanation

Rationale It is necessary to monitor blood levels for theophylline and digoxin to prevent toxicity. Both of those drugs can accumulate in the blood and reach toxic levels. The other medications are not known to accumulate and cause toxicity if taken as prescribed.

992) Which responses would the nurse offer a client who asks whether she should take soy supplements to decrease menopausal symptoms and improve bone density? Select all that apply. One, some, or all responses may be correct. These supplements can cause high cholesterol. Estrasorb is applied like a lotion but can't transfer to men. Soy supplements might interfere with thyroid hormone absorption. These products have fewer contraindications than other estrogen products. Some women have reported nausea, bloating, diarrhea, and abdominal pain.

Rationale It is true that soy supplements might affect thyroid function and that some women have reported mild stomach and gastrointestinal side effects such as constipation, bloating, nausea, diarrhea, and abdominal pain. It is not accurate to say that soy supplements can raise cholesterol levels; in fact, they are commonly used to reduce blood lipids. Estrasorb is applied like a lotion but can transfer to men and increase their estradiol levels. It is a common misconception that 'natural' products such as soy have fewer contraindications than other products; in fact, they have many of the same contraindications and are not necessarily a safer alternative to traditional medications. Test-Taking Tip: Be alert for details about what you are being asked to do. In this question type, you are asked to select all options that apply to a given situation or client. All options likely relate to the situation, but only some of the options may relate directly to the situation.

362) Which information would the nurse give to a client interested in kava and valerian supplements for the treatment of anxiety and insomnia? Select all that apply. One, some, or all responses may be correct. Valerian can cause restlessness and insomnia. Both kava and valerian are contraindicated with liver disease. Kava is contraindicated with Parkinson disease or depression. Kava can cause temporary yellow discoloration of the skin, hair, and nails. Both kava and valerian can have adverse effects on the central nervous system.

Rationale It is true that valerian can cause restlessness and insomnia. Both kava and valerian can cause liver toxicity and liver injury; both are contraindicated with liver disease. Kava is also contraindicated with Parkinson disease or depression and can worsen symptoms of both. Kava can cause temporary yellow discoloration of the skin, hair, and nails. It is not true that both kava and valerian have adverse effects on the central nervous system; only valerian does, including depression, nausea, and vomiting.

829) The health care provider prescribes lidocaine to treat a ventricular dysrhythmia in a client with cirrhosis of the liver. Which alterations in the usual lidocaine dosage would the nurse anticipate for this client? Higher than usual dosage to compensate for the impaired liver function Lower than usual dosage because the medication is metabolized at a diminished rate Reduced dosage because other organs will compensate for the sluggish liver Equal dosage to that needed for other clients but used over a shorter duration

Rationale Less than the usual adult dose will be prescribed because the liver will not be able to break down lidocaine as effectively as necessary. A dose higher to compensate for the impaired liver function increases the concentration of lidocaine in the blood, leading to toxicity. Lidocaine is metabolized by the liver; other organs cannot assist in the process. This may be life threatening because the client cannot metabolize lidocaine at the required rate, and toxicity may result.

1064) A client with Parkinson disease is admitted to the hospital. Which medication is prescribed to improve the physical manifestations of Parkinson disease? Carbidopa-levodopa Isocarboxazid Dopamine Pyridoxine (vitamin B 6)

Rationale Levodopa crosses the blood-brain barrier and converts to dopamine, a substance depleted in Parkinson disease. Isocarboxazid is a monoamine oxidase inhibitor used for the treatment of psychological symptoms associated with severe depression, not physiological symptoms of Parkinson disease. Dopamine is not prescribed for this purpose because it does not cross the blood-brain barrier. Pyridoxine can reverse the effects of some antiparkinsonian medications and is contraindicated. Test-Taking Tip: Avoid selecting answers that state hospital rules or regulations as a reason or +Rationale for action.

Question 8 An 80-year-old client who is taking digoxin reports nausea, vomiting, abdominal cramps and halo vision. Which laboratory result should the nurse evaluate first? Question 8 Answer Choices APotassium levels Potassium levels BBlood pH Blood pH CMagnesium levels Magnesium levels DBlood urea nitrogen Blood urea nitrogen Question Explanation

Rationale Nausea, vomiting, abdominal cramps and halo vision are classic signs of digitalis toxicity. The most common cause of digitalis toxicity is a low potassium level. Clients are to be taught that it is important to have adequate potassium intake, especially if taking loop or thiazide diuretics that enhance the loss of potassium.

192) Which mechanism of action would the nurse identify for levodopa therapy prescribed to a client diagnosed with Parkinson disease? Blocks the effects of acetylcholine Increases the production of dopamine Restores the dopamine levels in the brain Promotes the production of acetylcholine

Rationale Levodopa is a precursor of dopamine, a catecholamine neurotransmitter; it increases dopamine levels in the brain that are depleted in Parkinson disease. Blocking the effects of acetylcholine is accomplished by anticholinergic medications. Increasing the production of dopamine is ineffective because it is believed that the cells that produce dopamine have degenerated in Parkinson disease. Levodopa does not affect acetylcholine production. Test-Taking Tip: Being emotionally prepared for an examination is key to your success. Proper use of resources over an extended period of time ensures your understanding and increases your confidence about your nursing knowledge. Your lifelong dream of becoming a nurse is now within your reach! You are excited, yet anxious. This feeling is normal. A little anxiety can be good because it increases awareness of reality, but excessive anxiety has the opposite effect, acting as a barrier and keeping you from reaching your goal. Your attitude about yourself and your goals will help keep you focused, adding to your strength and inner conviction to achieve success.

Question 14 A nurse is administering lidocaine to a client with a myocardial infarction. Which assessment finding requires the nurse's immediate action? Question 14 Answer Choices ARespiratory rate of 22 Respiratory rate of 22 BPulse rate of 48 beats per minute Pulse rate of 48 beats per minute CCentral venous pressure reading of 9 mm Hg Central venous pressure reading of 9 mm Hg DBlood pressure of 144/92 Blood pressure of 144/92 Question Explanation

Rationale Lidocaine can cause significant bradycardia and hypotension. A pulse of 48 beats per minute needs immediate attention and is often treated with atropine. At this time the respiratory rate of 22 and blood pressure of 144/92 should be monitored. A normal central venous pressure ranges from 4 to 12 mm Hg. A central venous pressure above 12 my indicate hypervolemia or cardiac failure.

745) Which principle explains how loop diuretics promote diuresis? Osmosis Filtration Diffusion Active transport

Rationale Loop diuretics inhibit the reabsorption of sodium and water in the ascending loop of Henle. The increased sodium load in the distal tubule causes the passive transfer of water from the glomerular filtrate to urine through the process of osmosis. Filtration refers to solutes; solutes are not being passed into the urine. Diffusion is not specific to fluid; osmosis is. Active transport requires energy; water is passively moved from tubule cells to the urine. Test-Taking Tip: Read the question carefully before looking at the answers: (1) Determine what the question is really asking; look for key words; (2) Read each answer thoroughly and see if it completely covers the material asked by the question; (3) Narrow the choices by immediately eliminating answers you know are in.

1482) An adolescent prescribed loratadine 10 mg daily for hay fever is concerned the medication will cause drowsiness during the school day. Which action would the school nurse take? Explain this medication rarely causes drowsiness. Advise to take half a tablet in the morning before school. Suggest skipping the next day's dose if hay fever is better. Recommend contacting the allergist for a prescription containing a stimulant.

Rationale Loratadine causes little or no drowsiness or anticholinergic side effects. Even if the medication did cause drowsiness, the nurse does not have the legal authority to alter the prescribed dose. It is not necessary to call the allergist because loratadine rarely causes drowsiness.

124) The nurse provides care for a client with a long history of alcohol abuse. Which medication would the nurse anticipate will be prescribed for the client to prevent symptoms of withdrawal? Lorazepam Phenobarbital Chlorpromazine Disulfiram

Rationale Lorazepam is most effective in preventing the signs and symptoms associated with withdrawal from alcohol. It depresses the central nervous system by potentiating gamma-aminobutyric acid, an inhibitory neurotransmitter. Phenobarbital is used to prevent withdrawal symptoms associated with barbiturate use. Chlorpromazine, an antipsychotic medication, is not used for alcohol withdrawal. Disulfiram does not prevent symptoms; it is aversion therapy that causes symptoms when alcohol is ingested. Test-Taking Tip: Pace yourself during the testing period and work as accurately as possible. Do not be pressured into finishing early. Do not rush! Students who achieve higher scores on examinations are typically those who use their time judiciously.

798) A client is treated with lorazepam for status epilepticus. Which effect of lorazepam is the reason it is given? Decreases anxiety associated with seizures Promotes rest after the seizure episode Depresses the central nervous system (CNS) Provides amnesia for the convulsive episode

Rationale Lorazepam is used to treat status epilepticus because it depresses the CNS. It also functions as an anxiolytic and sedative and can cause anterograde amnesia; however, these are not the reasons it is prescribed for status epilepticus. STUDY TIP: Begin studying by setting goals. Make sure they are realistic. A goal of scoring 100% on all exams is not realistic, but scoring an 85% may be a better goal.

159) In addition to hydration, parenteral lorazepam is prescribed for a client during alcohol withdrawal delirium. Which primary purpose accurately explains why this medication is given during detoxification? To prevent injury when seizures occur To enable the client to sleep better during periods of agitation To reduce the anxiety tremor state and prevent more serious withdrawal symptoms To calm the client and promote acceptance of the treatment plan

Rationale Lorazepam potentiates the actions of gamma-aminobutyric acid, which reduces anxiety and irritability that are common during withdrawal. This medication helps reduce the risk of seizures but does not prevent physical injury if a seizure occurs. Although the medication may enable the client to sleep better during periods of agitation, this is not its primary objective. The ability of the client to accept treatment depends on readiness to accept the reality of the problem.

527) A 6-year-old child with asthma is prescribed an inhaled corticosteroid. The nurse would conclude the mother understands teaching about the medication side effects when the mother makes which statement? 'I'll watch for frequent urination.' 'I'll check for white patches in the mouth.' 'I'll be alert for short episodes of not breathing.' 'I'll monitor for an increased blood glucose level.'

Rationale Oral candidiasis is a potential side effect of inhaled steroids because of steroids' anti-inflammatory effect; the child should be taught to rinse the mouth after each inhalation. Frequent urination is not a side effect of steroid therapy. Apneic episodes are not a side effect of steroid therapy. Hyperglycemia is not a side effect of inhaled steroid therapy; it may occur when steroids are administered for a systemic effect.

1372) A 20-year-old woman visiting the clinic says that she wishes to begin using depot medroxyprogesterone acetate as a form of birth control. Which important information would the nurse include when teaching the client about this medication? 'Medroxyprogesterone offers protection against the herpes simplex virus.' 'You will need a repeat injection every 6 months.' 'Increase your intake of iron-rich foods to prevent anemia from increased blood loss during menstruation.' 'Increase your calcium intake and exercise because loss of bone mineral density may occur.'

Rationale Loss of bone mineral density is a significant side effect of depot medroxyprogesterone acetate, and increased calcium intake and exercise should be encouraged. Medroxyprogesterone should be administered every 11 to 13 weeks; 6 months is too long before the next dose. Menstrual periods usually lighten or disappear over time. Medroxyprogesterone confers no protection against herpes simplex virus. Test-Taking Tip: You have at least a 25% chance of selecting the correct response in multiple-choice items. If you are uncertain about a question, eliminate the choices that you believe are wrong and then call on your knowledge, skills, and abilities to choose from the remaining responses.

1079) A client is admitted to the hospital with a diagnosis of heart failure and acute pulmonary edema. The health care provider prescribes furosemide 40 mg intravenous (IV) stat to be repeated in 1 hour. Which nursing action will best evaluate the effectiveness of the furosemide in managing the client's condition? Performing daily weights Auscultating breath sounds Monitoring intake and output Assessing for dependent edema

Rationale Maintaining adequate gas exchange and minimizing hypoxia with pulmonary edema are critical; therefore assessing the effectiveness of furosemide therapy as it relates to the respiratory system is most important. Furosemide inhibits the reabsorption of sodium and chloride from the loop of Henle and distal renal tubule, causing diuresis; as diuresis occurs fluid moves out of the vascular compartment, thereby reducing pulmonary edema and the bilateral crackles. Although a liter of fluid weighs approximately 2.2 pounds (1 kg) and weight loss will reflect the amount of fluid lost, it will take time before a change in weight can be measured. Although identifying a greater output versus intake indicates the effectiveness of furosemide, it is the client's pulmonary status that is most important with acute pulmonary edema. Although the lessening of a client's dependent edema reflects effectiveness of furosemide therapy, it is the client's improving pulmonary status that is the best indicator of how furosemide improves the client's condition. STUDY TIP: Study goals should set out exactly what you want to accomplish. Do not simply say, 'I will study for the exam.' Specify how many hours, what day and time, and what material you will cover.

403) Which primary anxiolytic medication would the nurse anticipate developing a teaching plan for when a client with social anxiety disorder has a history of exhibiting an intense, irrational fear of being scrutinized by others? Select all that apply. One, some, or all responses may be . Sertraline Paroxetine Alprazolam Venlafaxine Clonazepam

Rationale Manifestations of social anxiety disorder include stuttering, sweating, palpitations, dry throat, and muscle tension. Clients with this disorder exhibit an intense, irrational fear of being scrutinized by others. Alprazolam and clonazepam are benzodiazepines that are well tolerated in clients, and the benefits are immediate. Sertraline and paroxetine are selective serotonin reuptake inhibitors that are also used in the treatment of social anxiety disorder, but they do not act quickly. Venlafaxine is used to treat posttraumatic stress disorder.

575) The nurse is caring for a 6-year-old child who has undergone craniotomy. The parents ask what effect mannitol has. Which response by the nurse is most appropriate? 'It relieves cerebral pressure.' 'It increases the bladder's filtration rate.' 'It reduces glucose excretion in the urine.' 'It decreases the peripheral retention of fluid.'

Rationale Mannitol is an osmotic diuretic used to relieve cerebral edema. The bladder is a storage basin and is not involved with filtration; mannitol acts in the kidneys. Mannitol is an osmotic diuretic that affects neither the body's excretion of glucose nor peripheral edema. Test-Taking Tip: After you have eliminated one or more choices, you may discover that two of the options are very similar. This can be very helpful because it may mean that one of these look-alike answers is the best choice and the other is a very good distractor. Test both of these options against the stem. Ask yourself which one completes the incomplete statement grammatically and which one answers the question more fully and completely. The option that best completes or answers the stem is the one you should choose. Here, too, pause for a few seconds, give your brain time to reflect, and recall may occur.

363) Which assessment would the nurse perform specific to the safe administration of intravenous mannitol? Body weight daily Urine output hourly Vital signs every 2 hours Level of consciousness every 8 hours

Rationale Mannitol, an osmotic diuretic, increases the intravascular volume that must be excreted by the kidneys. The client's urine output should be monitored hourly to determine the client's response to therapy. Although mannitol results in an increase in urinary excretion that is reflected in a decrease in body weight (1 L of fluid is equal to 2.2 pounds [1 kg]), a daily assessment of the client's weight is too infrequent to assess the client's response to therapy. Urine output can be monitored hourly and is a more frequent, accurate, and efficient assessment than is a daily weight. Vital signs should be monitored every hour considering the severity of the client's injury and the administration of mannitol. Although the level of consciousness should be monitored with a head injury, conducting assessments every 8 hours is too infrequent to monitor the client's response to therapy. Test-Taking Tip: Do not worry if you select the same numbered answer repeatedly; there usually is no pattern to the answers.

813) A health care provider prescribes mannitol for a client with a head injury. Which mechanism of action is responsible for therapeutic effects of this medication? Decreasing the production of cerebrospinal fluid Limiting the metabolic requirements of the brain Drawing fluid from brain cells into the bloodstream Preventing uncontrolled electrical discharges in the brain

Rationale Mannitol, an osmotic diuretic, pulls fluid from the brain to relieve cerebral edema. Mannitol's diuretic action does not decrease the production of cerebrospinal fluid. Mannitol does not affect brain metabolism; rest and lowered body temperature reduce brain metabolism. Preventing uncontrolled electrical discharges in the brain is the action of phenytoin sodium, not mannitol.

216) A client with a diagnosis of dementia of the Alzheimer type has been taking donepezil 10 mg/day for 3 months. The client's partner calls the clinic and reports that the client has increasing restlessness and agitation accompanied by nausea. Which advice would the nurse give the partner? Give the medication with food. Administer the medication at bedtime. Omit 1 dose today and start with a lower dose tomorrow. Bring the partner to the clinic for testing and a physical examination.

Rationale Many people with dementia experience physical problems such as urinary tract infections but cannot adequately verbalize what is happening. They may just become more restless and agitated. Because the client has been taking this dose for 3 months, the problems probably are not being caused by the medication. The client should be brought in for an evaluation. Taking the medication with meals is recommended to decrease gastrointestinal side effects, but this client is experiencing more than this. Donepezil can cause insomnia. The client is already restless and agitated. Taking the medication at bedtime will not help. The nurse would not advise a modification of the dosage without consulting the health care provider.

993) Which statements would the nurse include when teaching a client who is considering medroxyprogesterone injection as a form of hormonal birth control? Select all that apply. One, some, or all responses may be correct. It lasts for 3 months. It can cause nausea and vomiting. It contains both progestin and estrogen. It decreases the risk of age-related loss of bone density. Side effects include drowsiness or insomnia and depression. The most severe undesirable effects are liver dysfunction and thromboembolic disorders.

Rationale Medroxyprogesterone does last for 3 months. It can cause nausea and vomiting, as well as lethargy or insomnia and depression, although the most undesirable severe effects are liver dysfunction and thromboembolic disorders, including thrombophlebitis, cerebrovascular disorder, pulmonary embolism, and retinal thrombosis. Medroxyprogesterone does not contain both progestin and estrogen; it is progestin-only. Medroxyprogesterone does not decrease the risk of age-related loss of bone density; in fact, it has a black-box warning because it can cause bone mineral density loss that puts women at higher risk for osteoporosis and bone fractures later in life.

1215) A client who is a smoker expresses a desire to postpone her first pregnancy for at least 5 years and declines to use a barrier method. Which method would the nurse anticipate providing education for? A birth control patch A vaginal ring Medroxyprogesterone Combined oral contraceptive pills

Rationale Medroxyprogesterone is a long-acting progestin-only contraceptive that is less likely to cause cardiovascular problems in women who smoke than contraceptives containing estrogen might. Vaginal rings, combined oral contraceptive pills, and the birth control patch all contain estrogen and are not recommended for women who smoke. STUDY TIP: Record the information you find to be most difficult to remember on 3' × 5' cards and carry them with you in your pocket or purse. When you are waiting in traffic or for an appointment, just pull out the cards and review again. This 'found' time may add points to your test scores that you have lost in the past.

753) A health care provider prescribes metaproterenol for a client. For which therapeutic effect would the nurse monitor the client? Induced sedation Relaxed bronchial spasm Decreased blood pressure Productive cough

Rationale Metaproterenol stimulates beta receptors of the sympathetic nervous system, causing bronchodilation and an increased rate and strength of cardiac contractions. Barbiturates and hypnotics produce sedation. Antihypertensives and diuretics help decrease blood pressure. Expectorants mobilize respiratory secretions, promoting a productive cough. STUDY TIP: The old standbys of enough sleep and adequate nutritional intake also help keep excessive stress at bay. Although nursing students learn about the body's energy needs in anatomy and physiology classes, somehow they tend to forget that glucose is necessary for brain cells to work. Skipping breakfast or lunch or surviving on junk food puts the brain at a disadvantage.

919) A client with hypertensive heart disease who had an acute episode of heart failure is to be discharged on a regimen of metoprolol and digoxin. Which outcome would the nurse anticipate when metoprolol is administered with digoxin? Headaches Bradycardia Hypertension Junctional tachycardia

Rationale Metoprolol and digoxin both exert a negative chronotropic effect, resulting in a decreased heart rate. Metoprolol reduces, not produces, headaches. These medications may cause hypotension, not hypertension. These medications may depress nodal conduction; therefore junctional tachycardia would be less likely to occur.

716) The client wants to know why midazolam will be administered preoperatively. Which reason would the nurse provide? It reduces pain. It induces sedation. It prevents respiratory depression. It limits oral secretions.

Rationale Midazolam, a benzodiazepine, reduces anxiety and induces sedation. Analgesics are given to reduce pain. It does not prevent respiratory depression and has been known to promote apnea. Atropine, an anticholinergic, is given to decrease oral and respiratory secretions. Test-Taking Tip: Avoid taking a wild guess at an answer. However, should you feel insecure about a question, eliminate the alternatives that you believe are definitely in and reread the information given to make sure you understand the intent of the question. This approach increases your chances of randomly selecting the answer or getting a clearer understanding of what is being asked. Although there is no penalty for guessing, the subsequent question will be based, to an extent, on the response you give to the question at hand; that is, if you answer a question inly, the computer will adapt the next question accordingly based on your knowledge and skill performance on the examination up to that point.

99) Which medication is indicated to treat shift-work sleep disorder (SWSD)? Caffeine Modafinil Atomoxetine Methylphenidate

Rationale Modafinil is a unique nonamphetamine stimulant used to treat SWSD. This medication promotes wakefulness in clients suffering from excessive sleepiness associated with SWSD. Caffeine is a central nervous stimulant used to promote wakefulness, but this medication is not as effective in the treatment of SWSD. Atomoxetine is a nonstimulant used to treat attention-deficit/hyperactivity disorder (ADHD). Methylphenidate is considered a first-choice medication for the treatment of ADHD.

1488) Which side effect would the nurse assess for in a child receiving prednisone? Alopecia Anorexia Weight loss Mood changes

Rationale Mood swings may result from steroid therapy. Alopecia does not result from steroid therapy. An increased appetite, not anorexia, results from steroid therapy. Weight gain, not weight loss, results from steroid therapy.

757) A health care provider prescribes digoxin for a client. The nurse teaches the client to be alert for which common early indication of acute digoxin toxicity? Vomiting Urticaria Photophobia Respiratory distress

Rationale Nausea, vomiting, anorexia, and abdominal pain are early indications of acute toxicity in approximately 50% of clients who take a cardiac glycoside, such as digoxin. Urticaria is a rare, not common, manifestation of digoxin toxicity. Photophobia is a later, not early, manifestation of digoxin toxicity. Respiratory distress is not directly associated with digoxin toxicity. Test-Taking Tip: Being emotionally prepared for an examination is key to your success. Proper use of resources over an extended period of time ensures your understanding and increases your confidence about your nursing knowledge. Your lifelong dream of becoming a nurse is now within your reach! You are excited, yet anxious. This feeling is normal. A little anxiety can be good because it increases awareness of reality, but excessive anxiety has the opposite effect, acting as a barrier and keeping you from reaching your goal. Your attitude about yourself and your goals will help keep you focused, adding to your strength and inner conviction to achieve success.

Question 9 A client who has been diagnosed with Raynaud's disease and hypertension is prescribed nifedipine. For which side effect should the nurse monitor the client? Question 9 Answer Choices ACyanosis of the lips Cyanosis of the lips BDecreased urine output Decreased urine output CIncreased pain in fingers Increased pain in fingers DFacial flushing Facial flushing Question Explanation

Rationale Nifedipine is a calcium channel blocker (CCB) used in the treatment of Raynaud's disease and hypertension by producing vasodilation. As a result of this vasodilating effect, facial flushing can occur. Cyanosis of the lips and decreased urinary output are not expected findings with nifedipine. Raynaud's disease causes vasoconstriction, resulting in pain in the fingers that should decrease when nifedipine is taken.

Question 18 Question 18 The nurse is teaching a client with stable angina about their new prescription for nitroglycerin transdermal patch. Which instructions should the nurse include? Select all that apply. Remove the patch if ankle edema occurs Apply the patch to a hairless area of the body Notify your provider for persistent dizziness or any fainting episode Apply a second patch with chest pain Plan for patch-free time, usually overnight Rotate the application area Question Explanation

Rationale Nitroglycerin (NTG) acts directly on vascular smooth muscle to promote vasodilation. It decreases the pain of exertional angina primarily by decreasing cardiac oxygen demand. NTG comes in a variety of routes of administration. NTG patches contain a reservoir from which the drug is slowly released. Following release, the drug is absorbed through the skin and then into the blood. The rate of release is constant and, depending on the patch used, can range from 0.1 to 0.8 mg/ hr. Effects begin within 30 to 60 minutes and persist as long as the patch remains in place (up to 14 hours). Patches are applied once daily to a hairless area of skin. The site should be rotated to avoid local irritation. Tolerance develops if patches are used continuously (24 hours a day every day). Accordingly, a daily "patch-free" interval of 10 to 12 hours is recommended. This can be accomplished by applying a new patch each morning, leaving it in place for 12 to 14 hours, and then removing it in the evening. NTG can cause orthostatic hypotension and the client should let their provider know if dizziness and lightheadedness persist or the client has a fainting (syncopal) episode as these may indicate that the NTG dose needs to be adjusted/decreased. The other instructions are not appropriate for this medication.

Question 4 The nurse working in an intensive care unit is caring for a client diagnosed with acute angina. The client is receiving an intravenous infusion of nitroglycerin. What is the priority assessment for this client? Question 4 Answer Choices AHeart rate Heart rate BNeurologic status Neurologic status CUrine output Urine output DBlood pressure Blood pressure Question Explanation

Rationale Nitroglycerin (NTG) is a vasodilator used to promote myocardial tissue perfusion and relieve chest pain associated with coronary artery occlusion. The systemic vasodilation that occurs as a result of this medication can cause profound hypotension. The client's blood pressure should be evaluated every 15 minutes until stable, and then every 30 minutes to every hour thereafter. Clients receiving IV nitroglycerin should also be placed on continuous ECG monitoring. NTG is not known to affect neurologic status, urine output or heart rate.

985) Sublingual nitroglycerin has been prescribed for a client with unstable angina. Which client response indicates that nitroglycerin is effective? Pain subsides as a result of arteriole and venous dilation. Pulse rate increases because the cardiac output has been stimulated. Sublingual area tingles because sensory nerves are being triggered. Capacity for activity improves as a response to increased collateral circulation.

Rationale Nitroglycerin causes vasodilation, increasing the flow of blood and oxygen to the myocardium and reducing anginal pain. An increased pulse rate does not indicate effectiveness; it is a side effect of nitroglycerin. The tingling indicates that the medication is fresh; relief of pain is the only indicator of effectiveness. Nitroglycerin does not promote the formation of new blood vessels. Test-Taking Tip: If you are unable to answer a multiple-choice question immediately, eliminate the alternatives that you know are in and proceed from that point. The same goes for a multiple-response question that requires you to choose two or more of the given alternatives. If a fill-in-the-blank question poses a problem, read the situation and essential information carefully and then formulate your response.

Question 11 The nurse on a cardiac unit is caring for a client who is receiving nitroglycerin intravenously for unstable angina. During administration of the medication, which assessment is the priority? Question 11 Answer Choices ARespiratory rate Respiratory rate BCardiac enzymes Cardiac enzymes CCardiac rhythm Cardiac rhythm DBlood pressure Blood pressure Question Explanation

Rationale Nitroglycerin is a drug that is used to provide relief from myocardial chest pain and treat hypertensive emergencies. Nitroglycerin causes vasodilation. Common adverse effects of nitroglycerin include hypotension, headache and dizziness; therefore, monitoring the client's blood pressure is the priority. Nitroglycerin does not affect respirations, cardiac enzyme levels or heart rhythm.

Question 3 The nurse is providing discharge instructions to a client with a prescription for sublingual nitroglycerin. The nurse should inform the client to prepare for this most common side effect? Question 3 Answer Choices AHeadache Headache BDepression Depression CDry mouth Dry mouth DAnorexia Anorexia Question Explanation

Rationale Nitroglycerin is a potent vasodilator and a headache is the most common side effect. The headache comes on suddenly and can be severe, thus the client should be prepared for this effect. The other side effects listed are common side effects of oral medications, but not specifically to nitroglycerin.

843) Which instructions will the nurse give a client for whom nitroglycerin tablets are prescribed? Limit the number of tablets to four per day. Discontinue the medication if a headache develops. Increase the number of tablets if dizziness is experienced. Ensure that the medication is stored in its original dark container.

Rationale Nitroglycerin is sensitive to light and moisture, so it must be stored in a dark, airtight container. Limit the number of tablets to four per day, taken as needed. If more than three tablets are necessary in a 15-minute period, emergency medical attention should be received. A headache may be an expected side effect, and the medication should not be discontinued. Dizziness indicates the dosage may need to be decreased by the health care provider. Test-Taking Tip: After you have eliminated one or more choices, you may discover that two of the options are very similar. This can be very helpful, because it may mean that one of these look-alike answers is the best choice and the other is a very good distractor. Test both of these options against the stem. Ask yourself which one completes the incomplete statement grammatically and which one answers the question more fully and completely. The option that best completes or answers the stem is the one you should choose. Here, too, pause for a few seconds, give your brain time to reflect, and recall may occur.

952) A client who had a myocardial infarction receives a prescription for a nitroglycerin patch. Which statement would the nurse identify as the purpose of the nitroglycerin patch? Decreased heart rate lowers cardiac output. Increased cardiac output increases oxygen demand. Decreased cardiac preload reduces cardiac workload. Peripheral venous and arterial constriction increases peripheral resistance.

Rationale Nitroglycerin reduces cardiac workload by decreasing the preload of the heart by its vasodilating effect. It decreases blood pressure, not heart rate (which may increase to compensate for the decreased blood pressure). It decreases, not increases, oxygen demand. Nitroglycerin dilates, not constricts, peripheral veins and arteries.

852) A client with midsternal pain presents to the emergency department. Vital signs are stable. Which form of nitroglycerin would the nurse anticipate giving initially? Oral capsule Sublingual spray Intravenous solution Transdermal patch

Rationale Nitroglycerin spray provides prompt relief of symptoms. The nurse administers one to two sprays, up to a maximum of three sprays, onto or under the tongue every 5 minutes until pain is relieved. If unrelieved after three sprays, intravenous (IV) nitroglycerin may be considered. Both the transdermal and oral forms of nitroglycerin are used for prophylactic purposes, not management of acute pain.

1145) The nurse is preparing a teaching plan for a client prescribed nitroglycerin sublingual. Which would the nurse include in the teaching? 'Place the tablet under the tongue or between the cheek and gums.' 'It takes 30 to 45 minutes for the nitroglycerin to achieve its effect.' 'If dizziness occurs, take a few deep breaths and lean the head back.' 'To facilitate absorption, drink a large glass of water after taking the medication.'

Rationale Nitroglycerin sublingual tablets should not be chewed, crushed, or swallowed. They work much faster when absorbed through the lining of the mouth. Clients are instructed to place the tablet under the tongue or between the cheek and gums and let it dissolve. The client should not eat, drink, smoke, or use chewing tobacco while a tablet is dissolving; this will decrease the effectiveness of the medication. If taken with water, the tablet is washed away from the site of absorption or may be swallowed. Nitroglycerin sublingual tablets usually give relief in 1 to 5 minutes. If a client experiences dizziness or lightheadedness, the client is instructed to take several deep breaths and bend forward with the head between the knees. This position promotes blood flow to the head.

390) Which information would the nurse include when preparing a teaching plan for a client prescribed sublingual nitroglycerin? "Place the tablet under the tongue or between the cheek and gums." "It takes 30 to 45 minutes for the nitroglycerin to achieve its effect." "If dizziness occurs, take a few deep breaths and lean the head back." "To facilitate absorption, drink a large glass of water after taking the medication."

Rationale Nitroglycerin sublingual tablets should not be chewed, crushed, or swallowed. They work much faster when absorbed through the lining of the mouth. Clients are instructed to place the tablet under the tongue or between the cheek and gums and let it dissolve. The client should not eat, drink, smoke, or use chewing tobacco while a tablet is dissolving; this will decrease the effectiveness of the medication. Nitroglycerin sublingual tablets usually give relief in 1 to 5 minutes. If a client experiences dizziness or lightheadedness, the client is instructed to take several deep breaths and bend forward with the head between the knees. This position promotes blood flow to the head. If taken with water, the tablet is washed away from the site of absorption or may be swallowed.

1091) Which criterion is an indicator that the nitroglycerin sublingual tablets have lost their potency? Sublingual tingling is experienced. The tablets are more than 3 months old. The headache is less severe. Onset of relief is delayed.

Rationale Nitroglycerin tablets are affected by light, heat, and moisture. Loss of potency can occur after 3 months, reducing the medication's effectiveness in relieving pain. A new supply should be obtained routinely. Experiencing sublingual tingling indicates the tablets have retained their potency. Headaches may decrease over time; this is not an indicator of medication potency. A delay in relief reflects the ischemia, not the medication.

Question 16 Propranolol is prescribed for a client with coronary artery disease (CAD). The nurse should consult with the health care provider (HCP) before giving this medication when the client reports a history of which condition? Question 16 Answer Choices AAsthma Asthma BDeep vein thrombosis Deep vein thrombosis CMyocardial infarction Myocardial infarction DPeptic ulcer disease Peptic ulcer disease Question Explanation

Rationale Non-cardioselective beta-blockers such as propranolol block b1- and b2-adrenergic receptors and can cause bronchospasm, especially in clients with a history of asthma. Beta-blockers will have no effect on the client's peptic ulcer disease or risk for DVT. Beta-blocker therapy is recommended after an MI.

1381) A client taking oral contraceptives for 3 months tells the nurse she has breakthrough bleeding between menstrual cycles. For which causative factor would the nurse first assess in the client? Illness Anorexia nervosa Ectopic pregnancy Nonadherence to protocol

Rationale Nonadherence to the instructions for taking the oral contraceptive can alter hormone levels, and breakthrough bleeding may occur as a result. Illness and anorexia nervosa are more likely to cause amenorrhea, not breakthrough bleeding. If nonadherence is determined not to be a concern, then the nurse would assess for far less likely causes such as ectopic pregnancy.

Question 17 The nurse is caring for a client who is being treated for heart failure. After completing the medication reconciliation process, the nurse notes that the prescriber has added lisinopril 5mg orally bid. Which medication from the list below should the nurse question due to possible drug-to-drug interaction with lisinopril? Medication Administration Record Name Dose Route Frequency Metoprolol 50 mg Orally BID Glipizide 5 mg Orally Daily Naproxen 200 mg Orally TID Enoxaparin 40 mg SQ Q12H Question 17 Answer Choices AMetoprolol Metoprolol BGlipizide Glipizide CNaproxen Naproxen DEnoxaparin Enoxaparin Question Explanation

Rationale Nonsteroidal anti-inflammatory (NSAIDs) drugs, such as naproxen, reduce the antihypertensive effects of angiotensin converting enzyme (ACE) inhibitors such as lisinopril. The use of NSAIDs and ACE inhibitors may also predispose patients to develop acute renal failure. Additionally, naproxen increases the risk of heart attack or stroke with heart disease. The nurse should clarify the naproxen with the health care provider. The other medications are not known to interact with lisinopril.

Question 14 The nurse is providing discharge education to a client diagnosed with coronary artery disease. The client is prescribed to use a nitroglycerin transdermal patch at home. Which statement by the client indicates a understanding of safe medication administration? Question 14 Answer Choices A"I will remove the old patch and cleanse the area before applying a new patch." "I will remove the old patch and cleanse the area before applying a new patch." B"This drug can lead to hypertension. So, I will monitor my blood pressure at home." "This drug can lead to hypertension. So, I will monitor my blood pressure at home." C"I will keep a record of chest pain occurrences now that I have this patch." "I will keep a record of chest pain occurrences now that I have this patch." D"I can place this patch on broken skin. It will absorb better." "I can place this patch on broken skin. It will absorb better." Question Explanation

Rationale Numerous administration errors have been reported with nitroglycerin paste and patches. The errors include improper storage and basic administration. The client should be taught to remove the previous patch before applying the new patch and to properly label the tube of nitroglycerin paste and keep it out of the reach of children. When selecting an area to place the patch, the skin should be intact and show no signs of irritation. Nitroglycerin paste has been used erroneously as lotion and caused toxic effects. Nitroglycerin causes vasodilation, which increases the blood supply through the coronary arteries. This may cause hypotension in clients. Some other common side effects include lightheadedness, nausea, dizziness, headache and redness or irritation of the skin covered by the patch.

Question 10 The nurse is caring for an 83-year-old client who is experiencing a sudden onset of confusion. Which medication most likely contributed to this change? Question 10 Answer Choices ACardiac glycoside Cardiac glycoside BAnticoagulant Anticoagulant CLiquid antacid Liquid antacid DAntihistamine Antihistamine Question Explanation

Rationale Older adults are more susceptible to the side effects of anticholinergic medications, such as antihistamines. Antihistamines often cause confusion in the older adult, especially at high doses. Cardiac glycosides, anticoagulants and antacids are not associated with confusion or mental status changes in the older adult.

Question 10 A nurse notes an abrupt onset of confusion in an 85-year-old client. Which newly prescribed medication most likely caused this change in the client's mental status? Question 10 Answer Choices AWarfarin Warfarin BMetoprolol Metoprolol CPantoprazole Pantoprazole DDiphenhydramine Diphenhydramine Question Explanation

Rationale Older adults are susceptible to the side effect of anticholinergic medications, such as antihistamines. Diphenhydramine is a first-generation histamine blocker. Older antihistamines often cause confusion, especially at higher doses. In fact, first-generation antihistamines are included in the Beers Criteria for Potentially Inappropriate Medication Use in Older Adults. Metoprolol (a beta blocker), pantoprazole (a proton pump inhibitor) and warfarin (an anticoagulant) are not known to cause mental status changes.

Question 2 The nurse is talking with a client who was admitted with an acute myocardial infarction due to coronary artery disease. The clients asks what the purpose for the prescribed carvedilol is. How should the nurse respond? Question 2 Answer Choices A"A beta blocker will prevent postural hypotension." "A beta blocker will prevent postural hypotension." B"Most people develop hypertension after a heart attack." "Most people develop hypertension after a heart attack." C"This drug will decrease the workload on your heart." "This drug will decrease the workload on your heart." D"Beta blockers will help to increase your heart rate." "Beta blockers will help to increase your heart rate." Question Explanation

Rationale One action of beta blockers is to decrease systemic vascular resistance by dilation of the arterioles. This is useful for clients with coronary artery disease and will reduce the risk of another MI or a sudden cardiac event. Some of the more commonly prescribed beta blockers include metoprolol and carvedilol (Coreg). The other responses are in.

Question 14 The nurse is preparing to administer an albuterol nebulizer treatment to a patient with asthma. Which assessment finding should be brought to the health care provider's attention prior to administering the medication? Question 14 Answer Choices ATemperature of 101&deg;F (38.3&deg;C) Temperature of 101°F (38.3°C) BHeart rate of 116 bpm Heart rate of 116 bpm CRespiratory rate of 28 Respiratory rate of 28 DLower extremity edema Lower extremity edema Question Explanation

Rationale One of the more common adverse effects of beta-adrenergic medications, such as albuterol, is an increase in heart rate. Normal resting heart rate for children 10-years-old and older is the same as adults: 60 to 100 bpm. The nurse should report the heart rate to the health care provider prior to administering the medication.

1398) The nurse is counseling a 34-year-old client who has requested a prescription for oral contraceptives. Which condition would warrant additional discussion? Anemia Depression Hypertension Dysmenorrhea

Rationale One of the side effects of oral contraceptives is hypertension; therefore they are contraindicated for any woman who already has hypertension, particularly at the client's age or older. Anemia is not a contraindication for women who want to take oral contraceptives because oral contraceptives may help this condition by decreasing bleeding. Depression is not a contraindication for women who want to take oral contraceptives. Oral contraceptives may be prescribed for women with menstrual difficulties such as dysmenorrhea.

1402) A client is concerned about taking hormones for birth control. Which contraceptives, explained to the client by the nurse, have a hormonal component? Select all that apply. One, some, or all responses may be correct. Oral contraceptives Diaphragm Cervical cap Female condom Foam spermicide Transdermal agents

Rationale Oral contraceptives have a hormonal component. Transdermal agents have a hormonal component. The diaphragm, cervical cap, and female condom act as barriers. Foam spermicides kill the sperm; there is no hormonal effect. Test-Taking Tip: Pace yourself during the testing period and work as accurately as possible. Do not be pressured into finishing early. Do not rush! Students who achieve higher scores on examinations are typically those who use their time judiciously.

1375) A 31-year-old client is seeking contraceptive information. While obtaining the client's history, which factor indicates to the nurse that oral contraceptives are contraindicated? Older than 30 years Current hypothyroidism Two multiple pregnancies Blood pressure 162/110

Rationale Oral contraceptives may cause or exacerbate hypertension; even borderline hypertension places the client at risk for a brain attack. Oral contraceptives are not contraindicated for women older than 30 years of age if there are no known risk factors. There is no relationship between oral contraceptives and multiple births. Contraceptives are not contraindicated in clients who have hypothyroidism.

1433) The nurse teaches a client about side effects that necessitate discontinuation of oral contraceptives. Which statement made by the client indicates that the teaching was effective? 'I'll stop taking the pills if I have chest pain.' 'I'll stop taking the pills if I start to retain fluid.' 'I'll stop taking the pills if I have white discharge from the vagina.' 'I'll stop taking the pills if I have pain during the middle of my cycle.'

Rationale Oral contraceptives should be discontinued with any symptom that may be related to a pulmonary embolus. Fluid retention is a common side effect of increased estrogen and progestin; discontinuation of the contraceptive is unnecessary. Leukorrhea may be a sign of infection, not a side effect of oral contraceptives. Abdominal pain in the middle of the menstrual cycle is not expected while an oral contraceptive is being taken. Abdominal pain in the area of an ovary that occurs midway during the menstrual cycle (mittelschmerz) usually indicates ovulation.

859) A client takes furosemide and digoxin for heart failure. Why would the nurse advise the client to drink a glass of orange juice every day? Maintaining potassium levels Preventing increased sodium levels Limiting the medications' synergistic effects ing the associated dehydration

Rationale Orange juice is an excellent source of potassium. Furosemide promotes excretion of potassium, which can result in hypokalemia. Digoxin toxicity can occur in the presence of hypokalemia. Neither medication increases sodium levels. Digoxin does not potentiate the action of furosemide; therefore the client should not experience dehydration. Orange juice will not prevent an interaction between digoxin and furosemide. Test-Taking Tip: After choosing an answer, go back and reread the question stem along with your chosen answer. Does it fit ly? The choice that grammatically fits the stem and contains the information is the best choice.

134) A client is prescribed alprazolam. Which action must the nurse include in the client assessment during the initiation of therapy? Measure the client's urine output. Examine the client's pupils daily. Check the client's blood pressure. Assess the abdomen for distention.

Rationale Orthostatic hypotension is a common side effect of alprazolam that occurs early in therapy, so checking the client's blood pressure is the appropriate action. An alteration in urine output is not a common side effect, but it may occur after prolonged use. Central nervous system depression is not an early side effect, but it may occur after prolonged use; examining the client's pupils daily is not appropriate at this time. Distention is not a common side effect, but distention from constipation may occur after prolonged use. STUDY TIP: In the first pass through the exam, answer what you know and skip what you do not know. Answering the questions you are sure of increases your confidence and saves time. This is buying you time to devote to the questions with which you have more difficulty.

1155) A client is prepared for a supratentorial craniotomy with burr holes, and an intravenous infusion of mannitol is instituted. The nurse concludes that this medication is primarily given for which purpose? Lower blood pressure Prevent hypoglycemia Increase cardiac output Decrease fluid in the brain

Rationale Osmotic diuretics remove excessive cerebrospinal fluid (CSF), reducing intracranial pressure. Osmotic diuretics increase, not decrease, the blood pressure by increasing the fluid in the intravascular compartment. Osmotic diuretics do not directly influence blood glucose levels. Although there is an increase in cardiac output when the vascular bed expands as CSF is removed, it is not the primary purpose of administering the medication.

Question 8 The nurse is administering an osmotic diuretic to a client with a traumatic brain injury. Which finding best indicates that the medication was effective? Question 8 Answer Choices A250 mL clear, yellow urine output over four hours 250 mL clear, yellow urine output over four hours BClear bilateral lung sounds to posterior auscultation Clear bilateral lung sounds to posterior auscultation CIntracranial pressure reading of 14 mmHg Intracranial pressure reading of 14 mmHg DBilateral ovoid pupils that are slow to constrict Bilateral ovoid pupils that are slow to constrict Question Explanation

Rationale Osmotic diuretics, such as mannitol, are used to reduce intracranial or intraocular pressure. Intracranial pressure (ICP) for a client with a head injury should be less than 20 mmHg and the osmotic diuretic may be administered to reduce a high ICP. The osmotic diuretic will reduce the amount of water normally reabsorbed by the renal tubules and loop of Henle, so urinary output is increased, which is an expected occurrence, but does not indicate effectiveness of the medication. Ovoid pupils may indicate the presence of cerebral hypertension. An osmotic diuretic is not intended to reduce pulmonary edema, thus clear lung sounds are not an indicator for effectiveness of the diuretic for this particular client.

715) A client with a seizure disorder will begin taking phenytoin. Which instructions will the nurse give to the client? 'Take the medication on an empty stomach.' 'Provide meticulous oral hygiene.' 'Taper off the medication if seizures are controlled for 3 months.' 'Stop taking the medication if you become pregnant.'

Rationale Phenytoin can cause gingival overgrowth that increases the risk for periodontal disease. The medication should be taken with food or milk to decrease gastrointestinal side effects. The health care provider should oversee any dosage adjustment or discontinuation. Although it can affect the developing fetus, current evidence suggests that the effect of seizure activity on the developing fetus may cause even greater problems.

532) A child is prescribed intravenous mannitol. The nurse understands mannitol belongs to which classification of diuretics? Loop Osmotic Potassium sparing Carbonic anhydrase inhibitor

Rationale Osmotic diuretics, such as mannitol, increase the osmotic pressure of glomerular filtrate and thus decrease absorption of sodium; they are used to treat cerebral edema and increased intraocular pressure. Loop diuretics, such as furosemide, inhibit resorption of sodium and potassium in the loop of Henle; they are used for heart failure and pulmonary edema. Potassium-sparing diuretics, such as spironolactone, interfere with sodium resorption in the distal tubules, thus decreasing potassium excretion; they are used to treat cirrhotic ascites and pulmonary edema. Carbonic anhydrase inhibitors, such as acetazolamide, increase sodium excretion by decreasing sodium-hydrogen ion exchange. They are used to treat seizure disorders and open-angle glaucoma.

466) Which primary reason identifies why oxazepam is given during detoxification? Prevents injury when seizures occur Enables the client to sleep better during periods of agitation Encourages the client to accept treatment for alcoholism Minimizes withdrawal symptoms the client may experience

Rationale Oxazepam potentiates the actions of gamma-aminobutyric acid, especially in the limbic system and reticular formation, and thus it minimizes withdrawal symptoms. This medication helps reduce the risk for seizures but does not prevent injury during a seizure. Enabling the client to sleep better during periods of agitation is not the purpose of the medication. The ability of the client to accept treatment depends on the person's readiness to accept the reality of the problem. Test-Taking Tip: Prepare for exams when and where you are most alert and able to concentrate. If you are most alert at night, study at night. If you are most alert at 2 AM, study in the early morning hours. Study where you can focus your attention and avoid distractions. This may be in the library or in a quiet corner of your home. The key point is to keep on doing what is working for you. If you are distracted or falling asleep, you may want to change when and where you are studying.

797) A client has a tonic-clonic seizure. The nurse anticipates that the health care provider will prescribe the intravenous administration of which medication? Naloxone Diazepam Epinephrine HCl Atropine

Rationale Parenterally administered diazepam is a benzodiazepine that has muscle relaxant and anticonvulsant effects that help limit massive muscular spasms. Naloxone does not limit seizures; it is an opioid antagonist and is used for opioid overdose. Epinephrine HCl does not limit seizures; it increases contractility of the heart. Atropine sulfate is not used for seizures. It is used for bradycardia resulting from vagal overstimulation. STUDY TIP: You have a great resource in your classmates. We all have different learning styles, strengths, and perspectives on the material. Participating in a study group can be a valuable addition to your nursing school experience.

891) A client is experiencing both tingling of the extremities and tetany. The nurse will review the client's laboratory report to check for which electrolyte abnormality? Hypokalemia Hypocalcemia Hyponatremia Hypochloremia

Rationale Paresthesias (tingling of the extremities) and tetany are signs of hypocalcemia. These are not expected findings for hypokalemia, hyponatremia, or hypochloremia.

323) Which effect would the nurse assess for in a client with Alzheimer disease who has been taking galantamine and is newly prescribed paroxetine for depression? Allergic Dystonic Additive Extrapyramidal

Rationale Paroxetine and galantamine potentiate each other's actions. Giving these medications concurrently will not precipitate an allergic reaction. Dystonic effects are associated with antipsychotic medications. Extrapyramidal effects occur with antipsychotic medications. Test-Taking Tip: Avoid taking a wild guess at an answer. However, should you feel insecure about a question, eliminate the alternatives that you believe are definitely in, and reread the information given to make sure you understand the intent of the question. This approach increases your chances of randomly selecting the answer or getting a clearer understanding of what is being asked. Although there is no penalty for guessing, the subsequent question will be based, to an extent, on the response you give to the question at hand; that is, if you answer a question inly, the computer will adapt the next question accordingly based on your knowledge and skill performance on the examination up to that point.

Question 3 A client with a history of asthma is admitted for a minor surgical procedure. Preoperatively, the peak flow is measured at 480 liters/minute. Postoperatively, the client reports chest tightness and the peak flow is now 200 liters/minute. What should the nurse do first? Question 3 Answer Choices ANotify both the surgeon and primary care provider Notify both the surgeon and primary care provider BRepeat the peak flow reading in 30 minutes Repeat the peak flow reading in 30 minutes CAdminister the PRN dose of albuterol Administer the PRN dose of albuterol DApply oxygen at two liters per nasal cannula Apply oxygen at two liters per nasal cannula Question Explanation

Rationale Peak flow monitoring during exacerbations of asthma is recommended for clients with moderate-to-severe persistent asthma to determine the severity of the exacerbation and to guide the treatment. A peak flow reading of less than 50% of the client's baseline reading is a medical alert condition and a short-acting beta agonist must be taken immediately. Notifying the health care provider is important, but that is not what would be done first. First, the client needs assistance. Oxygen administration will not be effective if the airway constriction is not relieved with the albuterol. Leaving the client and returning in 30 minutes will do nothing to help a client in acute distress.

807) Two weeks after starting phenobarbital sodium, the client develops new symptoms. Which symptom is probably caused by the medication? Cognitive impairment Frequent urination Abdominal pain Mania

Rationale Phenobarbital can cause significant lethargy, depression, and cognitive impairment. Frequent urination and abdominal pain are not typically associated with this medication. Depression, not mania, is a common adverse effect for this medication. Test-Taking Tip: Key words or phrases in the stem of the question such as first, primary, early, or best are important. Similarly, words such as only, always, never, and all in the alternatives are frequently evidence of a wrong response. As in life, no real absolutes exist in nursing; however, every rule has its exceptions, so answer with care.

812) A client who is receiving phenytoin asks why folic acid was prescribed. Which explanation would the nurse provide? Phenytoin inhibits absorption of folate from foods. Folic acid potentiates the action of phenytoin. Absorption of iron from foods is improved. Neuropathy caused by phenytoin is prevented.

Rationale Phenytoin inhibits folic acid absorption and potentiates the effects of folic acid antagonists. Folic acid diminishes, not potentiates, the effects of phenytoin. Absorption of iron from foods and prevention of neuropathy caused by phenytoin are not effects of folic acid.

Question 14 The nurse is reinforcing teaching for a client with chronic kidney disease about the prescribed aluminum hydroxide. Which is the best statement by the nurse about this medication? Question 14 Answer Choices A"It decreases phosphate levels." "It decreases phosphate levels." B"It increases urine output." "It increases urine output." C"It reduces potassium levels." "It reduces potassium levels." D"It controls stomach acid secretions." "It controls stomach acid secretions." Question Explanation

Rationale Phosphates tend to accumulate in the client with chronic kidney disease due to decreased filtration capacity of the kidneys. Antacids that contain aluminum such as aluminum hydroxide (Amphojel) are commonly used to lower phosphate levels. Aluminum binds phosphates in the gastrointestinal tract and prevents their absorption. Aluminum hydroxide neutralizes stomach acid already present, but does not control gastric acid production or secretion. It does not affect potassium absorption or levels.

1086) Which medication is unsafe to administer as an intravenous (IV) bolus? Saline flush Potassium chloride Naloxone Adenosine

Rationale Potassium chloride given as an IV bolus can cause cardiac arrest. It must be diluted and infused slowly through an IV infusion pump. Saline flush, naloxone, and adenosine are appropriate to be given as an IV bolus undiluted.

1020) Intravenous (IV) potassium is prescribed for a client with a diagnosis of hypokalemia. Which statement about administration of IV potassium is accurate? Oliguria is an indication for withholding IV potassium. Rapid infusion of potassium prevents burning at the IV site. Clients with severe deficits should be given IV push potassium. Average IV dosage of potassium should not exceed 60 mEq in 1 hour.

Rationale Potassium chloride should not be given unless renal flow is adequate; otherwise, the potassium chloride will accumulate in the body, causing hyperkalemia. Rapid infusion may cause severe pain at the infusion site and precipitate cardiac arrest. Potassium chloride must be well diluted or it will precipitate cardiac arrest. A dose of 60 mEq per hour of potassium chloride is too high.

320) Which food would the nurse encourage a client to eat while receiving treatment to prevent hypokalemia? Broccoli Oatmeal Fried rice Canned carrots

Rationale Potassium is plentiful in green leafy vegetables; broccoli provides 207 mg of potassium per half cup. Oatmeal provides 73 mg of potassium per half cup. Rice provides 29 mg of potassium per half cup. Cooked fresh carrots provide 172 mg of potassium per half cup; canned carrots provide only 93 mg of potassium per half cup. STUDY TIP: Record the information you find to be most difficult to remember on 3" × 5" cards and carry them with you in your pocket or purse. When you are waiting in traffic or for an appointment, just pull out the cards and review again. This "found" time may add points to your test scores that you have lost in the past.

1186) A client with systemic lupus erythematosus is taking prednisone. Which foods would the nurse encourage the client to eat while receiving treatment to prevent hypokalemia? Broccoli Oatmeal Fried rice Cooked carrots

Rationale Potassium is plentiful in green leafy vegetables; broccoli provides 207 mg of potassium per half cup. Oatmeal provides 73 mg of potassium per half cup. Rice provides 29 mg of potassium per half cup. Cooked fresh carrots provide 172 mg of potassium per half cup; canned carrots provide only 93 mg of potassium per half cup. STUDY TIP: Record the information you find to be most difficult to remember on 3' × 5' cards and carry them with you in your pocket or purse. When you are waiting in traffic or for an appointment, just pull out the cards and review again. This 'found' time may add points to your test scores that you have lost in the past.

436) Which teaching would a nurse give to a client with a prescription for potassium supplements? To report any abdominal distress To use salt substitutes to season food To take the medication on an empty stomach To increase the dosage if muscle cramps occur

Rationale Potassium supplements can cause gastrointestinal ulceration and bleeding. Most salt substitutes contain potassium, and their use with potassium supplements can cause hyperkalemia. Because they can be irritating to the stomach, potassium supplements should not be taken on an empty stomach. Although muscle cramps may indicate hypokalemia, clients should not adjust their own dosage.

823) Potassium supplements are prescribed for a client receiving diuretic therapy. Which client statement indicates that the teaching about potassium supplements is understood? 'I will report any abdominal distress.' 'I should use salt substitutes with my food.' 'The medication must be taken on an empty stomach.' 'The dosage is if my urine output increases.'

Rationale Potassium supplements can cause gastrointestinal ulceration and bleeding. Most salt substitutes contain potassium, and their use with potassium supplements can cause hyperkalemia. Because they can be irritating to the stomach, potassium supplements should not be taken on an empty stomach. An increase in urine output is the therapeutic effect of diuretic therapy, not potassium supplements. An adverse effect of potassium supplements is oliguria.

256) Which information would the nurse provide when administering the first dose of prednisone prescribed to a client with an exacerbation of colitis? "Prednisone protects you from getting an infection." "The medication may cause weight loss by decreasing your appetite." "Prednisone is not curative but does cause a suppression of the inflammatory process." "The medication is relatively slow in precipitating a response but is effective in reducing symptoms."

Rationale Prednisone inhibits phagocytosis and suppresses other clinical phenomena of inflammation; this is a symptomatic treatment that is not curative. Prednisone suppresses the immune response, which increases the potential for infection. The appetite is increased with prednisone; weight gain may result from the increased appetite or from fluid retention. Generally, the response to prednisone is rapid. STUDY TIP: Rest is essential to the body and brain for good performance; think of it as recharging the battery. A run-down battery provides only substandard performance. For most students, it is better to spend 7 hours sleeping and 3 hours studying than to cut sleep to 6 hours and study 4 hours. The improvement in the rested mind's efficiency will balance out the difference in the time spent studying. Knowing your natural body rhythms is necessary when it comes to determining the amount of sleep needed for personal learning efficiency.

146) Which explanation would the nurse provide for administering prednisone to a client with an exacerbation of colitis? The client will be protected from getting an infection. Symptoms associated with the colitis will decrease slowly over time. Although the medication causes anorexia, weight loss may not occur. Although the medication decreases intestinal inflammation, it will not cure the colitis.

Rationale Prednisone inhibits phagocytosis and suppresses other clinical phenomena of inflammation; this is a symptomatic treatment that is not curative. The medication suppresses the immune response and increases the potential for infection. The response usually is rapid. Appetite is increased; weight gain may result from this or from fluid retention. Test-Taking Tip: Answer every question. A question without an answer is always a wrong answer, so go ahead and guess.

Question 11 The client who was admitted with exacerbation of ulcerative colitis has developed hyperglycemia. Which medication that the client was prescribed most likely caused this adverse drug effect? Question 11 Answer Choices ADicyclomine Dicyclomine BAcetaminophen Acetaminophen CPrednisone Prednisone DDiphenoxylate/atropine Diphenoxylate/atropine Question Explanation

Rationale Prednisone is a corticosteroid, specifically a glucocorticoid. Corticosteroid therapy may be prescribed during exacerbations of ulcerative colitis to decrease inflammation. Common adverse effects include hyperglycemia, osteoporosis, peptic ulcer disease and an increased risk for infection. The nurse should monitor clients who are receiving prednisone for hyperglycemia. Dicyclomine hydrochloride and diphenoxylate with atropine are cholinergic blocking drugs prescribed for gas (flatus) and diarrhea, commonly seen with ulcerative colitis. Acetaminophen is a non-narcotic analgesic given for mild-to-moderate pain. None of those drugs are known to cause an elevated blood sugar.

537) A child recovering from a severe asthma attack is given oral prednisone 15 mg twice daily. Which intervention would be a priority for the nurse? Having the child rest as much as possible Checking the child's eosinophil count daily Preventing exposure of the child to infection Offering sips of water when administering the medication

Rationale Prednisone reduces the child's resistance to certain infectious processes and, as an anti-inflammatory medication, masks infection. The child will self-limit activity depending on respiratory status. The eosinophil count is often consistently increased in children with asthma. The child will need adequate hydration to help loosen and expel mucus.

1507) Which condition contraindicates the use of ginseng herbal therapy? Pregnancy Schizophrenia Bipolar depression Alzheimer disease

Rationale Pregnancy is contraindicated for ginseng herbal therapy. Schizophrenia, bipolar depression, and Alzheimer disease are contraindicated for St. John's Wort herbal therapy.

23) Which condition contraindicates the use of ginseng herbal therapy? Pregnancy Schizophrenia Bipolar depression Alzheimer disease

Rationale Pregnancy is contraindicated for ginseng herbal therapy. Schizophrenia, bipolar depression, and Alzheimer disease are contraindicated for St. John's Wort herbal therapy.

Question 11 The nurse is caring for a client who is receiving procainamide intravenously. It is most important that the nurse monitors which parameter? Question 11 Answer Choices ASerum potassium levels Serum potassium levels BHourly urinary output Hourly urinary output CContinuous ECG readings Continuous ECG readings DNeurological signs Neurological signs Question Explanation

Rationale Procainamide is used to suppress cardiac arrhythmias. When administered intravenously, it must be accompanied by continuous cardiac monitoring.

990) A client with rheumatoid arthritis has been taking a corticosteroid medication for the past year. Prolonged use of corticosteroids puts this client at increased risk for which complication? Decreased white blood cells Increased C-reactive protein Increased sedimentation rate Decreased serum glucose levels

Rationale Prolonged use of steroids may cause leukopenia as a result of bone marrow depression. C-reactive protein and sedimentation rate are elevated in acute inflammatory diseases; steroids help decrease them. Serum glucose levels increase with steroid use.

Question 3 A 42-year-old male client diagnosed with hypertension tells the nurse he no longer wants to take the prescribed propranolol. Which client statement best explains the reason why he does not want to take this medication? Question 3 Answer Choices A"I have difficulty falling asleep." "I have difficulty falling asleep." B"I'm having problems with my stomach." "I'm having problems with my stomach." C"I'm experiencing decreased sex drive." "I'm experiencing decreased sex drive." D"I feel so tired all the time." "I feel so tired all the time." Question Explanation

Rationale Propranolol is a beta-blocker used to treat many conditions, such as essential tremors, angina, hypertension and heart rhythm disorders. Common side effects of this drug include nausea, diarrhea, constipation, stomach cramps, rash, tiredness, dizziness, sleep problems and vision changes. Additionally, propranolol may cause decreased sex drive, impotence or difficulty having an orgasm in men. The clients can be switched to an alternative antihypertensive, such as an angiotensin-converting enzyme (ACE) inhibitor or a calcium channel blocker.

Question 4 The nurse is providing information to a client about propranolol. Which statement by the client indicates the teaching has been effective? Question 4 Answer Choices A"I should expect to feel nervousness during the first few weeks." "I should expect to feel nervousness during the first few weeks." B"I can have a heart attack if I stop this medication suddenly." "I can have a heart attack if I stop this medication suddenly." C"I could have an increase in my heart rate for a few weeks." "I could have an increase in my heart rate for a few weeks." D"I may experience seizures if I stop the medication abruptly." "I may experience seizures if I stop the medication abruptly." Question Explanation

Rationale Propranolol is commonly used to treat hypertension, abnormal heart rhythms, heart disease and certain types of tremors. It is in a class of medications called beta blockers. Suddenly discontinuing a beta blocker can cause angina, hypertension, dysrhythmias, or even a myocardial infarction (i.e., heart attack).

591) Which effect would the nurse assess a teenager for if more than the recommended dose of oxymetazoline nasal spray is taken? Nasal polyps Ringing in the ears Bleeding tendencies Increased nasal congestion

Rationale With frequent and continued use, oxymetazoline can cause rebound congestion of mucous membranes. Nasal polyps may be associated with allergies but are unrelated to nasal spray use. Ringing in the ears (tinnitus) is not associated with oxymetazoline, although this medication may cause hypotension, tachycardia, and dizziness. Bleeding tendencies are related to inadequate clotting mechanisms, which are not associated with the use of this nasal spray.

Question 17 Question 17 The nurse is caring for a client who is experiencing excessive bleeding after receiving unfractionated heparin sodium. Which orders should the nurse anticipate from the health care provider? Select all that apply. Administer vitamin K. Obtain prothrombin time (PT)/international normalized ratio (INR). Administer protamine sulfate. Obtain activated partial thromboplastin time (aPTT). Change prescription to enoxaparin. Question Explanation

Rationale Protamine sulfate is the antidote used to reverse the anticoagulant effects of heparin. A serum aPTT or PTT lab test is used to evaluate the anticoagulation effect of heparin. Vitamin K is the antidote for warfarin. A serum PT/INR lab test is used to monitor the therapeutic effectiveness of warfarin. Enoxaparin is another type of heparin and would be contraindicated for this client.

1122) Which vitamin is essential for the synthesis of prothrombin by the liver? B 12 C D K

Rationale Prothrombin is synthesized in the liver in the presence of vitamin K; vitamin K initiates the vital process of coagulation. Vitamin B 12 is needed for hemoglobin synthesis. Vitamin C plays a role in collagen formation. Vitamin D is involved in calcium absorption and metabolism. Test-Taking Tip: Prepare for exams when and where you are most alert and able to concentrate. If you are most alert at night, study at night. If you are most alert at 2 AM, study in the early morning hours. Study where you can focus your attention and avoid distractions. This may be in the library or in a quiet corner of your home. The key point is to keep on doing what is working for you. If you are distracted or falling asleep, you may want to change when and where you are studying.

Question 18 The nurse is preparing a client with rheumatoid arthritis (RA) for discharge to an assisted living facility. Which statement about the prescribed oral glucocorticoid is correct? Question 18 Answer Choices A"The medication will reverse the joint deterioration of RA." "The medication will reverse the joint deterioration of RA." B"You will be taking the medication for several years." "You will be taking the medication for several years." C"It is normal to experience some memory loss or hallucinations." "It is normal to experience some memory loss or hallucinations." D"The medication will be gradually tapered off over 5 to 7 days." "The medication will be gradually tapered off over 5 to 7 days." Question Explanation

Rationale RA is an autoimmune, inflammatory disease that affects the joints. It is a progressive disease that causes joint deterioration and destruction, joint deformities and functional limitations for affected clients. The main goal of pharmacotherapy for RA is symptom relief. Glucocorticoids are anti-inflammatory drugs, which can relieve symptoms of RA and may also delay disease progression. For generalized symptoms related to RA, oral glucocorticoids are indicated. The most commonly employed oral glucocorticoids are prednisone and prednisolone. Glucocorticoids can slow disease progression, but will not reverse it. Treatment with glucocorticoids for RA is usually limited to short courses. Adverse psychological reactions such as hallucinations, memory loss or other psychoses must be reported to the provider and may require discontinuation of the glucocorticoid. To minimize adrenal insufficiency when glucocorticoids are discontinued, doses should be tapered very gradually.

701) The nurse is administering 40 mg of furosemide intravenously. Which sensation reported by the client would the nurse consider when determining that it is being administered too quickly? Full bladder Buzzing ears Fast heartbeat Numb arms and legs

Rationale Rapid administration of furosemide can cause tinnitus (a perceived ringing or buzzing in the ears), loss of hearing, and ear pain. Furosemide has a diuretic effect, so a full bladder with a need to urinate is an anticipated response unrelated to speed of administration. Furosemide does not affect the heart rate. Furosemide does not cause peripheral neuropathy.

1220) The nurse is caring for a client who has just received epidural anesthesia. The nurse would monitor for which adverse effects? Uterine atony Hypotension Decreased urine production Precipitous second stage of labor

Rationale Regional anesthesia lowers the blood pressure, which puts both mother and fetus in jeopardy. The client may not have the sensation to void, but the amount of urine manufactured does not decrease because a regional block does not affect the kidneys. Epidural anesthesia does not shorten the second stage of labor and does not cause uterine atony.

945) Which pain characteristic would the nurse expect to observe when a client is experiencing anginal pain? Unchanged by rest Precipitated by light activity Described as a knifelike sharpness Relieved by sublingual nitroglycerin

Rationale Relief by sublingual nitroglycerin is a classic reaction because it causes vasodilation of peripheral veins and arteries, thereby decreasing oxygen demand by decreasing preload. To a lesser extent, sublingual nitroglycerin dilates coronary arteries, which increases oxygen to the myocardium, thereby decreasing pain. Immediate rest frequently relieves anginal pain. Angina usually is precipitated by exertion, emotion, or a heavy meal. Angina usually is described as tightness, indigestion, or heaviness.

1105) Which instruction would the nurse include in a teaching plan for nitroglycerin patches? 'Apply the patch on a distal extremity.' 'Remove a previous patch before applying the next one.' 'Massage the area gently after applying the patch to the skin.' 'Apply a warm compress to the site before attaching the patch.'

Rationale Removing the previous patch before applying the next patch ensures that the client receives just the prescribed dose. Ideally, a patch should be removed after 12 to 14 hours to avoid the development of tolerance. The patch should be rotated among hair-free and scar-free sites; acceptable sites include the chest, upper abdomen, proximal anterior thigh, or upper arm. The patch should be gently pressed against the skin to ensure adherence; it should not be massaged. Applying a warm compress to the site before attaching the patch is unnecessary and can result in excessive absorption of the medication. STUDY TIP: Rest is essential to the body and brain for good performance; think of it as recharging the battery. A run-down battery provides only substandard performance. For most students, it is better to spend 7 hours sleeping and 3 hours studying than to cut sleep to 6 hours and study 4 hours. The improvement in the rested mind's efficiency will balance out the difference in the time spent studying. Knowing your natural body rhythms is necessary when it comes to determining the amount of sleep needed for personal learning efficiency.

309) Which medication would the nurse instruct a client to avoid while taking alprazolam? Select all that apply. One, some, or all responses may be . Opioids Alcohol Barbiturates Antidepressants First-generation antipsychotics

Rationale Respiratory depression can occur if a client combines benzodiazepines with opioids, alcohol, or barbiturates. Antidepressants and first-generation antipsychotics are safe to take with benzodiazepines.

894) A client who takes daily megadoses of vitamins is hospitalized with joint pain, loss of hair, yellow pigmentation of the skin, and an enlarged liver due to vitamin toxicity. Which type of toxicity would the nurse suspect? Retinol (vitamin A) Thiamine (vitamin B 1) Pyridoxine (vitamin B 6) Ascorbic acid (vitamin C)

Rationale Retinol is lipid soluble and eliminated by the liver. Joint pain, hair loss, jaundice, anemia, irritability, pruritus, and enlarged liver and spleen are signs of vitamin A toxicity. Thiamine, pyridoxine, and ascorbic acid are water soluble, so they are typically excreted in the urine before toxic blood levels can be achieved. However, excess thiamine may elicit an allergic reaction in some individuals, excess vitamin C (ascorbic acid) may cause diarrhea or renal calculi, and ultrahigh doses (about 800 times the normal dose) of pyridoxine (vitamin B 6) can promote neuropathy. Remember that lipid-soluble vitamins normally take longer to eliminate and accumulate faster than water-soluble vitamins. STUDY TIP: Begin studying by setting goals. Make sure they are realistic. A goal of scoring 100% on all exams is not realistic but scoring an 85% may be a better goal.

1077) The health care provider prescribes finasteride for a client with benign prostatic hyperplasia (BPH). The client would like to take saw palmetto instead of the finasteride. Which information would the nurse provide to the client about this herbal supplement? 'Research has shown that saw palmetto is no better than a placebo.' 'You can take both; saw palmetto doesn't require a prescription.' 'The herbal supplement will relieve symptoms by altering the size of the prostate.' 'Substituting saw palmetto is a good option to avoid all the bad side effects of finasteride.'

Rationale Rigorous research has demonstrated no significant difference between saw palmetto and a placebo. The health care provider must be consulted regarding the client's desire to change the prescribed therapy. Saw palmetto should be taken with food to limit gastrointestinal side effects. Saw palmetto does not alter the size of the prostate gland. Substituting something that is ineffective is not a good solution regardless of issues surrounding side effects. STUDY TIP: Develop a realistic plan of study. Do not set rigid, unrealistic goals.

Question 3 The nurse is discharging a client on oral potassium replacement. Which of the following statements requires further teaching by the nurse? Question 3 Answer Choices A"I can still take my nonsteroidal anti-inflammatory medications occasionally for my arthritis pain." "I can still take my nonsteroidal anti-inflammatory medications occasionally for my arthritis pain." B"I will continue to use salt substitutes to flavor my food." "I will continue to use salt substitutes to flavor my food." C"I will take my furosemide first thing in the morning." "I will take my furosemide first thing in the morning." D"I will read the food labels for added potassium." "I will read the food labels for added potassium." Question Explanation

Rationale Salt substitutes are made using potassium. As the client is taking potassium supplements, they should avoid salt substitutes to prevent hyperkalemia from occurring. NSAIDS can be used occasionally. The furosemide should be taken in the morning. Some low-sodium prepared foods may contain potassium, so reading the labels is important.

658) A client with chronic obstructive pulmonary disease prepares to take a medication that is delivered via a nebulizer. Which instruction would the nurse provide when teaching about use of the nebulizer? 'Hold your breath, spray the medication into your mouth, then inhale deeply.' 'Depress the canister as you inhale deeply, then hold your breath for at least 10 seconds.' 'Seal your lips around the mouthpiece and breathe in and out, taking slow, deep breaths.' 'Inhale the medication from the nebulizer, remove the mouthpiece to exhale and then repeat.'

Rationale Sealing the lips around the mouthpiece ensures that medication is delivered on inspiration; slow, deep breaths promote better deposition and efficacy of medication deep into the lungs. The breath should not be held during administration. A nebulizer treatment delivers medication by inhaling it into the mouth through a mouthpiece, not a canister. Removing the mouthpiece from the mouth to exhale allows valuable aerosolized medication to be deposited into the air; therefore the client will not receive the full dose of aerosolized medication.

219) Which instruction about phenytoin will the nurse provide during discharge teaching to a client with epilepsy who is prescribed phenytoin for seizure control? "Antiseizure medications will probably be continued for life." "Phenytoin prevents any further occurrence of seizures." "This medication needs to be taken during periods of emotional stress." "Your antiseizure medication usually can be stopped after a year's absence of seizures."

Rationale Seizure disorders usually are associated with marked changes in the electrical activity of the cerebral cortex, requiring prolonged or lifelong therapy. Seizures may occur despite medication therapy; the dosage may need to be adjusted. A therapeutic blood level must be maintained through consistent administration of the medication irrespective of emotional stress. Absence of seizures will probably result from medication effectiveness rather than from ion of the pathophysiological condition.

147) A client abruptly stops taking a barbiturate. Which withdrawal complication would the nurse anticipate that the client may experience? Ataxia Seizures Diarrhea Urticaria

Rationale Seizures are a serious side effect that may occur with abrupt withdrawal from barbiturates. Ataxia, diarrhea, and urticaria are not associated with barbiturate withdrawal.

633) Which B vitamin deficiency will result in Wernicke encephalopathy? B 3 (niacin) B 1 (thiamine) B 2 (riboflavin) B 6 (pyridoxine)

Rationale Severe deficiency of thiamine will result in Wernicke encephalopathy. Niacin deficiency causes pellagra. Riboflavin deficiency can result in cutaneous, oral, and corneal changes. Pyridoxine deficiency can progress to sideroblastic anemia, neurological disturbances, and xanthurenic aciduria, among other problems.

981) Hydrochlorothiazide (HCTZ) has been prescribed for a client with hypertension. The client reports hearing that furosemide is more effective and requests a prescription change. How will the nurse respond? 'HCTZ has fewer side effects.' 'HCTZ does not cause dizziness.' 'HCTZ is only taken when needed.' 'HCTZ does not cause dehydration.'

Rationale Side effects from thiazides generally are minor and rarely result in discontinuation of therapy. Dizziness is a side effect of all diuretics. There is a potential for dehydration with all diuretics. All diuretic medications are taken regularly as directed.

1104) Digoxin is prescribed for a client with heart failure. The nurse will assess for which signs and symptoms that indicate digoxin toxicity? Select all that apply. One, some, or all responses may be . Nausea Yellow vision Irregular pulse Increased urine output Heart rate of 64 beats/minute

Rationale Signs and symptoms of digoxin toxicity include bradycardia, headache, dizziness, confusion, nausea, and visual disturbances (blurred vision or yellow vision). In addition, electrocardiogram (ECG) findings may include heart block, atrial tachycardia with block, or ventricular dysrhythmias, all causing an irregular pulse. Increased urine output is an expected effect of improved cardiac output; a pulse rate of 64 beats/minute is an acceptable rate when a client is receiving digoxin. Test-Taking Tip: Avoid spending excessive time on any one question. Most questions can be answered in 1 to 2 minutes.

696) A client who takes multiple medications complains of severe nausea, and the client's heartbeat is irregular and slow. The nurse determines that these signs and symptoms are toxic effects of which medication? Digoxin Captopril Furosemide Morphine sulfate

Rationale Signs of digoxin toxicity include cardiac dysrhythmias, anorexia, nausea, vomiting, and visual disturbances. Although nausea and heart block may occur with captopril, these symptoms rarely are seen; drowsiness and central nervous system disturbances are more common. Toxic effects of morphine are slow, deep respirations, stupor, and constricted pupils; nausea is a side effect, not a toxic effect. Toxic effects of furosemide are renal failure, blood dyscrasias, and loss of hearing.

987) Which instructions will the nurse include in the teaching plan for a client who will be taking simvastatin? Select all that apply. One, some, or all responses may be . Increase dietary intake of potassium. Avoid prolonged exposure to the sun. Schedule regular ophthalmic examinations. Take the medication at least half an hour before meals. Contact your health care provider if skin becomes gray-bronze.

Rationale Simvastatin increases photosensitivity; the client should avoid sun exposure and use sunblock. The client should be monitored for the adverse effects of glaucoma and cataracts. Gray-bronze skin and unexplained muscle pain are signs of rhabdomyolysis. Rhabdomyolysis, a life-threatening response, is the disintegration of muscle associated with myoglobin in the urine. Simvastatin does not affect levels of potassium. The medication is most effective when taken at bedtime because cholesterol synthesis is highest at night.

1067) A health care provider prescribes simvastatin 20 mg daily for elevated cholesterol and triglyceride levels for a female client. Which advice is important for the nurse to teach when the client initially takes the medication? Take the medication with breakfast. Have liver function tests every 6 months. Wear sunscreen to prevent photosensitivity reactions. Inform the health care provider if you wish to become pregnant.

Rationale Simvastatin is a teratogen that is contraindicated in pregnancy because it is capable of causing fetal damage. Simvastatin should be taken in the evening because most cholesterol is synthesized between midnight and 3:00 AM. Liver function tests should be done at 6 to 12 weeks initially and only then every 6 months. Although wearing sunscreen should be taught, sensitivity reactions are a rare occurrence, and this is not as important.

Question 15 The inpatient hospital nurse is caring for a client with hypokalemia. The health care provider prescribed a potassium intravenous (IV) infusion of 40 mEq potassium chloride in 250 mL normal saline to be infused over 4 hours. The nurse receives the infusion from the pharmacy. Which action should the nurse take next? Question 15 Answer Choices AConfirm patency of the peripheral venous access device and start the infusion Confirm patency of the peripheral venous access device and start the infusion BNotify the health care provider of the inappropriate dose of the prescribed IV potassium Notify the health care provider of the inappropriate dose of the prescribed IV potassium CAsk another nurse to verify the prescription, IV solution and serum potassium level Ask another nurse to verify the prescription, IV solution and serum potassium level DAsk another nurse to witness the addition of the prescribed potassium to the IV solution Ask another nurse to witness the addition of the prescribed potassium to the IV solution Question Explanation

Rationale Since potassium chloride is considered a high alert medication, especially when given IV, having two nurses verify the order and IV bag is recommended. The nurses should compare the supplied IV bag to the prescriber's order. If potassium IV is infused too rapidly or in too high a dose, it can cause dysrhythmias and cardiac arrest. In addition, the second nurse should also verify the client's most recent serum potassium level to ensure that the prescription is appropriate. The prescribed dose and amount of IV solution is within normal range for IV potassium replacement therapy. Potassium should never be added by a nurse to an IV bag.

587) Status epilepticus develops in an adolescent with a seizure disorder who is taking antiseizure medication. Which reason would the nurse identify as the most common reason for the development of status epilepticus? The provider failed to account for a growth spurt. The amount prescribed is insufficient to cover activities. The prescribed antiseizure medication probably is not taken consistently. The client is prescribed a medication that is ineffective in preventing seizures.

Rationale Skipping doses of the medication is a form of denial that an adolescent client may engage in once the seizures are controlled; also, adolescents tend to feel invincible. The dosage is based not on activity but on the type of seizure. Medications are prescribed according to the type of seizure and are effective if taken consistently. The dosage of antiepileptic medications is based on many factors, including age, type of medication, and presence of infection, as well as on changes in height and weight.

1256) A client who has a habit of smoking is on estrogen therapy. Which condition is the client at most risk of developing? Thrombosis Gastrointestinal upset Risk of developing cancer Decreased effectiveness of estrogen

Rationale Smoking along with estrogen therapy may increase the risk of thrombosis. Estrogen taken on an empty stomach may cause gastrointestinal upset. Estrogen is not prescribed to clients with endometrial or breast cancer. The effectiveness of estrogen decreases with the use of anticoagulants, rifampicin, and St. John wort.

1115) A female client receiving cortisone therapy for adrenal insufficiency expresses concern that she is developing facial hair. How would the nurse respond? 'It is just another sign of adrenal insufficiency.' 'This side effect will disappear after therapy.' 'This is not important as long as you are feeling better.' 'The medication contains a hormone that causes male characteristics.'

Rationale Some cortisol derivatives possess 17-keto-steroid (androgenic) properties, which result in hirsutism. Facial hair is not a sign of the illness; it results from androgens that are present in cortisol. Hirsutism will be a long-term problem because therapy is provided on a long-term, usually lifelong, basis. The response 'This is not important as long as you are feeling better' doesn't address the client's concerns. Test-Taking Tip: Be aware that information from previously asked questions may help you respond to other examination questions.

1076) A client with cirrhosis of the liver has been taking chlorothiazide. The provider adds spironolactone to the client's medication regimen to prevent which condition? Hyponatremia Hypokalemia Ascites Peripheral neuropathy

Rationale Spironolactone is a potassium-sparing diuretic often used in conjunction with thiazide diuretics. The provider was prompted to add spironolactone to the chlorothiazide to prevent potassium loss. It stimulates sodium excretion so will not prevent hyponatremia. Spironolactone is a relatively weak diuretic that will not have a significant effect on ascites. Peripheral neuropathy is not a concern in this scenario and spironolactone would not have an effect on it if it was a concern. Test-Taking Tip: Notice how the subjects of the questions are related and, through that relationship, the answers to some of the questions may be provided within other questions of the test.

438) Which adverse medication effect will a nurse monitor for in a client with cirrhosis of the liver who develops ascites and is prescribed spironolactone? Bruising Tachycardia Hyperkalemia Hypoglycemia

Rationale Spironolactone is a potassium-sparing diuretic that is used to treat clients with ascites; therefore the nurse would monitor the client for signs and symptoms of hyperkalemia. Bruising and purpura are associated with cirrhosis, not with the administration of spironolactone. Spironolactone does not cause tachycardia. Spironolactone does not cause hypoglycemia. STUDY TIP: Determine whether you are a "lark" or an "owl." Larks, day people, do best getting up early and studying during daylight hours. Owls, night people, are more alert after dark and can remain up late at night studying, catching up on needed sleep during daylight hours. It is better to work with natural biorhythms than to try to conform to an arbitrary schedule. You will absorb material more quickly and retain it better if you use your most alert periods of each day for study. Of course, it is necessary to work around class and clinical schedules. Owls should attempt to register in afternoon or evening lectures and clinical sections; larks do better with morning lectures and day clinical sections.

1130) A client with cirrhosis of the liver develops ascites, and the health care provider prescribes spironolactone. The nurse will monitor the client for which adverse medication effect? Bruising Tachycardia Hyperkalemia Hypoglycemia

Rationale Spironolactone is a potassium-sparing diuretic that is used to treat clients with ascites; therefore the nurse would monitor the client for signs and symptoms of hyperkalemia. Bruising and purpura are associated with cirrhosis, not with the administration of spironolactone. Spironolactone does not cause tachycardia. Spironolactone does not cause hypoglycemia. STUDY TIP: Determine whether you are a 'lark' or an 'owl.' Larks, day people, do best getting up early and studying during daylight hours. Owls, night people, are more alert after dark and can remain up late at night studying, catching up on needed sleep during daylight hours. It is better to work with natural biorhythms than to try to conform to an arbitrary schedule. You will absorb material more quickly and retain it better if you use your most alert periods of each day for study. Of course, it is necessary to work around class and clinical schedules. Owls should attempt to register in afternoon or evening lectures and clinical sections; larks do better with morning lectures and day clinical sections.

Question 9 The nurse is administering spironolactone for a client diagnosed with cirrhosis of the liver and ascites. Which electrolyte should the nurse anticipate to be spared when giving this medication? Question 9 Answer Choices ASodium Sodium BPhosphate Phosphate CPotassium Potassium DAlbumin Albumin Question Explanation

Rationale Spironolactone is a potassium-sparing diuretic. Indications for this medication include edema associated with heart failure, cirrhosis, and nephrotic syndrome. The nurse should anticipate that potassium is spared and should watch for signs of heart arrhythmias if the potassium is too elevated. This type of diuretic inhibits the action of aldosterone on the kidneys, which does not allow the body to reabsorb sodium. An adverse effect could be hyponatremia. This medication has no effects on phosphate and albumin is not an electrolyte.

661) Which medication requires the nurse to monitor the client for signs of hyperkalemia? Furosemide Metolazone Spironolactone Hydrochlorothiazide

Rationale Spironolactone is a potassium-sparing diuretic; hyperkalemia is an adverse effect. Furosemide, metolazone, and hydrochlorothiazide generally cause hypokalemia. Test-Taking Tip: You have at least a 25% chance of selecting the response in multiple-choice items. If you are uncertain about a question, eliminate the choices that you believe are wrong and then call on your knowledge, skills, and abilities to choose from the remaining responses.

Question 15 The nurse is admitting a client to the hospital with findings of liver failure and ascites. A health care provider (HCP) orders spironolactone. The nurse understands that the pharmacological effects of the medication, are which of the following? Question 15 Answer Choices ACombines safely with antihypertensives Combines safely with antihypertensives BDepletes potassium reserves Depletes potassium reserves CPromotes sodium and chloride excretion Promotes sodium and chloride excretion DIncreases aldosterone levels Increases aldosterone levels Question Explanation

Rationale Spironolactone is considered a diuretic, that is indicated for individuals with hypertension, edema, congestive heart failure and potassium loss. Spironolactone promotes sodium and chloride excretion while sparing potassium and decreasing aldosterone levels. Spironolactone is often combined with other diuretics and anti-hypertensive agents. Kidney function and electrolytes should be monitored more closely when spironolactone is used in combination with other medications. The medication is considered a potassium-sparing diuretic, because as aldosterone levels decrease and sodium and water is excreted, potassium is spared. A major side effect of spironolactone is hyperkalemia.

941) Which dietary choices will the nurse instruct the client taking spironolactone to avoid increasing? Select all that apply. One, some, or all responses may be . Potatoes Red meat Cantaloupe Wheat bread Flavored yogurt

Rationale Spironolactone is potassium-sparing, and beverages and foods containing potassium such as potatoes, cantaloupe, bananas, avocados, oranges, dates, apricots, and raisins should not be increased beyond the client's ordinary consumption to prevent hyperkalemia. Red meat may need to be limited for other reasons not related to spironolactone. Whole grains are associated with prevention of constipation and should not be avoided. Dairy products are rich in sodium and calcium; spironolactone may cause hyponatremia.

319) A client reports to the nurse, 'I've been using St. John's wort to try and feel more like myself again. I'm not sure whether it's going to work.' Which symptom would the nurse further assess? Depression Sleep disturbances Diminished cognitive ability Sensory-perceptual disturbances

Rationale St. John's wort is an herb marketed as a natural way to improve mood and ease feelings of depression. Because St. John's wort is considered a dietary supplement, it is not regulated by the Food and Drug Administration as medications are. It has not been shown to exert positive effects in people with sleep disturbances, diminished cognitive abilities, or sensory-perceptual disturbances. Test-Taking Tip: Once you have decided on an answer, look at the stem again. Does your choice answer the question that was asked? If the question stem asks 'why,' be sure the response you have chosen is a reason. If the question stem is singular, then be sure the option is singular, and the same for plural stems and plural responses. Many times, checking to make sure that the choice makes sense in relation to the stem will reveal the correct answer.

1) Which condition is contraindicated for St. John's wort herbal therapy? Anxiety Seizures Dementia Cardiac disease

Rationale St. John's wort is contraindicated for dementia; this herbal therapy is used to treat anxiety. Bupropion therapy is contraindicated for seizures. Valerian (Valeriana officinalis) is contraindicated for cardiac disease.

1113) A client is scheduled for an adrenalectomy. Which action would the nurse expect in the plan of care? Provide a low-protein diet. Administer parenteral corticosteroids. Collect a preoperative 24-hour urine specimen. Withhold all medications 48 hours before surgery.

Rationale Steroid therapy usually is given intravenously or intramuscularly preoperatively and continued intraoperatively to prepare for the acute adrenal insufficiency that follows surgery. The diet must supply ample protein and potassium. A 24-hour urine specimen is unnecessary. Corticosteroids must be administered preoperatively to prevent adrenal insufficiency during surgery, so withholding all medications for 48 hours before surgery is contraindicated.

1187) A client presents with extensive lesions caused by psoriasis. Which intervention would the nurse anticipate providing teaching on? Advising sunscreen and special clothing Topical application of steroids Potassium permanganate baths Débridement of necrotic plaques

Rationale Steroids are applied locally, and the lesions usually are covered with plastic wrap at night to reverse the inflammatory process. Solar rays may be used for treatment, but other forms of ultraviolet light are preferred. Potassium permanganate is an antiseptic astringent used on infected, draining, or vesicular lesions. The plaques are not necrotic and do not require débriding.

318) Which intervention would the nurse anticipate providing teaching on when a client presents with extensive lesions caused by psoriasis? Advising sunscreen and special clothing Topical application of steroids Potassium permanganate baths Débridement of necrotic plaques

Rationale Steroids are applied locally, and the lesions usually are covered with plastic wrap at night to reverse the inflammatory process. Solar rays may be used for treatment, but other forms of ultraviolet light are preferred. Potassium permanganate is an antiseptic astringent used on infected, draining, or vesicular lesions. The plaques are not necrotic and do not require débriding.

1189) The nurse provides client teaching on the administration of a topical steroid application to a basal cell carcinoma surgical site. The nurse evaluates the teaching as effective when the client identifies which action as the primary purpose of the medication? Preventing infection of the wound Increasing fluid loss from the skin Reducing inflammation at the surgical site Limiting itching around the area of the lesion

Rationale Steroids are used for their anti-inflammatory, vasoconstrictive, and antipruritic effects. Steroids increase the incidence of infections because they are anti-inflammatory agents and mask the symptoms of infection. Steroids increase fluid retention because they promote the reabsorption of sodium from the tubular fluid into the plasma. Although steroid ointments have an antipruritic effect, their major purpose after surgery is their systemic anti-inflammatory effect.

314) Which action is the primary purpose of a topical steroid application to a basal cell carcinoma surgical site? Preventing infection of the wound Increasing fluid loss from the skin Reducing inflammation at the surgical site Limiting itching around the area of the lesion

Rationale Steroids are used for their anti-inflammatory, vasoconstrictive, and antipruritic effects. Steroids increase the incidence of infections because they are anti-inflammatory agents and mask the symptoms of infection. Steroids increase fluid retention because they promote the reabsorption of sodium from the tubular fluid into the plasma. Although steroid ointments have an antipruritic effect, their major purpose after surgery is their systemic anti-inflammatory effect.

774) A client with myasthenia gravis is to receive immunosuppressive therapy with corticosteroids. Which mechanism of action assures the nurse that this therapy will be effective? Inhibits the breakdown of acetylcholine at the neuromuscular junction Stimulates the production of acetylcholine at the neuromuscular junction Decreases the production of autoantibodies that attack acetylcholine receptors Promotes the removal of autoantibodies that impair the transmission of impulses

Rationale Steroids decrease the body's immune response, limiting the production of antibodies that attack acetylcholine receptors at the neuromuscular junction. Inhibiting the breakdown of acetylcholine at the neuromuscular junction is the action of anticholinergic medications. Stimulating the production of acetylcholine at the neuromuscular junction is not the action of immunosuppressives. Promoting the removal of autoantibodies that impair the transmission of impulses is the +Rationale for plasmapheresis.

808) A client with rheumatoid arthritis asks the nurse why it is necessary to inject hydrocortisone into the knee joint. Which reason would the nurse include in a response to this question? Lubricates the joint Reduces inflammation Provides physiotherapy Prevents ankylosis of the joint

Rationale Steroids have an anti-inflammatory effect that can reduce arthritic pannus formation. Injecting hydrocortisone into the joint does not provide lubrication. Injection of a medication into a joint is not physiotherapy. Ankylosis refers to fusion of joints. It is only indirectly influenced by steroids, which exert their major effect on the inflammatory process. Test-Taking Tip: Many times, the is the longest alternative given, but do not count on it. Item writers (those who write the questions) are also aware of this and attempt to avoid offering you such 'helpful hints.'

1202) A 28-year-old woman who is a smoker seeks advice about oral contraceptives. Which response by the nurse is appropriate? 'Oral contraceptives can cause thrombophlebitis.' 'Oral contraceptives must be used with other methods.' 'Some oral contraceptives can be used without concern.' 'Some oral contraceptives are safe, but others are not safe.'

Rationale Studies have shown that women who smoke at least a pack of cigarettes a day are more prone to cardiovascular problems such as thrombophlebitis. Using oral contraceptives with other methods of contraception is not necessary if there are no contraindications; oral contraceptives are effective used alone. There is no 'safe' oral contraceptive for all women or one that may be used without concern; any client at risk should be informed of the potential consequences of taking an oral contraceptive.

1108) A client has a prescription for a sublingual nitroglycerin tablet. Which technique will the nurse teach the client to use? Place the pill inside the cheek and let it dissolve. Place the pill under the tongue and let it dissolve. Chew the pill thoroughly and then swallow it. Swallow the pill with a full glass of water.

Rationale Sublingual medication is placed under the tongue and is quickly absorbed through the mucous membranes into blood. The buccal route requires placing medication between the cheek and gums. Chewing the pill and then swallowing it may be done for oral administration of some large pills, but not with the sublingual route of administration. Taking the pill with water is required with the oral route of administration of medication, but not with sublingual. In addition, a full glass of water may be an excessive amount of fluid to swallow one pill. STUDY TIP: Becoming a nursing student automatically increases stress levels because of the complexity of the information to be learned and applied and because of new constraints on time. One way to decrease stress associated with school is to become very organized so that assignment deadlines or tests do not come as sudden surprises. By following a consistent plan for studying and completing assignments, you can stay on top of requirements and thereby prevent added stress.

Question 15 A child is treated with succimer for lead poisoning. Which of these assessments is the priority? Question 15 Answer Choices ACheck the client's serum potassium level. Check the client's serum potassium level. BCheck the client's blood calcium level. Check the client's blood calcium level. CTest the client's deep tendon reflexes. Test the client's deep tendon reflexes. DCheck the client's complete blood count with differential. Check the client's complete blood count with differential. Question Explanation

Rationale Succimer is used in the management of lead or other heavy metal poisoning. Although it is generally well-tolerated and has a relatively low toxicity, it may cause neutropenia. Succimer therapy should be withheld or discontinued if the absolute neutrophil count (ANC) is below 1,200/mm3. The normal range for an ANC is 1.5 to 8.0 (1,500 to 8,000/mm3). Therefore, the assessment priority in this scenario is checking the complete blood count (CBC) with differential which includes an ANC value.

122) The nurse is to administer a muscle relaxant, succinylcholine, just before a client undergoes electroconvulsive therapy. Which muscles does this medication inhibit? Biceps and triceps Facial and thoracic Intercostal and diaphragmatic Sternocleidomastoid and abdominal

Rationale Succinylcholine causes paralysis of muscles including the intercostals and diaphragm, so artificial support of respiration is required to sustain life during the procedure. Inhibition of the biceps, triceps, facial, thoracic, sternocleidomastoid, and abdominal muscles is the purpose of the medication, not a disadvantage of it. Test-Taking Tip: Pace yourself when taking practice quizzes. Because most nursing exams have specified time limits, you should pace yourself during the practice testing period accordingly. It is helpful to estimate the time that can be spent on each item and still complete the examination in the allotted time. You can obtain this figure by dividing the testing time by the number of items on the test. For example, a 1-hour (60-minute) testing period with 50 items averages 1.2 minutes per question. The NCLEX exam is not a timed test. Both the number of questions and the time to complete the test varies according to each candidate's performance. However, if the test taker uses the maximum of 5 hours to answer the maximum of 265 questions, each question equals 1.3 minutes.

835) A health care provider in the emergency department identifies that a client is in cardiogenic shock. Which type of medication is indicated for management of this condition? Loop diuretic Cardiac glycoside Sympathomimetic Alpha-adrenergic blocker

Rationale Sympathomimetics are vasopressors that induce arterial constriction, which increases venous return and cardiac output. Diuretics promote excretion of fluid, which is not indicated. Cardiac glycosides slow and strengthen the heartbeat; they do not increase the blood pressure and may decrease it. Alpha-adrenergic blockers decrease peripheral resistance, resulting in a decreased blood pressure.

266) Which times for the medication schedule would a nurse teach when corticosteroid therapy is prescribed for a client with an exacerbation of ulcerative colitis? At bedtime with a snack Three times a day with meals In the early morning with food One hour before or 2 hours after eating

Rationale Taking the medication in the early morning mimics usual adrenal secretions; food helps reduce gastric irritation. Diurnal rhythms may be altered, and steroids are ulcerogenic; they should be taken with more than just a snack. Steroids cause gastric irritation and should be taken with food. Although food helps decrease gastric irritation, dividing the dose and taking it throughout the day may alter regular diurnal rhythms; it should be taken in the early morning with food.

178) Which instruction will the nurse include when performing discharge teaching to a client now receiving hydrocortisone by mouth after stabilization of an acute adrenal insufficiency? "Eat a diet high in sodium." "Take the medication with food." "Maintain the same dose indefinitely." "Eliminate a dose if side effects occur."

Rationale Taking the medication with food minimizes the side effect of gastrointestinal irritation; the health care provider should be notified immediately if abdominal pain or tarry stools occur. The diet should be low in sodium because cortisone can cause fluid retention. The dose may have to be adjusted with health care provider supervision when the client is under physical or emotional stress. Cortisone levels must be maintained; changes in dosage must be supervised by the health care provider. Test-Taking Tip: Make educated guesses when necessary.

1420) The nurse provides client teaching on the use of oral contraceptives. Which statement made by the client indicates to the nurse that teaching was effective? 'I will take my pill at the same time every day.' 'I can stop the pill and try to get pregnant right away.' 'I may miss two periods and not worry about being pregnant.' 'I am so glad we won't have to use condoms even if I miss just one pill during the month.'

Rationale Taking the pill at the same time every day makes it more effective, and the client should be instructed to do so. A woman should wait 2 to 3 months after stopping the oral contraceptive pill before attempting to become pregnant. If two consecutive menstrual cycles are missed, the client should stop the contraceptive pill and perform a pregnancy test. The client should use a barrier method of contraception for the first month of pill use and when a pill is missed to help prevent conception. STUDY TIP: Try to decrease your workload and maximize your time by handling items only once. Most of us spend a lot of time picking up things we put down rather than putting them away when we have them in hand. Going straight to the closet with your coat when you come in instead of throwing it on a chair saves you the time of hanging it up later. Discarding junk mail immediately and filing the rest of your bills and mail as they come in rather than creating an ever-growing stack saves time when you need to find something quickly. Filing all items requiring further attention in some fashion helps you remember to take care of things on time rather than being so engrossed in your schoolwork that you forget about them. Many nursing students have had their power or telephone service cut off because the bill simply was forgotten or buried in a pile of old mail.

Question 10 A client with benign prostatic hypertrophy has been prescribed tamsulosin. Which statement by the nurse correctly describes how this medication works? Question 10 Answer Choices A"This medication will shrink your enlarged prostate gland." "This medication will shrink your enlarged prostate gland." B"This medication will eliminate your nocturia." "This medication will eliminate your nocturia." C"Your sexual desire will increase with this medication." "Your sexual desire will increase with this medication." D"This medication will improve the flow of urine." "This medication will improve the flow of urine." Question Explanation

Rationale Tamsulosin is an alpha-adrenergic blocker that is prescribed to promote bladder and prostate gland relaxation for clients with benign prostatic hypertrophy or hyperplasia (BPH). Tamsulosin will relax the smooth muscle of the bladder neck and prostate, allowing urine to flow more easily. Tamsulosin does not shrink the prostate, nor does it increase sexual desire. Although tamsulosin may reduce episodes of having to void during the night (nocturia), it might not eliminate them.

1520) An adolescent with hay fever has been taking a prescribed first-generation antihistamine every 8 hours for the past 2 days. The adolescent tells the nurse, 'This medicine is making me sleepy.' Which response by the nurse would be most appropriate? 'Take half a tablet before school.' 'Try omitting the early morning dose.' 'The drowsiness usually decreases after several days.' 'I'll write your teacher a note to explain your inability to concentrate in class while taking this medicine.'

Rationale Telling the adolescent that the drowsiness will likely disappear after a few days addresses the adolescent's concern; central nervous system depressant effects may diminish or spontaneously disappear after several days of therapy; however, if this does not occur, a second-generation antihistamine may be warranted. Nurses do not have the legal authority to instruct a client to alter the dosage of a prescribed medication. The side effect of drowsiness often diminishes within several days, so it would be inappropriate to write a note that addresses the duration of treatment until it is determined that this will be a problem. STUDY TIP: Remember that intelligence plays a vital role in your ability to learn. However, being smart involves more than just intelligence. Being practical and applying common sense are also part of the learning experience.

Question 6 The nurse is caring for a client who received tenecteplase to open an occluded coronary artery. Which finding should be of highest concern for the nurse? Question 6 Answer Choices AEpistaxis Epistaxis BBleeding gums Bleeding gums CHematemesis Hematemesis DUrinary retention Urinary retention Question Explanation

Rationale Tenecteplase, a thrombolytic agent, breaks down a thrombus by stimulating the plasmin system. The plasmin system is a natural anticlotting system, which breaks down fibrin and dissolves any clots. Since this medication is not specific to a certain type of clot, the client should be expected to have an increased bleeding risk after administration. The most common adverse effect of thrombolytic medications is bleeding and hemorrhage. The nurse should monitor the client for signs and symptoms of abnormal bleeding. Hematemesis means vomiting blood. This is usually related to a bleeding gastric ulcer and should be of highest concern. Epistaxis (nose bleed) and bleeding gums are usually minor bleeding and can be easily monitored by the nurse.

1473) A health care provider prescribes teriparatide for a client with osteoporosis. Which statement about this medication would the nurse recognize as accurate? It requires increased intake of vitamin A. It prevents existing bone from being destroyed. Sunscreen should be used to prevent vitamin D absorption. Osteoblastic activity is stimulated more than osteoclastic activity.

Rationale Teriparatide is a 34-amino acid polypeptide that represents the biologically active part of human parathyroid hormone; it enhances bone microarchitecture and increases bone mass and strength by stimulating activity by osteoblasts. Supplemental intake of vitamin A should not exceed recommended daily requirements; too much vitamin A has been associated with bone loss and an increased incidence of fractures. Alendronate sodium, a regulator of bone metabolism, not teriparatide, inhibits osteoclast-mediated bone resorption, minimizing bone destruction and loss of bone density. Sunscreen should be avoided to promote exposure to the sun so that vitamin D can be converted in the skin; vitamin D helps the body absorb calcium. Sunscreen should be used after 5 to 20 minutes of exposure to prevent the negative effects of prolonged exposure to ultraviolet rays.

689) Which information would the nurse include when teaching a client about warfarin? Periodic blood testing is necessary. Increase intake of green leafy vegetables. Limit the amount of daily physical activity. It should be continued for minor surgical procedures.

Rationale Testing is essential to determine dosing; a therapeutic prothrombin time (PT) ranges from 1.3 to 1.5 times greater than the control and is equal to an international normalized ratio (INR) of 2 to 3 times control. Green leafy vegetables are high in vitamin K, which may decrease medication effectiveness if eaten in large amounts. Physical activities do not need to be limited; however, the type (e.g., contact sports such as football) may need to be restricted. Warfarin will need to be stopped for most dental, medical, and surgical procedures; the provider should be contacted regarding the need to hold the medication.

687) A client taking multiple medications for hypertension develops a persistent, hacking cough. Which antihypertensive medication class would the nurse identify as the likely cause of the cough? Thiazide diuretics Calcium channel blockers Direct renin inhibitors Angiotensin-converting enzyme (ACE) inhibitors

Rationale The ACE breaks down kinins. When ACE is inhibited, the increase of kinins in the lung can cause bronchial irritation, leading to the common adverse effect sometimes referred to as an ACE cough. A cough is not a side effect of thiazide diuretics, calcium channel blockers, or direct renin inhibitors. Test-Taking Tip: Never leave a question unanswered. Even if answering is no more than an educated guess on your part, go ahead and mark an answer. You might be right, but if you leave it blank, you will certainly be wrong and lose precious points.

1198) A pregnant client with severe preeclampsia is receiving intravenous magnesium sulfate. Which item would the nurse keep at the bedside in case of magnesium sulfate toxicity? Oxygen Naloxone Calcium gluconate Suction equipment

Rationale The antagonist of magnesium sulfate is calcium gluconate. Oxygen is ineffective if the action of magnesium is not reversed. Naloxone is unnecessary; it is an opioid antagonist. Suction equipment may be necessary if the client has excessive secretions after a seizure. The priority intervention is to try to prevent a seizure. STUDY TIP: A helpful method for decreasing test stress is to practice self-affirmation. After you have adequately studied and really know the material, start looking in the mirror each time you pass one and say to yourself—preferably out loud—'I know this material, and I will do well on the test.' After several times of watching and hearing yourself reaffirm your knowledge, you will gain inner confidence and be able to perform much better during the test period. This technique really works for students who are adventurous enough to use it. It may feel silly at first, but if it works, who cares? It will work for performing skills in clinical as well, as long as you have practiced the skill sufficiently.

493) Which drink would a nurse teach a client on warfarin to avoid? Apple juice Grape juice Orange juice Cranberry juice

Rationale The antioxidants in cranberry juice may inhibit the mechanism that metabolizes warfarin, causing elevations in the international normalized ratio, resulting in hemorrhage. Apple juice, grape juice, and orange juice are fine to drink.

979) A client who is receiving atenolol for hypertension frequently reports feeling dizzy. Which effect of atenolol is responsible for this response? Depleting acetylcholine Stimulating histamine release Blocking the adrenergic response Decreasing adrenal release of epinephrine

Rationale The beta-adrenergic blocking effect of atenolol decreases the heart's rate and contractility; it may result in orthostatic hypotension and decreased cerebral perfusion, causing dizziness. Depleting acetylcholine is not an action of atenolol. Stimulating histamine release is not an action of atenolol. Decreasing adrenal release of epinephrine is not an action of atenolol.

828) A client who recently started receiving oral corticosteroids for a severe allergic reaction is instructed that the dosage will be reduced gradually until all medication is stopped at the end of 2 weeks. Which reason would the nurse provide for this gradual reduction in dosage? Discontinuing the medication too fast will cause the allergic reaction to reappear. Slow reduction of the medication will prevent a physiological crisis because the adrenal glands are suppressed. The health care provider is attempting to determine the minimal dose that will be effective for the allergy. Sudden cessation of the medication will cause development of serious side effects, such as moon face and fluid retention.

Rationale The body's natural corticosteroid production has been suppressed during treatment; avoiding abrupt cessation of the medication will give the body time to adjust to less and less of the exogenous source and to resume secretion of endogenous corticosteroid. Not completing the course of therapy, rather than stopping it quickly, may cause signs and symptoms of the allergy to recur. The health care provider has already determined the correct dosage, and it has been prescribed. Moon face and fluid retention are associated with long-term steroid use, not with the cessation of therapy.

1501) The nurse is caring for a child receiving furosemide for pulmonary edema. Which nursing intervention(s) would the nurse implement? Select all that apply. One, some, or all responses may be . Checking the child's weight every day Administering the medication on an empty stomach Calculating the dose of medication as carefully as possible Exposing the child to sunlight for increasing periods Assessing the child regularly to help prevent electrolyte loss

Rationale The child's weight should be checked and recorded daily to aid in the assessment of therapeutic and adverse effects. Pediatric doses should be calculated carefully to prevent an accidental overdose. Pediatric clients are at greater risk of electrolyte loss; therefore they require closer and more cautious assessment to help prevent hypertension and stroke. Furosemide may cause stomach upset if it is taken on an empty stomach; the child should be given the medication with food to help prevent gastric upset. A child taking diuretics should not be exposed to sunlight for long periods, because this action may precipitate fluid volume loss and heatstroke. Test-Taking Tip: Identifying content and what is being asked about that content is critical to your choosing the response. Be alert for words in the stem of the item that are the same or similar in nature to those in one or two of the options.

1173) A male client receiving prolonged steroid therapy complains of always being thirsty and urinating frequently. Which is the nurse's initial action? Have the client assessed for an enlarged prostate. Obtain a urine specimen from the client to test for ketonuria. Perform a finger stick to test the client's blood glucose level. Assess the client's lower extremities for the presence of pitting edema.

Rationale The client has signs of an increased serum glucose level, which may result from steroid therapy; testing the blood glucose level is a method of gathering more data. The symptoms are not those of benign prostatic hyperplasia. The blood glucose level, not the amount of ketones in the urine, should be assessed. The symptoms presented are not those of fluid retention but of hyperglycemia. Test-Taking Tip: Pace yourself during the testing period and work as accurately as possible. Do not be pressured into finishing early. Do not rush! Students who achieve higher scores on examinations are typically those who use their time judiciously.

359) Which action will a nurse take when a male client receiving prolonged steroid therapy complains of always being thirsty and urinating frequently? Have the client assessed for an enlarged prostate. Obtain a urine specimen from the client to test for ketonuria. Perform a finger stick to test the client's blood glucose level. Assess the client's lower extremities for the presence of pitting edema.

Rationale The client has signs of an increased serum glucose level, which may result from steroid therapy; testing the blood glucose level is a method of gathering more data. The symptoms are not those of benign prostatic hyperplasia. The blood glucose level, not the amount of ketones in the urine, should be assessed. The symptoms presented are not those of fluid retention but of hyperglycemia. Test-Taking Tip: Pace yourself during the testing period and work as accurately as possible. Do not be pressured into finishing early. Do not rush! Students who achieve higher scores on examinations are typically those who use their time judiciously.

1194) A client diagnosed with a transient ischemic attack (TIA) related to hypertension is discharged with a prescription of hydrochlorothiazide. Which instruction would the nurse include when teaching about this medication? 'Resume regular eating habits.' 'Drink a protein supplement daily.' 'Avoid eating foods high in insoluble fiber.' 'Increase the intake of potassium-rich foods.'

Rationale The client must increase the dietary intake of potassium because of potassium loss associated with hydrochlorothiazide. The client should be taught about medication-induced deficiencies, which may necessitate a change in diet, and not just return to regular eating habits once home. Protein supplements are not necessary, and protein may be obtained from meat, fish, and dairy products in the diet or complementary vegetable and grain proteins. Foods high in insoluble fiber are part of the food pyramid and should be included in the diet. Test-Taking Tip: Do not read information into questions, and avoid speculating. Reading into questions creates errors in judgment.

300) Which instruction would the nurse include when teaching about hydrochlorothiazide given to a client diagnosed with a transient ischemic attack (TIA) related to hypertension? "Resume regular eating habits." "Drink a protein supplement daily." "Avoid eating foods high in insoluble fiber." "Increase the intake of potassium-rich foods."

Rationale The client must increase the dietary intake of potassium because of potassium loss associated with hydrochlorothiazide. The client should be taught about medication-induced deficiencies, which may necessitate a change in diet, and not just return to regular eating habits once home. Protein supplements are not necessary, and protein may be obtained from meat, fish, and dairy products in the diet or complementary vegetable and grain proteins. Foods high in insoluble fiber are part of the food pyramid and should be included in the diet. Test-Taking Tip: Do not read information into questions, and avoid speculating. Reading into questions creates errors in judgment.

Question 6 The nurse is teaching the client how to properly use a dry powder capsule inhaler. How should the nurse instruct the client to use this type of inhaler? Question 6 Answer Choices AShake inhaler before putting it in mouth Shake inhaler before putting it in mouth BSeal lips tightly around mouthpiece and inhale rapidly and deeply Seal lips tightly around mouthpiece and inhale rapidly and deeply CRinse mouthpiece in hot soapy water after using Rinse mouthpiece in hot soapy water after using DBreathe in medicine slowly and deeply for about 3-5 seconds Breathe in medicine slowly and deeply for about 3-5 seconds Question Explanation

Rationale The client should breathe in quickly and deeply for up to 10 seconds when using a dry powder capsule inhaler. The client should not shake this type of inhaler. The mouthpiece can be rinsed with warm water but without soap or detergent.

186) Which instruction regarding nutrition will the nurse give a client discharged after a short hospitalization for an episode of a transient ischemic attack (TIA) related to hypertension who is on a regimen that includes chlorothiazide? "Eat more dark green, leafy vegetables such as spinach." "Substitute a potassium-based salt substitute for table salt." "Return to previous eating habits." "Increase intake of dairy products."

Rationale The client should increase the dietary intake of potassium because of potassium loss associated with chlorothiazide. Leafy green vegetables are high in potassium and should be encouraged. Salt substitutes should only be used if prescribed by the provider; otherwise, they should be discouraged because electrolyte abnormalities may occur without close monitoring. Returning to previous eating habits may be unsafe for those who do not consume a nutritional diet; the client should be taught about medication-induced deficiencies and how to try to prevent future TIAs. Dairy products should be limited, unless fat-free, because they are high in saturated fats. Test-Taking Tip: Come to your test prep with a positive attitude about yourself, your nursing knowledge, and your test-taking abilities. A positive attitude is achieved through self-confidence gained by effective study. This means (a) answering questions (assessment), (b) organizing study time (planning), (c) reading and further study (implementation), and (d) answering questions (evaluation).

460) Which information is most important for the nurse to teach a client prescribed an antihypertensive medication to be taken once in the morning and a 2-gram sodium diet? "Avoid adding salt to cooked foods." "Use less salt when preparing foods." "Take your medicine exactly as prescribed." "Measure your blood pressure every morning."

Rationale The most effective way to lower the blood pressure is to take the prescribed medication daily. Restricting salt in the diet will help limit fluid retention and thus reduce the blood pressure, but it is not as effective as an antihypertensive. Salt should not be added during food preparation. The natural sodium content of foods should be calculated in a 2-gram sodium diet. It is not necessary to take daily blood pressure measurements unless specifically prescribed to do so by the primary health care provider.

1034) The nurse provides instructions about how to use a metered-dose inhaler (MDI) to a client with chronic obstructive pulmonary disease. The nurse concludes that additional teaching is needed when the client demonstrates which technique? Places the tip of the inhaler just past the lips Holds the inspired breath for at least 3 seconds Activates the inhaler during inspiration Inhales rapidly with the lips sealed around the nebulizer opening

Rationale The client should inhale slowly rather than rapidly when using a metered-dose inhaler (MDI) to optimize delivery of the nebulized medication into the lungs. If the client has a dry powder inhaler (DPI), then rapid inhaling would be an important action because the powder is not nebulized. The MDI should be gently held in the mouth just past the lips to deliver the medication into the airway. Holding the inspired breath for at least 3 seconds promotes contact of the medication with the bronchial mucosa. The inhaler should be activated during inspiration.

1200) A client asks the nurse what she should do if she forgets to take her contraceptive pill 1 day. Which response by the nurse is appropriate? 'Take your pills as instructed.' 'Call your primary health care provider immediately.' 'Continue as usual, and there shouldn't be a problem.' 'On the next day take 1 pill in the morning and 1 pill before bedtime.'

Rationale The client should make up for the missed pill by taking 2 pills the next day; taking 1 pill in the morning and 1 pill in the evening decreases the chance of the client becoming nauseated. Telling the client to take her pills as instructed does not explain what is to be done if a pill is missed; missing 1 pill can alter hormone levels and predispose the client to becoming pregnant. It is unnecessary to call the primary health care provider unless other problems are identified. Telling the client that there should be no problem if she continues as usual is incorrect advice; again, missing 1 pill can alter hormone levels and predispose a woman to pregnancy.

511) The nurse teaches a client about cortisone therapy. Which statements made by the client indicate the need for further teaching? Select all that apply. One, some, or all responses may be correct. 'I should take 3 tablets at a time.' 'I should take the tablet with water.' 'I should take the tablet twice a week.' 'I should take the tablet on an empty stomach.' 'I should take the tablet with a meal.'

Rationale The client should take the medication as prescribed. The client should not take 3 tablets at a time because this action may lead to drug toxicity. Cortisone therapy involves the administration of 25 to 50 mg of cortisone on a daily basis. Cortisone should be taken with a meal or a snack; taking the medication on an empty stomach would cause gastric irritation. Tablets can be taken with any fluid such as water or fruit juice.

Question 5 The nurse is providing care for a client admitted to the hospital with a diagnosis of digoxin toxicity. The client reports more than usual urine output over the previous 48 hours, because of the prescribed diuretic. Which assessment finding does the nurse anticipate? Question 5 Answer Choices AMuscle weakness or cramping Muscle weakness or cramping BBlood in the urine Blood in the urine CHypertension Hypertension DTinnitus Tinnitus Question Explanation

Rationale The client with heart failure on digoxin and a diuretic is at risk for hypokalemia. The digoxin binds to the potassium receptor of the sodium/potassium ATPase pump. The increased urine output makes hypokalemia likely and thus it is more likely for digoxin toxicity to occur. Symptoms of hypokalemia include muscle weakness and cramping. The digoxin toxicity will not cause blood in the urine, or tinnitus or hypertension.

Question 8 The nurse is caring for a client who is receiving a continuous intravenous heparin infusion. The client's most recent activated partial thromboplastin time (aPTT) is 120 seconds. Which medication should the nurse plan to administer? Question 8 Answer Choices AProtamine Protamine BNaloxone Naloxone CVitamin K Vitamin K DEnoxaparin Enoxaparin Question Explanation

Rationale The client's aPTT is much higher than the typical desired therapeutic range of 1.5-2.5 the control value and places the client at great risk for uncontrolled bleeding. Protamine sulfate is the medication used to reverse the effects of heparin; it is a heparin antagonist. Neutralization of heparin occurs immediately and lasts for 2 hours, after which additional protamine may be needed. Protamine is administered by slow IV injection (no faster than 20 mg/ min or 50 mg in 10 minutes). Dosage is based on the fact that 1 mg of protamine will inactivate approx. 100 units of heparin. Vitamin K is used to reverse the effects of warfarin. Naloxone is used to reverse the effects of opioids. Enoxaparin is another anticoagulant (low molecular weight heparin).

692) Which clinical finding indicates that a client taking digoxin may have developed digoxin toxicity? Constipation Decreased urination Cardiac dysrhythmias Metallic taste in the mouth

Rationale The development of cardiac dysrhythmias is often a sign of digoxin toxicity. Constipation is not a sign of toxicity; gastrointestinal signs and symptoms of toxicity include anorexia, nausea, vomiting, and diarrhea. Decreased urination is not a sign of toxicity. Digoxin does not cause a metallic taste in the mouth.

943) The international normalized ratio (INR) results of a client receiving warfarin have been variable. Which factor can help the nurse identify the cause of the INR fluctuations? Intake of foods high in potassium Serum glucose level Platelet count Adherence to the prescribed medication regimen

Rationale The dosage of warfarin is adjusted according to INR results; if the client fails to take the medication as prescribed, test results will not be reliable in monitoring the client's response to therapy. Intake of foods high in vitamin K, not potassium, is important to ascertain. Low platelet counts increase the risk for bleeding; however, they do not affect warfarin activity or the INR. Serum glucose levels do not affect the warfarin or the INR.

1341) Which conditions would the nurse identify as decreasing the effectiveness of estrogen therapy? Select all that apply. One, some, or all responses may be correct. Habit of smoking Use of anticoagulants Use of tricyclic antidepressants Presence of endometrial cancer Presence of thromboembolic disorders

Rationale The effectiveness of estrogen therapy decreases with smoking and decreases with the use of anticoagulants. The use of tricyclic antidepressants along with estrogen may result in medication toxicity. Estrogen therapy is not recommended in clients with endometrial cancer and thromboembolic disorders because it may increase the risk of these complications. Test-Taking Tip: Make certain that the answer you select is reasonable and obtainable under ordinary circumstances and that the action can be carried out in the given situation.

Question 18 The nurse is evaluating the effectiveness of therapy for a client who received albuterol via nebulizer during an acute episode of shortness of breath due to asthma. Which finding is the best indicator that the therapy was effective? Question 18 Answer Choices AAccessory muscle use has decreased. Accessory muscle use has decreased. BOxygen saturation is greater than 90%. Oxygen saturation is greater than 90%. CRespiratory rate is 16 breaths/minute. Respiratory rate is 16 breaths/minute. DNo wheezes are audible. No wheezes are audible. Question Explanation

Rationale The goal for treatment of an asthma attack is to relieve bronchospasms and keep the oxygen saturation greater than 90%. Albuterol is a short-acting inhaled beta2-adrenergic agonist and the treatment of choice for an acute asthma attack. Pulse oximetry is an objective data point that the nurse should use to determine oxygenation status of the client. The other client data may occur when the client is too fatigued to continue with the increased work of breathing required in an asthma attack and, therefore, should not be used to evaluate effectiveness of treatment.

895) A client hospitalized for uncontrolled hypertension and chest pain was started on a daily diuretic 2 days ago upon admission, with prescriptions for a daily basic metabolic panel. The client's potassium level this morning is 2.7 mEq/L (2.7 mmol/L). Which action will the nurse take next? Send another blood sample to the laboratory to retest the serum potassium level. Notify the health care provider that the potassium level is above normal. Notify the health care provider that the potassium level is below normal. No action is required because the potassium level is within normal limits.

Rationale The health care provider should be notified immediately because the client's potassium is below normal. The normal potassium level range is 3.5 mEq/L to 5.0 mEq/L (3.5-5.0 mmol/L). Clients on diuretic therapy require close monitoring of their electrolytes because some can cause hypokalemia, whereas others spare potassium, which can cause hyperkalemia. Retesting the serum potassium level is unnecessary and will delay the treatment required by the client. STUDY TIP: You have a great resource in your classmates. We all have different learning styles, strengths, and perspectives on the material. Participating in a study group can be a valuable addition to your nursing school experience.

718) A client with a myocardial infarction receives intravenous nitroglycerin to relieve pain. The nurse will assess for which medication side effect? Nausea Delirium Bradycardia Hypotension

Rationale The major action of intravenous nitroglycerin is venous and then arterial dilation, leading to a decrease in blood pressure and resulting in decreased cardiac workload. Nausea is not a common side effect of intravenous nitroglycerin. Nitroglycerin does not cause delirium. Reflex tachycardia may occur with the decrease in blood pressure.

1069) A client presents to the emergency department with chest pain. A myocardial infarction is suspected, and 500 mL of 5% dextrose in water (D 5W) with 50 mg of nitroglycerin intravenously (IV) has been prescribed. The nurse will monitor the client for which common side effect of nitroglycerin? Bradycardia Hypotension Nausea and vomiting Leg cramps

Rationale The major action of intravenous nitroglycerin is venous and then arterial dilation, leading to a decrease in blood pressure; orthostatic hypotension can occur. Bradycardia is not an anticipated response. Nausea and vomiting may occur but are not the most common side effects of IV nitroglycerin. Leg cramps are not a side effect of this medication.

Question 5 The nurse is teaching a client who has a new prescription for sublingual nitroglycerin. Which point should the nurse emphasize? Question 5 Answer Choices ATake the medication at the same time each day Take the medication at the same time each day BRest in bed for an hour after taking medication Rest in bed for an hour after taking medication CCarry the nitroglycerine with you at all times Carry the nitroglycerine with you at all times DKeep the medication bottle in the refrigerator Keep the medication bottle in the refrigerator Question Explanation

Rationale The medication should be kept in its original dark-colored glass container. Nitroglycerin should be carried by the client at all times so it can be used when anginal pain occurs. When needed, the client should sit and place tablet under his or her tongue. Sitting is safe because the drug can cause lightheadedness or dizziness, but it's not necessary to rest in bed. The client should never pack this and any other medications in a checked a bag when traveling.

1078) A client with hypertension is prescribed an angiotensin II receptor blocker (ARB). Which instructions will the nurse provide about this medication? Select all that apply. One, some, or all responses may be . 'Monitor the blood pressure daily.' 'Stop treatment if a cough develops.' 'Stop the medication if swelling of the mouth, lips, or face develops.' 'Have blood drawn for potassium levels 2 weeks after starting the medication.' 'Do not take nonsteroidal anti-inflammatory drugs (NSAIDs) concurrently with this medication.'

Rationale The medication should be stopped if angioedema occurs, and the health care provider should be notified. Electrolyte levels of potassium, sodium, and chloride should be obtained 2 weeks after the start of therapy and then periodically thereafter. Daily monitoring of blood pressure is not indicated. There is no need to avoid the use of NSAIDs while taking an ARB. A dry cough may occur during treatment with ARBs; however, it is not necessary to discontinue the medication because the cough usually resolves.

Question 9 The client has been treated with long-term glucocorticoid therapy. While completing the physical assessment, which finding should the nurse expect? Question 9 Answer Choices AJaundice Jaundice BPeripheral edema Peripheral edema CBuffalo hump Buffalo hump DIncreased muscle mass Increased muscle mass Question Explanation

Rationale The most common side effects of glucocorticoid therapy include increased appetite including weight gain, increased blood glucose, acne, thinning of the skin, easy bruising and change in body shape (increase in fatty tissue on the trunk with thinner legs and arms). The client may also develop a hump behind the shoulders due to the accumulation of fat on the back of the neck. This is referred to as a buffalo hump. Jaundice, peripheral edema and increased muscle mass are not side effects of glucocorticoid therapy.

Question 1 The nurse is monitoring a 4-month-old infant who is prescribed digoxin. The infant's blood pressure is 92/78 mm Hg; resting pulse is 78 beats per minute; respirations are 28 breaths per minute; and serum potassium level is 4.8 mEq/L. The infant is irritable and has vomited twice since receiving the morning dose of digoxin. Which finding is most indicative of digoxin toxicity? Question 1 Answer Choices AIrritability Irritability BVomiting Vomiting CBradycardia Bradycardia DDyspnea Dyspnea Question Explanation

Rationale The most common sign of digoxin toxicity in children is bradycardia which is a heart rate below 100 beats per minute in an infant. Normal resting heart rate for infants 1-11 months-old is 100-160 beats per minute.

Question 7 A client is being discharged with a prescription for warfarin. Which information is most important to be included in the nurse's discharge teaching? Question 7 Answer Choices ATake acetaminophen for minor pain Take acetaminophen for minor pain BUse a soft toothbrush Use a soft toothbrush CAvoid eating leafy green vegetables Avoid eating leafy green vegetables DReport nose or gum bleeding Report nose or gum bleeding Question Explanation

Rationale The most important teaching is to make sure that the client understands to report any sign of bleeding including nose or gum bleeding, blood noted in stools or urine, coughing up blood, or easy bruising. Dark green leafy vegetables are high in vitamin K which can lower the effectiveness of warfarin (Coumadin). Acetaminophen does not contain aspirin which can cause internal bleeding so is safe to use when taking warfarin. A soft toothbrush will be less irritating to the gums and therefore decrease the risk of bleeding gums. Although green leafy vegetables contain Vitamin K, it is no longer recommended to avoid them but to keep their intake consistent.

526) Which action would be the most appropriate way for the nurse to evaluate a child's understanding of how to use an inhaler? Asking questions about using the inhaler Having the child demonstrate inhaler use Explaining how the inhaler will be used at home Having the child tell the nurse about the technique that was learned

Rationale The nurse can best evaluate teaching by asking the learner for a return demonstration. Behavior, rather than words, more easily shows what has been learned. A child may be too young to know whether he or she has any questions. A demonstration, rather than an explanation, can be evaluated more readily. Telling the nurse about the technique that was learned is difficult for a younger child; the ability to articulate a concept is not that advanced—nor is the vocabulary. Test-Taking Tip: When using this program, be sure to note whether you guess at an answer. This will permit you to identify areas that need further review. Also it will help you see how correct your guessing can be.

Question 12 The nurse is caring for a client who was recently prescribed atropine as a treatment for symptomatic bradycardia. Which condition should the nurse question as a contraindication when taking this medication? Question 12 Answer Choices AUrinary incontinence Urinary incontinence BRight-sided heart failure Right-sided heart failure CGlaucoma Glaucoma DIncreased intracranial pressure Increased intracranial pressure Question Explanation

Rationale The nurse should question the use of atropine with a client who has glaucoma. Atropine is contraindicated in clients with angle-closure glaucoma because it can cause pupillary dilation with an increase in aqueous humor. This can lead to an increase in optic pressure causing blurred vision and ocular pain.

Question 16 A 48-year-old male client who is being admitted to the emergency department with an acute myocardial infarction (MI) gives the following list of medications to the nurse. Which medication would the nurse recognize as having the most immediate implications for the client's care? Question 16 Answer Choices ALosartan Losartan BCaptopril Captopril CFurosemide Furosemide DSildenafil Sildenafil Question Explanation

Rationale The nurse will need to avoid giving nitrates to the client because nitrate administration, commonly prescribed for clients experiencing an acute MI, is contraindicated in clients who are using sildenafil (a PDE5 inhibitor) because of the risk of severe hypotension caused by vasodilation. The other medications the client is taking should also be documented and reported to the health care provider (HCP) but do not have as immediate an impact on decisions about the client's treatment.

1065) The nurse is preparing to apply nitroglycerin ointment. Before applying the ointment, which action will the nurse take? Assess the client's pulse rate. Prepare the site with an alcohol swab. Shave the client's chest in the area for application. Use the dose measuring application paper and spread the ointment in a thin layer to the prescribed amount.

Rationale The nurse would use the dose measuring application paper supplied with the ointment and spread in a thin layer to the prescribed amount and place side down on the desired skin. The nurse would assess blood pressure reading, not pulse rate. There is no need to clean the site with alcohol before administration. Shaving is not recommended; a hairless site on the chest, back, abdomen, or anterior thigh should be selected.

763) Which method would the nurse recommend when teaching the client with asthma how to determine if an inhaler is empty? Track the number of doses taken. Taste the medication when sprayed into the air. Shake the canister. Place the canister in water to see if it floats.

Rationale The only way to determine if the canister is empty is to track the number of doses taken. It is wasteful to spray medication into the air; tasting it from the air is not an effective method of determining if the canister is empty. Shaking the canister is not effective; even if there is no more medication, some propellant may be left. It is futile to place the canister in water; the flotation test is ineffective.tory center in the brain.

691) Nitroglycerin sublingual tablets are prescribed for a client with the diagnosis of angina. The nurse advises the client to anticipate pain relief will begin within which period of time? 1 to 3 minutes 4 to 5 seconds 30 to 45 seconds 10 to 15 minutes

Rationale The onset of action of sublingual nitroglycerin tablets is rapid (1-3 minutes); duration of action is 30 to 60 minutes. If nitroglycerin is administered intravenously, the onset of action is immediate, and the duration is 3 to 5 minutes. It takes longer than 30 to 45 seconds for sublingual nitroglycerin tablets to have a therapeutic effect. Sustained-release nitroglycerin tablets start to act in 20 to 45 minutes, and the duration of action is 3 to 8 hours.

Question 12 The nurse is teaching a school-aged child and family members about the use of inhalers prescribed for asthma. Which statement made by a family member indicates an understanding of the nurse's instructions? Question 12 Answer Choices A"We will keep a chart of daily peak flow meter results." "We will keep a chart of daily peak flow meter results." B"We can rely on our child's self-report of symptoms." "We can rely on our child's self-report of symptoms." C"Monitoring our child's pulse rate is not necessary." "Monitoring our child's pulse rate is not necessary." D"Skin color changes in our child is an early warning sign for airway constriction." "Skin color changes in our child is an early warning sign for airway constriction." Question Explanation

Rationale The peak flow meter can help determine if the symptoms of asthma are in control or are worsening. It works by measuring how fast air comes out of the lungs when the client forcefully exhales (the peak expiatory flow or PEF). The client should record the highest of three readings in an asthma diary daily. Children ages 4 and up should be able to use a peak flow meter. A decrease in PEF is an early warning sign for airway constriction and should be immediately addressed. Family members should monitor the child's pulse rate and changes in skin color is a late sign.

779) The health care provider prescribes enoxaparin to be administered subcutaneously. To ensure client safety, which measure would the nurse take when administering this medication? Remove air pocket from the prepackaged syringe before administration. Rub the injection site for 30 seconds after administration. Administer the medication over 2 minutes. Administer in the abdomen area only.

Rationale The preferred site for enoxaparin administration is the abdomen. According to package directions, the air pocket in the prepackaged syringe should not be removed. Rubbing the injection site also is contraindicated. Subcutaneous injections should not be given over 2 minutes. STUDY TIP: Becoming a nursing student automatically increases stress levels because of the complexity of the information to be learned and applied and because of new constraints on time. One way to decrease stress associated with school is to become very organized so that assignment deadlines or tests do not come as sudden surprises. By following a consistent plan for studying and completing assignments, you can stay on top of requirements and thereby prevent added stress.

22) The health care provider prescribes alprazolam 5 mg by mouth three times a day for a client with anxiety. Which intervention will the nurse take before administering this prescription? Assess the apical pulse. Check the blood pressure. Encourage the ventilation of feelings. Clarify the prescription with the health care provider.

Rationale The prescribed dosage is excessive, and it must be questioned before its administration. Checking the apical pulse and/or blood pressure is not indicated. Encouraging the client to ventilate feelings does not affect the need to question the prescription. Therapeutic dosages of alprazolam range from 0.75 mg to 4 mg daily; the maximal daily dose for acute anxiety is 8 mg. Test-Taking Tip: Practicing a few relaxation techniques may prove helpful on the day of an examination. Relaxation techniques such as deep breathing, imagery, head rolling, shoulder shrugging, rotating and stretching of the neck, leg lifts, and heel lifts with feet flat on the floor can effectively reduce tension while causing little or no distraction to those around you. It is recommended that you practice one or two of these techniques intermittently to avoid becoming tense. The more anxious and tense you become, the longer it will take you to relax.

217) Which statement regarding mealtime administration by a client who has arthritis and is prescribed corticosteroid medication indicates that the teaching was effective? "This will decrease gastric irritation." "This will serve as a reminder to take the medication." "The presence of food will enhance absorption." "The medication is ineffective in an acid medium."

Rationale The presence of food limits the irritating effect of steroids on the gastric mucosa. Taking the medication at mealtime may help the client remember to take the medication, but it is not the reason for taking it with meals. Food does not increase or decrease absorption of steroids. The medication is not affected by an acid environment.

694) Which instruction would the nurse include when teaching the client about sublingual nitroglycerin? 'Once the tablet is dissolved, spit out the saliva.' 'Take tablets 3 minutes apart up to a maximum of five tablets.' 'Common side effects include headache and low blood pressure.' 'Once opened, the tablets should be refrigerated to prevent deterioration.'

Rationale The primary side effects of nitroglycerin are headache and hypotension. It is not necessary to spit out saliva into which nitroglycerin has dissolved. For pain that is not relieved, additional tablets may be taken every 5 minutes up to a total of three tablets. It should be stored at room temperature.

567) According to developmental norms for a 5-year-old child, the nurse would hold digoxin if an apical heart rate falls below which number? 70 beats/min 80 beats/min 90 beats/min 100 beats/min

Rationale The purpose of digoxin is to slow and strengthen the apical rate. The apical rate for a healthy child of 5 years is 70 to 110 beats/min. If the apical rate is slow, administration of the medication may lower the apical rate to an unsafe level. Test-Taking Tip: Pace yourself during the testing period and work as accurately as possible. Do not be pressured into finishing early. Do not rush! Students who achieve higher scores on examinations are typically those who use their time judiciously.

306) Which response would a nurse give to a client who takes furosemide and digoxin and reports that everything looks yellow? "This is related to your heart problems, not to the medication." "I will hold the medication until I consult with your health care provider." "It is a medication that is necessary, and that side effect is only temporary." "Take this dose, and when I see your health care provider, I will ask about it."

Rationale The response "I will hold the medication until I consult with your health care provider" is a safe practice because yellow vision indicates digitalis toxicity. The response "This is related to your heart problems, not to the medication" is in; yellow vision is not a symptom of heart disease. The response "It is a medication that is necessary, and that side effect is only temporary" is in; yellow vision is not a temporary side effect. The response "Take this dose, and when I see your health care provider, I will ask about it" is unsafe. Test-Taking Tip: Once you have decided on an answer, look at the stem again. Does your choice answer the question that was asked? If the question stem asks "why," be sure the response you have chosen is a reason. If the question stem is singular, then be sure the option is singular, and the same for plural stems and plural responses. Many times, checking to make sure that the choice makes sense in relation to the stem will reveal the answer.

1191) A client who takes furosemide and digoxin reports to the nurse that everything looks yellow. Which response by the nurse is most appropriate? 'This is related to your heart problems, not to the medication.' 'I will hold the medication until I consult with your health care provider.' 'It is a medication that is necessary, and that side effect is only temporary.' 'Take this dose, and when I see your health care provider, I will ask about it.'

Rationale The response 'I will hold the medication until I consult with your health care provider' is a safe practice because yellow vision indicates digitalis toxicity. The response 'This is related to your heart problems, not to the medication' is in; yellow vision is not a symptom of heart disease. The response 'It is a medication that is necessary, and that side effect is only temporary' is in; yellow vision is not a temporary side effect. The response 'Take this dose, and when I see your health care provider, I will ask about it' is unsafe.

1089) Sublingual nitroglycerin is prescribed for a client with a history of a myocardial infarction and atrial tachycardia. The nurse instructs the client about the prophylactic use of these tablets. Which statement by the client indicates the teaching was effective? 'I should take the medicine three times a day.' 'I will be sure to take my pulse after I have exercised.' 'It will be important to avoid activities that can cause angina.' 'I should take one tablet before attempting activity that has caused angina.'

Rationale The response about taking one tablet before activity that has caused angina indicates that the client understands the nurse's teaching. Taking a nitroglycerin tablet before such an activity probably will prevent an episode of angina, which is an example of prophylactic use of a medication. Taking the medicine three times a day is an example of scheduled administration of a nitrate medication for prophylaxis, but the client is being prescribed sublingual nitrate. The statement to avoid activities that can cause angina indicates avoidance of activity rather than taking medication to prevent angina during the activity. Blood pressure, not pulse, is the parameter most affected by nitroglycerin.

869) The nurse is providing discharge medication teaching to a client who will be taking furosemide and digoxin after discharge from the hospital. Which information is important for the nurse to include in the teaching plan? Maintenance of a low-potassium diet Avoidance of foods high in cholesterol Signs and symptoms of digoxin toxicity Importance of monitoring output

Rationale The risk of digoxin toxicity increases when the client is receiving digoxin and furosemide, a loop diuretic; loop diuretics can cause hypokalemia, which potentiates the effects of digoxin, leading to toxicity. Digoxin toxicity can result in dysrhythmias and death. When a client is receiving a loop diuretic, the diet should be high in potassium. Although teaching the need to avoid foods high in cholesterol may be included in the teaching plan, it is not the priority. It is not necessary to monitor output. STUDY TIP: The old standbys of enough sleep and adequate nutritional intake also help keep excessive stress at bay. Although nursing students learn about the body's energy needs in anatomy and physiology classes, somehow they tend to forget that glucose is necessary for brain cells to work. Skipping breakfast or lunch or surviving on junk food puts the brain at a disadvantage.

724) To prevent excessive bruising when administering subcutaneous heparin, which technique will the nurse employ? Administer the injection via the Z-track technique. Avoid massaging the injection site after the injection. Use 2 mL of sterile normal saline to dilute the heparin. Inject the medication into the vastus lateralis muscle in the thigh.

Rationale The site of the injection should not be massaged to avoid dispersion of the heparin around the site and subsequent bleeding into the area. The Z-track technique and the intramuscular route are not used with heparin; subcutaneous injection and intravenous administration are the routes appropriate for heparin administration. The medication should be injected into the subcutaneous tissue slowly, not quickly. Diluting heparin with normal saline is unnecessary. Generally, heparin is provided by the pharmacy department in single-dose syringes.

871) When a client with type 1 diabetes develops heart failure, digoxin is prescribed. Which nursing action is important to include when planning care? Administer the digoxin 1 hour after the client's morning insulin. Monitor the client for cardiac dysrhythmias. Monitor for increased risk of hyperglycemia. Increase digoxin dosage if insulin requirements are increased.

Rationale The speed of conduction is decreased when digoxin is given, and this can result in a variety of cardiac dysrhythmias. The risk for hyperglycemia is not increased. Administration times for insulin and digoxin do not have to be coordinated. Dosage of digoxin is not dependent on insulin dosage. STUDY TIP: Determine whether you are a 'lark' or an 'owl.' Larks, day people, do best getting up early and studying during daylight hours. Owls, night people, are more alert after dark and can remain up late at night studying, catching up on needed sleep during daylight hours. It is better to work with natural biorhythms than to try to conform to an arbitrary schedule. You will absorb material more quickly and retain it better if you use your most alert periods of each day for study. Of course, it is necessary to work around class and clinical schedules. Owls should attempt to register in afternoon or evening lectures and clinical sections; larks do better with morning lectures and day clinical sections.

878) A client is discharged with a prescription for sustained-release nitroglycerin. Which information will the nurse provide to the client? Swallow the capsule whole. Take the medication with milk. Place the capsule under the tongue. Crush the capsule and mix with soft food.

Rationale The sustained-release capsule should be swallowed whole on an empty stomach. The capsule should not be chewed or crushed because the 'beads' within the capsule are activated on a time-release schedule. Taking the capsule with milk isn't necessary; a full glass of water is sufficient. The sustained-release capsule is taken on an empty stomach. A sublingual tablet is held under the tongue, not swallowed; sustained-release nitroglycerin is a capsule that needs to be swallowed. A stinging feeling when the medication is under the tongue may occur with a sublingual nitroglycerin tablet; sustained-release nitroglycerin is a capsule that should be swallowed whole. Test-Taking Tip: Do not select answers that contain exceptions to the general rule, controversial material, or degrading responses.

582) A breast-feeding mother asks the nurse if the use of herbal medicines will increase breast milk supply. Which nursing response is most appropriate? 'It may be safe if taken with lots of water.' 'It does not effectively increase breast milk supply.' 'It may cause iron deficiency anemia in the infant.' 'You should speak to your health care provider about this.'

Rationale The use of herbs may increase breast milk supply, but research is limited, so the mother should consult with her health care provider. The herbs are safe for the mother with or without water. However, the priority in this case is to inform the parent of the adverse effects that can result in the infant. Early introduction of solids may increase the risk for iron deficiency anemia in the infant. The herbs increase breast milk supply.

720) Which clinical indicator would the nurse monitor to determine if the client's simvastatin is effective? Heart rate Triglycerides Blood pressure International normalized ratio (INR)

Rationale Therapeutic effects of simvastatin include decreased levels of serum triglycerides, low-density lipoprotein (LDL), and cholesterol. INR is not related to simvastatin; it is a measure used to evaluate blood coagulation. Heart rate and blood pressure are not related to simvastatin.

Question 9 A client who had surgery is discharged on warfarin. Which statement by the client is incorrect and indicates a need for further teaching? Question 9 Answer Choices A"I will report any bruises or unusual bleeding." "I will report any bruises or unusual bleeding." B"I know I must avoid crowds." "I know I must avoid crowds." C"I plan on using an electric razor for shaving." "I plan on using an electric razor for shaving." D"I will keep all laboratory appointments." "I will keep all laboratory appointments." Question Explanation

Rationale There are no specific reasons for the client on warfarin to avoid crowds. Clients should not use a straight edge razor, should report any unusual bleeding and must keep all laboratory appointments when taking the blood thinner warfarin.

Question 5 The nurse is providing discharge education to a client newly diagnosed with chronic obstructive pulmonary disease. The client is prescribed the diskus inhaler fluticasone propionate and salmeterol. The client asks, "How will I know when the inhaler is empty?" How should the nurse respond? Question 5 Answer Choices AShake the canister to detect any fluid movement Shake the canister to detect any fluid movement BThe number of doses that remain will be on the inhaler The number of doses that remain will be on the inhaler CDrop the canister in water to observe floating Drop the canister in water to observe floating DEstimate how many doses are usually in the canister Estimate how many doses are usually in the canister Question Explanation

Rationale There are several methods to monitoring the contents of an inhaler. New MDIs such as diskus inhalers often have counters on them. The counters record the number of doses left in the canister. If the MDI does not have this feature, the client should write the date a refill is needed. This can be done directly on the canister in a permanent marker. Manufacturers do not recommend floating inhalers. The shaking or estimation method will not be accurate.

1204) The nurse recognizes that hormonal therapy (HT) increases the risk of which condition in postmenopausal women? Breast cancer Rapid weight loss Accelerated bone loss Vaginal tissue atrophy

Rationale There is a relationship between HT that combines estrogen and progesterone compounds and an increased incidence of invasive breast cancer. One side effect of HT is weight gain with ankle and foot edema. Bone loss is slowed with HT. Vaginal tissue maintains turgor and lubrication with HT.

1061) Which physiological function is the last function lost during anesthesia induction? Gag reflex Eyelid reflexes Voluntary control Respiratory movement

Rationale There is no respiratory movement in stage 4 of anesthesia; before this stage, respirations are depressed but present. The gag reflex is lost in stage 3 of anesthesia. Eyelid reflexes are lost in stage 2 of anesthesia. Voluntary control is lost in stage 2 of anesthesia.

649) The client with hypokalemia reports nausea, vomiting, and seeing a yellow light around objects. Which of the client's medications is the likely cause of the client's symptoms? Digoxin Furosemide Propranolol Spironolactone

Rationale These are signs of digitalis toxicity, which is more likely to occur in the presence of hypokalemia. Although furosemide most likely contributed to the hypokalemia, the client's symptoms are consistent with digitalis toxicity. Although propranolol can cause nausea, vomiting, and blurred vision, the presence of hypokalemia and yellow vision are more suggestive of digitalis toxicity. A side effect of spironolactone is hyperkalemia, not hypokalemia.

751) While receiving an adrenergic beta 2 agonist medication for asthma, the client complains of palpitations, chest pain, and a throbbing headache. Which nursing action is the most appropriate? Withhold the medication and notify the health care provider. Tell the client that these are expected side effects from the medicine. Give instructions to breathe slowly and deeply for several minutes. Explain that the effects are temporary and will subside as medication tolerance develops.

Rationale These medications cause increased heart contraction (positive inotropic effect) and increased heart rate (positive chronotropic effect). If toxic levels are reached and side effects occur, the medication should be withheld until the health care provider is notified. Telling the client that these are expected side effects from the medicine is false reassurance and a false statement; they should not cause chest pain. Controlled breathing may be helpful in allaying a client's anxiety; however, the medication may be producing adverse effects and should be withheld. Test-Taking Tip: Come to your test prep with a positive attitude about yourself, your nursing knowledge, and your test-taking abilities. A positive attitude is achieved through self-confidence gained by effective study. This means (a) answering questions (assessment), (b) organizing study time (planning), (c) reading and further study (implementation), and (d) answering questions (evaluation).

154) Which statement by a client who had a laminectomy and is receiving a skeletal muscle relaxant that will be continued after discharge indicates that teaching was effective? "I'm going to take the medication between meals." "If the medication makes me sleepy, I'll stop taking it." "If the medication upsets my stomach, I'll take it with milk." "I'll take an extra dose of the medication before I do anything active."

Rationale These medications tend to irritate the gastric mucosa and should be taken with milk or food to limit gastrointestinal irritation. Drowsiness is an expected side effect; safety precautions are indicated, but the medication should not be discontinued. Taking an extra dose of the medication before activity can result in toxicity if the extra dose, in addition to the prescribed dose, exceeds the therapeutic range; the dosage prescribed by the health care provider should be followed.

1496) Which instruction would the nurse provide to parents of a school-age child who has been on long-term phenytoin therapy to prevent side effects? Provide good oral hygiene. Administer the medication between meals. Watch for a reddish-brown discoloration of urine. Supplement the diet with high-calorie foods.

Rationale These procedures reduce the risk for gingival hyperplasia, a side effect of phenytoin. This medication is strongly alkaline and should be administered with meals to help prevent gastric irritation. Discoloration of the urine may occur during medication excretion; it does not cause physiological problems. Avoiding overeating and overhydration may result in better seizure control.

135) Which body function maintained by thiamine (vitamin B 1) and niacin (vitamin B 3) will the nurse monitor when prescribed for a client with alcoholism? Neuronal activity Bowel elimination Efficient circulation Prothrombin development

Rationale Thiamine and niacin help convert glucose for energy and influence nerve activity. These vitamins do not affect elimination. These vitamins are not related to circulatory activity. Vitamin K, not thiamine and niacin, is essential for the manufacturing of prothrombin.

232) Which medication therapy is indicated for management of Wernicke encephalopathy associated with Korsakoff syndrome? Traditional phenothiazines Judicious use of antipsychotics Intramuscular injections of thiamine Oral administration of chlorpromazine

Rationale Thiamine is a coenzyme necessary for the production of energy from glucose. If thiamine is not present in adequate amounts, nerve activity is diminished and damage or degeneration of myelin sheaths occurs. A traditional phenothiazine is a neuroleptic antipsychotic that should not be prescribed because it is hepatotoxic. Antipsychotics must be avoided; their use has a higher risk of toxic side effects in older or debilitated persons. Chlorpromazine, a neuroleptic, cannot be used because it is severely toxic to the liver.

918) Sodium nitroprusside is prescribed for a client with a blood pressure of 260/120 mm Hg. The nurse recalls that sodium nitroprusside decreases blood pressure by which mechanism? Decreasing the heart rate Increasing cardiac output Increasing peripheral resistance Relaxing venous and arterial smooth muscles

Rationale This medication decreases blood pressure by relaxing venous and arteriolar smooth muscles and is used for immediate reduction of blood pressure. This medication may increase the heart rate as a response to vasodilation. It decreases cardiac workload by decreasing preload and afterload. It decreases peripheral resistance by dilating peripheral blood vessels.

899) The nurse provides instruction when the beta-blocker (BB) atenolol is prescribed for a client with moderate hypertension. Which client statement indicates to the nurse that further teaching is needed? 'I must take the medication before going to bed.' 'This medication will make me feel drowsy.' 'I need to count my pulse before taking the medication.' 'I will move slowly when changing positions from sitting to standing.'

Rationale This medication should be taken early in the morning to maximize its therapeutic effect. Orthostatic hypotension is a side effect of BBs, and the client should change positions slowly from sitting to standing to prevent dizziness and falls. Drowsiness is a side effect of BBs, and the client should be taught precautions to prevent injury. The pulse rate should be taken before administration because ventricular dysrhythmias and heart block may occur with BBs.

669) The nurse administers sodium polystyrene sulfonate to a client with chronic renal failure. Which finding provides evidence that the intervention is effective? Pruritus decreases. Mental status improves. Sodium decreases to 137 mEq/L (137 mmol/L). Potassium decreases to 4.2 mEq/L (4.2 mmol/L).

Rationale This resin exchanges sodium ions for potassium in the large intestine to lower the serum potassium level; 4.2 mEq/L (4.2 mmol/L) is in the expected range for potassium. Mental status improvement and relief of pruritus are not therapeutic effects of the medication. Sodium retention is an adverse effect; 137 mEq/L (137 mmol/L) is in the expected range for sodium.

714) A client has primary open-angle glaucoma. Which ophthalmic preparation is indicated to manage this condition? Tetracaine Fluorescein Timolol maleate Atropine sulfate

Rationale Timolol maleate is a beta-adrenergic antagonist that decreases aqueous humor production and increases outflow, thereby reducing intraocular pressure. Tetracaine is a topical anesthetic; it will not reduce the increased intraocular pressure associated with glaucoma. Fluorescein is a dye used to identify corneal abrasions and foreign bodies. Atropine sulfate, a mydriatic, is contraindicated because it dilates the pupil, obstructing drainage and increasing intraocular pressure.

Question 18 The nurse is discharging a client with a new prescription for tiotropium to help manage the symptoms of chronic obstructive pulmonary disease. What information should the nurse include in the discharge teaching? Question 18 Answer Choices AIt may be a few days before you feel the full effects of tiotropium. It may be a few days before you feel the full effects of tiotropium. BThis medication cannot be used to relieve sudden breathing problems. This medication cannot be used to relieve sudden breathing problems. CBe sure to swallow the capsules with a full glass of water. Be sure to swallow the capsules with a full glass of water. DA common side effect is nausea and loose stools. A common side effect is nausea and loose stools. Question Explanation

Rationale Tiotropium is a long-acting anticholinergic bronchodilator. The medication comes as a capsule to use with a specially designed inhaler - clients should never swallow the capsules. For new prescriptions, it's important to tell the client that they may start breathing better with the full dose but it may take a few weeks to feel the full effects. It cannot be used as a fast-acting inhaler. Due to its anticholinergic properties, it may cause constipation (not loose stools).

736) A health care provider prescribes tolterodine for a client with an overactive bladder. Which action is important to include in client teaching? Maintain a strict record of fluid intake and urinary output. Chew the extended-release capsule thoroughly before swallowing. Report episodes of diarrhea or any increase in respiratory secretions. Avoid activities requiring alertness until the response to medication is known.

Rationale Tolterodine, a urinary tract antispasmodic, may cause dizziness and blurred vision, placing the client at risk for injury. Although it is important to know if the client is experiencing anuria or overflow incontinence, which may indicate urinary retention, a detailed intake and output record is unnecessary. An extended-release capsule should be swallowed whole and should not be opened or chewed. If chewed or opened, the client will receive a surge of action and the long-term action of the medication is gone. Tolterodine is classified as an anticholinergic, and adverse reactions include constipation and dry mouth; diarrhea and an increase in respiratory secretions are associated with medications classified as cholinergics.

648) The nurse administers a parenteral preparation of potassium slowly to avoid which complication? Metabolic acidosis Cardiac arrest Seizure activity Respiratory depression

Rationale Too rapid an administration can cause hyperkalemia, which contributes to a long refractory period in the cardiac cycle, resulting in cardiac dysrhythmias and arrest. Although acidosis can cause hyperkalemia, hyperkalemia will not lead to acidosis. Hyperkalemia causes muscle flaccidity and weakness, not seizures. Respiratory depression can occur with too rapid intravenous (IV) magnesium administration, not potassium administration.

815) One week after being hospitalized for an acute myocardial infarction, a client reports nausea and loss of appetite. Which of the client's prescribed medications would be withheld and the health care provider notified? Digoxin Propranolol Furosemide Spironolactone

Rationale Toxic levels of digoxin stimulate the medullary chemoreceptor trigger zone, resulting in anorexia, nausea, and vomiting. Although anorexia, nausea, and vomiting may be side effects of furosemide, propranolol, and spironolactone, they do not indicate toxicity.

Question 6 A hospitalized 8-month-old infant is receiving digoxin to treat Tetralogy of Fallot. Prior to administering the next dose of the medication, the parent reports that the baby vomited one time, just after breakfast. The infant's heart rate is 92 bpm. What action should the nurse take? Question 6 Answer Choices AGive the scheduled dose after the client is done eating lunch. Give the scheduled dose after the client is done eating lunch. BHold the medication and notify the primary health care provider. Hold the medication and notify the primary health care provider. CReduce the next dose by half and then resume the normal medication schedule. Reduce the next dose by half and then resume the normal medication schedule. DDouble the next dose to make up for the medication lost from vomiting. Double the next dose to make up for the medication lost from vomiting. Question Explanation

Rationale Toxic side effects of digoxin include bradycardia, dysrhythmia, nausea, vomiting, anorexia, dizziness, headache, weakness and fatigue. It isn't typically necessary to hold the medication for infants and children if there is only one episode of vomiting. However, it is appropriate to hold the medication and notify the primary health care provider (HCP) of the vomiting episode and the lower than normal heart rate. A digoxin level may need to be drawn. The normal resting heart rate for infants 1 to 11 months old is 100 to 160 bpm.

975) Which advice will the nurse include when teaching a client about digoxin for left ventricular failure? Sleep flat in bed. Follow a low-potassium diet. Take the pulse three times a day. Report increasing fatigue.

Rationale Treatment with digoxin should improve fatigue associated with heart failure; if fatigue increases, it may reflect complications of therapy. Sleeping with the head slightly elevated facilitates respiration. The client needs potassium. A low-potassium diet when the client is taking digoxin predisposes the client to toxicity and dangerous dysrhythmias. To avoid becoming obsessed with the pulse rate, the client should take the pulse less often; once daily is adequate.

838) Which assessment finding would the nurse identify as an adverse effect of a client's lidocaine infusion? Tremors Tachypnea Tachycardia Hypertension

Rationale Tremors are a precursor to the major adverse effect of seizures. Respiratory changes may occur, but tachypnea is not typical. Bradycardia, which may lead to heart block, may occur, not tachycardia. Hypotension, not hypertension, may occur. Test-Taking Tip: Being emotionally prepared for an examination is key to your success. Proper use of resources over an extended period of time ensures your understanding and increases your confidence about your nursing knowledge. Your lifelong dream of becoming a nurse is now within your reach! You are excited, yet anxious. This feeling is normal. A little anxiety can be good because it increases awareness of reality; but excessive anxiety has the opposite effect, acting as a barrier and keeping you from reaching your goal. Your attitude about yourself and your goals will help keep you focused, adding to your strength and inner conviction to achieve success.

Question 1 A nurse is teaching an 80-year-old client how to use a metered dose inhaler. The nurse is concerned that the client is unable to coordinate the release of the medication during the inhalation phase. Which intervention should improve the delivery of the medication? Question 1 Answer Choices AAsk a family member to assist the client with the inhaler. Ask a family member to assist the client with the inhaler. BRequest a home health nurse to visit the client at home. Request a home health nurse to visit the client at home. CUse nebulized treatments at home instead. Use nebulized treatments at home instead. DAdd a spacer device to the inhaler canister. Add a spacer device to the inhaler canister. Question Explanation

Rationale Use of a spacer is especially useful with older adults because it allows more time to inhale and requires less eye-hand coordination. If the client is not using the metered dose inhaler (MDI) properly, the medication can get trapped in the upper airway and lead to dry mouth and throat irritation. Using a spacer will allow more drug to be deposited in the lungs and less in the mouth.

1549) Which finding in a menopausal client's health history would prevent the health care provider from prescribing hormone replacement therapy? Select all that apply. One, some, or all responses may be correct. Smoking Cirrhosis Cholecystitis Breast cancer Deep vein thrombosis

Rationale Use of estrogens can have major side effects, especially if the client smokes. The nurse would provide information to the client about smoking cessation. Clients with cirrhosis have a decreased ability to break down medications, especially estrogen. Cholecystitis can worsen in clients taking estrogen. Clients at risk for breast and endometrial cancer should not take estrogen because it can further increase the risk. Estrogens can lead to deep vein thrombosis.

959) Immediately after a bilateral adrenalectomy, a client is receiving corticosteroids that are to be continued after discharge from the hospital. Which statement by the client indicates to the nurse that additional education is needed? 'I need to have periodic tests of my blood for glucose.' 'I am glad that I only have to take the medication once a day.' 'I must take the medicine with meals.' 'I should tell my health care provider if I am overly restless or have trouble sleeping.'

Rationale Usually a larger dose is given at 8:00 AM and the second dose is given before 4:00 PM to mimic expected hormonal secretion and prevent insomnia. Having periodic blood tests for glucose is necessary because long-term administration of steroids leads to elevated blood glucose levels and possible steroid-induced diabetes. Oral corticosteroids should be taken with food or antacids to prevent gastric irritation and gastric hemorrhage. Neurological and emotional side effects, such as euphoria, mood swings, and sleeplessness, are expected.

1527) The nurse understands which antiepileptic medication would be used as the first-line treatment for absence seizures? Phenytoin Diazepam Valproic acid Acetazolamide

Rationale Valproic acid is used as the first-line treatment for absence seizures. Phenytoin is used to treat partial, secondary, and generalized tonic-clonic seizures. Diazepam is used to treat status epilepticus. Acetazolamide is used as an adjunct medication for the treatment of absence seizures.

816) Which client response indicates to the nurse that a vasodilator medication is effective? Absence of adventitious breath sounds Increase in the daily amount of urine produced Pulse rate decreases from 110 to 75 beats/minute Blood pressure changes from 154/90 to 126/72 mm Hg

Rationale Vasodilation will lower the blood pressure. The pulse rate is not decreased and may increase. Breath sounds are not directly affected by vasodilation, although vasodilator medications can decrease preload and afterload, which could indirectly affect breath sounds in heart failure. The urine output is not affected immediately, although control of blood pressure can help preserve renal function over time.

249) Which medication is indicated for emergency treatment of bleeding esophageal varices? Vasopressin Neostigmine Lansoprazole Phytonadione

Rationale Vasopressin is a vasoconstrictor that can be used to control gastrointestinal bleeding. Neostigmine inhibits cholinesterase, permitting acetylcholine to function; it is used primarily for myasthenia gravis. Lansoprazole is a proton pump inhibitor that is used for the treatment of gastric and duodenal ulcers. Phytonadione is vitamin K; it promotes formation of prothrombin in the liver. Although this medication may be helpful, its effects take too long to be of value in an emergency situation. STUDY TIP: Regular exercise, even if only a 10-minute brisk walk each day, aids in reducing stress. Although you may have been able to enjoy regular sessions at the health club or at an exercise class several times a week, you now may have to cut down on that time without giving up a set schedule for an exercise routine. Using an exercise bicycle that has a book rack on it at home, the YMCA, or a health club can help you accomplish two goals at once. You can exercise while beginning a reading assignment or while studying notes for an exam. Listening to lecture recordings while doing floor exercises is another option. At least a couple of times a week, however, the exercise routine should be done without the mental connection to school; time for the mind to unwind is necessary, too.

Question 7 The nurse is teaching a client about precautions while taking warfarin. The nurse should instruct the client to avoid foods with excessive amounts of which nutrient? Question 7 Answer Choices ACalcium Calcium BVitamin E Vitamin E CIron Iron DVitamin K Vitamin K Question Explanation

Rationale Vitamin K is an essential vitamin required for blood clotting. Eating foods with excessive amounts of Vitamin K may alter anticoagulant effects. Foods highest in vitamin K include (dried and fresh) herbs, dark leafy greens, scallions, Brussel sprouts, broccoli, chili powder, prunes, asparagus and cabbage.

1126) The surgeon prescribes vitamin K before surgery. The nurse recognizes that this is prescribed because vitamin K contributes to the formation of which substance? Bilirubin Prothrombin Thromboplastin Cholecystokinin

Rationale Vitamin K is necessary in the formation of prothrombin to prevent bleeding. It is a fat-soluble vitamin and is not absorbed from the gastrointestinal (GI) tract in the absence of bile. Bilirubin is the bile pigment formed by the breakdown of erythrocytes. Thromboplastin converts prothrombin into thrombin during the process of coagulation. Cholecystokinin is the hormone that stimulates contraction of the gallbladder. STUDY TIP: Avoid planning other activities that will add stress to your life between now and the time you take the licensure examination. Enough will happen spontaneously; do not plan to add to it.

446) Which substance does vitamin K contributes to the formation of? Bilirubin Prothrombin Thromboplastin Cholecystokinin

Rationale Vitamin K is necessary in the formation of prothrombin to prevent bleeding. It is a fat-soluble vitamin and is not absorbed from the gastrointestinal (GI) tract in the absence of bile. Bilirubin is the bile pigment formed by the breakdown of erythrocytes. Thromboplastin converts prothrombin into thrombin during the process of coagulation. Cholecystokinin is the hormone that stimulates contraction of the gallbladder. STUDY TIP: Avoid planning other activities that will add stress to your life between now and the time you take the licensure examination. Enough will happen spontaneously; do not plan to add to it.

420) Which prescription would the nurse anticipate for the client who takes a medication that interferes with fat absorption? High-fat diet Supplemental cod liver oil Total parenteral nutrition (TPN) Water-miscible forms of vitamins A and E

Rationale Vitamins A, D, E, and K are known as fat-soluble vitamins because bile salts and other fat-related compounds aid their absorption. A high-fat diet will not achieve the uptake of fat-soluble vitamins in this client. Supplemental cod liver oil will not achieve the uptake of fat-soluble vitamins in this client. TPN is unnecessary; a well-balanced diet is preferred. Water-miscible forms of vitamins A and E can be absorbed with water-soluble nutrients. Test-Taking Tip: Study wisely, not hard. Use study strategies to save time and be able to get a good night's sleep the night before your exam. Cramming is not smart, and it is hard work that increases stress while reducing learning. When you cram, your mind is more likely to go blank during a test. When you cram, the information is in your short-term memory, so you will need to relearn it before a comprehensive exam. Relearning takes more time. The stress caused by cramming may interfere with your sleep. Your brain needs sleep to function at its best.

564) An infant with congenital heart disease is prescribed digoxin and furosemide upon discharge. Which sign would the nurse instruct the parents to be alert for? Difficulty feeding with vomiting Cyanosis during periods of crying Daily naps lasting more than 3 hours A pulse rate faster than 100 beats/min

Rationale Vomiting and feeding issues are early signs of digoxin toxicity. Cyanosis is expected in a crying infant with heart disease because the energy expenditure exceeds the body's ability to meet the oxygen demand. Long naps are expected; infants routinely require several naps, and an infant with heart disease requires long rest periods. The pulse rate of an infant receiving digoxin should remain faster than 100 beats/min.

1486) The nurse is reviewing medication instructions with parents of an infant receiving digoxin and spironolactone. Which parental response indicates instructions have been understood? Activity should be restricted. Orange juice should be given daily. Vomiting should be reported to the health care provider. Anti-inflammatory medications should be avoided.

Rationale Vomiting is a classic sign of digoxin toxicity, and the health care provider must be notified. Infants regulate their own activity according to their energy level. Orange juice is rarely needed because spironolactone spares potassium. There is no restriction on anti-inflammatory medications with spironolactone.

617) The nurse is monitoring a 6-year-old child for toxicity precipitated by digoxin. Which sign of digoxin toxicity would the nurse monitor for? Oliguria Vomiting Tachypnea Splenomegaly

Rationale Vomiting is a sign of digoxin toxicity in children. Oliguria is associated with renal failure, not toxicity. Tachypnea is associated with heart failure, not toxicity. Splenomegaly is associated with heart failure, specifically right ventricular failure. Test-Taking Tip: Monitor questions that you answer with an educated guess or for which you changed your answer from the first option you selected. This will help you analyze your ability to think critically. Usually your first answer is and should not be changed without reason.

1035) A client with a partial occlusion of the left common carotid artery is to be discharged while still receiving warfarin. Which adverse effect will the nurse identify as a reason for the client to seek medical consultation? Select all that apply. One, some, or all responses may be correct. Hematuria Hemoptysis Delayed clotting from minor cuts and scrapes Bleeding from gums when brushing teeth Vomiting coffee-ground emesis

Rationale Warfarin causes an increase in the prothrombin time and International Normalized Ratio (INR) level, leading to an increased risk for bleeding. Any abnormal or prolonged bleeding must be reported, because it may indicate an excessive level of the medication. Common side effects including bruising, delayed clotting and bleeding gums do not require immediate intervention. However, hematuria and hemoptysis are evidence of more serious bleeding and require immediate attention. Coffee-ground emesis is a sign of gastric bleeding.

679) The laboratory report establishes that the client has a warfarin overdose. Which antidote would the nurse anticipate administering? Physostigmine Vitamin K Iron dextran Protamine sulfate

Rationale Warfarin inhibits formation of vitamin K-dependent clotting factors. Its effect is overcome by increasing vitamin K. Iron dextran is an iron supplement, not an antidote for warfarin. Protamine sulfate is the antidote for heparin overdose. Physostigmine is an antidote for anticholinergic overdose. Test-Taking Tip: Do not worry if you select the same numbered answer repeatedly, because there usually is no pattern to the answers.

97) Which antidote would the nurse anticipate administering to a client whose laboratory report establishes a warfarin overdose? Physostigmine Vitamin K Iron dextran Protamine sulfate

Rationale Warfarin inhibits formation of vitamin K-dependent clotting factors. Its effect is overcome by increasing vitamin K. Physostigmine is an antidote for anticholinergic overdose. Iron dextran is an iron supplement, not an antidote for warfarin. Protamine sulfate is the antidote for heparin overdose. Test-Taking Tip: Do not worry if you select the same numbered answer repeatedly, because there usually is no pattern to the answers.

806) Warfarin is prescribed for the client who takes phenytoin for a seizure disorder. Which medication interaction complicates seizure therapy? Warfarin inhibits the metabolism of phenytoin. Warfarin decreases phenytoin absorption. Phenytoin competes with warfarin for receptor occupation. Warfarin promotes excretion of phenytoin.

Rationale Warfarin inhibits metabolism of phenytoin, which can result in phenytoin toxicity. Warfarin does not decrease phenytoin absorption. Phenytoin and warfarin act on different receptors. Excretion of phenytoin is not increased. Test-Taking Tip: Look for options that are similar in nature. If all are , either the question is poor, or all options are in, the latter of which is more likely. Example: If the answer you are seeking is directed to a specific treatment and all but one option deal with signs and symptoms, you would be in choosing the treatment-specific option.

Question 6 The nurse has provided instructions to a client on the use of warfarin. Which statement by the client requires further teaching? Question 6 Answer Choices A"If I become constipated, I can take laxatives containing magnesium salts." "If I become constipated, I can take laxatives containing magnesium salts." B"If I develop a headache, I should take ibuprofen to help my pain." "If I develop a headache, I should take ibuprofen to help my pain." C"If I develop an itchy rash, I will use a cream with diphenhydramine." "If I develop an itchy rash, I will use a cream with diphenhydramine." D"If I catch a cold, I will use guaifenesin to make my cough better" "If I catch a cold, I will use guaifenesin to make my cough better" Question Explanation

Rationale Warfarin is an anticoagulant that prolongs bleeding time and is used to treat and prevent blood clots. One of the most serious side effects of warfarin is excessive bleeding and hemorrhage. Warfarin interacts with a number of other drugs. Clients taking warfarin should not take nonsteroidal anti-inflammatory drugs (NSAIDs) such as ibuprofen at the same time due to an increased risk for bleeding. There are no known drug interactions between warfarin and laxatives containing magnesium salts, guaifenesin, or diphenhydramine cream. As a result, they may be taken together.

Question 3 The nurse is reinforcing medication interactions with a client who is taking warfarin. Which over-the-counter (OTC) medication should the nurse remind the client to avoid? Question 3 Answer Choices ADiphenhydramine Diphenhydramine BAcetaminophen Acetaminophen CNaproxen Naproxen DPantoprazole Pantoprazole Question Explanation

Rationale Warfarin is an anticoagulant. OTC medications that interact with warfarin should be avoided. Naproxen, a nonsteroidal anti-inflammatory drug (NSAID), is a commonly used OTC analgesic. Naproxen can prolong bleeding time and should therefore be avoided by clients who take anticoagulants. The other medications are not contraindicated when taking warfarin.

1083) A client is taking warfarin. If an antidote is needed, which agent will be used? Vitamin K Fibrinogen Prothrombin Protamine sulfate

Rationale Warfarin sodium inhibits vitamin K; therefore vitamin K is the antidote for warfarin sodium. Fibrinogen and prothrombin are blood-clotting factors, not the antidotes for warfarin sodium. Protamine sulfate is the antidote for heparin, not warfarin sodium.

Question 12 A nurse is educating a client about the use of warfarin at home. The nurse should reinforce the need for the client to monitor which of the following? Question 12 Answer Choices AExtended exposure to outdoor sunlight Extended exposure to outdoor sunlight BConsistent intake of foods high in vitamin K Consistent intake of foods high in vitamin K CAvoidance of public transportation and large groups of people Avoidance of public transportation and large groups of people DLimit of strenuous physical exercise Limit of strenuous physical exercise Question Explanation

Rationale Warfarin, an oral anticoagulant, works by causing a decrease in the vitamin K-dependent clotting factors produced by the liver. Due to this mechanism of action, vitamin K is used as the antidote for warfarin overdose. A diet high in vitamin K could counteract the therapeutic effect of warfarin. Foods high in vitamin K include dark green leafy vegetables, tomatoes, bananas, cheese and fish. Best practice no longer recommends limiting the intake of Vitamin K-containing foods, instead it is recommended to keep the intake of foods high in Vitamin K 'consistent'. The other actions do not pertain to warfarin.

296) Which side effect of prolonged cortisone therapy for adrenal insufficiency would the nurse teach the client and family to expect? Select all that apply. One, some, or all responses may be correct. Oliguria Anorexia Weakness Moon face Weight gain Nervousness

Rationale Weakness occurs because of muscle wasting as a result of the catabolic effects of cortisol. Hypokalemia may also cause weakness; potassium is lost in the urine as sodium is retained. Accumulation of adipose tissue occurs in the face (moon face), trunk (truncal obesity), and cervical area (buffalo hump). Weight gain occurs because of increased appetite and fluid retention; 1 liter of fluid is equal to 2.2 pounds (1 kilogram). Cortisone increases sodium and water retention but does not cause oliguria; glucose levels also increase, which, if extreme, will cause polyuria. The appetite usually increases, not decreases. Cortisone increases blood glucose levels, which, if extreme, will cause lethargy, not nervousness. Test-Taking Tip: Come to your test prep with a positive attitude about yourself, your nursing knowledge, and your test-taking abilities. A positive attitude is achieved through self-confidence gained through effective study. This means (a) answering questions (assessment), (b) organizing study time (planning), (c) reading and further study (implementation), and (d) answering questions (evaluation).

501) When the nurse is administering intravenous potassium to a client with hypokalemia, which finding is most important to communicate to the health care provider? U waves on cardiac monitor QRS duration of 0.28 seconds Decreased bowel sounds Weakened grip strength

Rationale When administering intravenous potassium supplements, it is important to evaluate for clinical manifestations of hyperkalemia. Widening of the Q waves is a potentially fatal manifestation of hyperkalemia (because it may lead to cardiac arrest) and would be communicated rapidly to the health care provider so that the infusion can be stopped and the potassium level can be rechecked. The other findings would be reported to the health care provider but are expected with hypokalemia and are not an indication for a change in treatment. U waves are an expected manifestation of hypokalemia because of changes in ventricular repolarization. Decreased bowel sounds may occur because of decreased peristalsis caused by low potassium levels but should improve with potassium administration. Weakened grips may occur with hypokalemia because normal extracellular potassium levels are needed for skeletal muscle contraction.

481) Which reason would an intravenous infusion of 5% dextrose with 0.45% sodium chloride and 20 mEq of potassium be prescribed for a client with a nasogastric (NG) tube set to low intermittent suction? Prevent constipation Prevent dehydration Prevent vomiting Prevent electrolyte imbalance

Rationale When clients do not receive nutrients or fluids by mouth and have a loss of electrolytes through the removal of gastric secretions via an NG tube, electrolyte imbalance is a primary concern. Constipation is usually not a concern in this situation. Although dehydration is a possible effect of an NG tube that removes gastric secretions and fluid, electrolyte balance is still the priority. An NG tube set to low intermittent suction usually relieves nausea and vomiting. Test-Taking Tip: Become familiar with reading questions on a computer screen. Familiarity reduces anxiety and decreases errors.

1547) Which statement by the nurse reflects teaching for a client recently initiated on anticonvulsants? Select all that apply. One, some, or all responses may be . 'It is important to take the medication at the same time every day with meals.' 'It is important to not drink excessive amounts of caffeine-containing beverages or alcohol.' 'Avoid driving or hazardous activities until any side effects such as drowsiness can be determined.' 'Some anticonvulsants interfere with vitamin and mineral absorption, so you may need a supplement.' 'Oral hygiene, such as gum massage and tooth brushing, is important to combat the gingival hyperplasia that some anticonvulsant medication can cause.'

Rationale When initiating anticonvulsant medication, it is important to maintain consistent levels of the medication, so the patient would take it at the same time every day and with meals for absorption. It is important not to drive or perform any hazardous activities as new medications can cause drowsiness. There is some evidence that caffeinated and alcoholic beverages lower seizure threshold. Adding a multivitamin supplement is recommended, especially if the anticonvulsant is the kind that interferes with absorption. Frequent dental examinations and good oral hygiene are the first line of defense against the gingival hyperplasia caused by some anticonvulsants. Test-Taking Tip: Be alert for details about what you are being asked to do. In this question type, you are asked to select all options that apply to a given situation or client. All options likely relate to the situation, but only some of the options may relate directly to the situation.

371) Which medications would the nurse identify as commonly used as an adjunct during conscious sedation for minor surgeries? Diazepam Midazolam Lorazepam Clonazepam

Rationale When used in conjunction with an opioid analgesic, midazolam causes conscious sedation, which is a semiconscious state suitable for minor surgeries and endoscopic procedures. Diazepam and lorazepam are used for anesthetic induction, preanesthetic medication, and the production of conscious sedation when used in conjunction with opioid analgesics. Clonazepam is used to treat seizures and anxiety.

748) List in order the steps the nurse teaches the client to follow when using a metered-dose inhaler (MDI). 1. Shake the inhaler for 30 seconds. 2. Exhale slowly and deeply to empty the air from the lungs. 3. Hold the inhaler upright in the mouth. 4. Start breathing in and press down on the inhaler once. Answer: 1,2,3,4,5

Rationale When using an MDI, the medication should be shaken for 30 seconds to ensure that the medication is mixed. Exhaling completely maximizes emptying the lungs. The inhaler should be held upright in the mouth past the teeth with the lips closed around the mouthpiece (closed mouth method) or held upright 1 to 2 cm in front of the open mouth (open mouth method). Inhalation is begun at the same time that the device is compressed to ensure that maximum medication reaches the lungs.

131) Which item will the nurse teach a client who has had an excision of a thrombosed external hemorrhoid to use when cleaning the anus after a bowel movement? Betadine pads Soft facial tissue Witch hazel-moistened pads Sterile 4 × 4-inch (10 × 10 cm) pads

Rationale Witch hazel-moistened pads are not irritating and are soothing to the anal mucosa. Betadine may cause excessive drying and irritation. The rectum always is contaminated; external cleansing with Betadine will not appreciably affect the bacteria present. Dry facial tissue is irritating and can cause trauma. Sterile gauze pads are unnecessary; the rectal area is considered contaminated.

215) Which sign of hypokalemia will the nurse monitor for in a client receiving furosemide? Chvostek sign Muscle weakness Anxious behavior Abdominal cramping

Rationale With hypokalemia, failure occurs in myoneural conduction and smooth muscle functioning, resulting in fatigue and muscle weakness. Chvostek sign, the contraction of the facial muscles in response to a light tap over the facial nerve in front of the ear, is associated with hypocalcemia; low calcium levels allow sodium to move into excitable cells, increasing depolarization and nerve excitability. Anxiety and irritability are associated with hyperkalemia. Hyperkalemia affects the nervous and muscular systems; fatigue, weakness, and lethargy are associated with hypokalemia. Decreased gastrointestinal motility occurs with hypokalemia; abdominal cramping is associated with hyperkalemia and is caused by hyperactivity of smooth muscles.

817) A client is receiving clonidine for hypertension. Which side effect of clonidine will the nurse include when providing medication education? Xerostomia Diarrhea Euphoria Photosensitivity

Rationale Xerostomia (dry mouth) is one of the common side effects of this medication. The reaction usually diminishes over the first 2 to 4 weeks of therapy. This medication causes constipation, not diarrhea. This medication may cause depression, anxiety, fatigue, and drowsiness, not euphoria. Photosensitivity is not a side effect of this medication.

437) Which adverse effect would the nurse continually assess for in a client receiving valproic acid? Yellow sclerae Motor restlessness Ringing in the ears Torsion of the neck

Rationale Yellow sclerae are a sign of jaundice; pancreatitis and hepatic failure are life-threatening adverse effects of valproic acid. The client must have frequent liver function tests. Motor restlessness (akathisia) is associated with antipsychotic medications. Ringing or buzzing in the ears (tinnitus) is associated with aspirin. Torsion of the neck (torticollis) because of contracted cervical muscles is associated with antipsychotic medications. Test-Taking Tip: Pace yourself during the testing period and work as accurately as possible. Do not be pressured into finishing early. Do not rush! Students who achieve higher scores on examinations are typically those who use their time judiciously.

847) A client has been receiving digoxin. The client calls the clinic and complains of 'yellow vision.' Which response would the nurse provide? 'This is related to your illness rather than to your medication.' 'This is an expected side effect; you will become accustomed to it over time.' 'This side effect is only temporary. You should continue the medication.' 'The medication may need to be discontinued. Come to the clinic this afternoon.'

Rationale Yellow vision indicates digoxin toxicity; the medication should be withheld until the health care provider can assess the client and check the digoxin blood level. Yellow vision is related to digoxin therapy, not the client's underlying medical condition. Yellow vision is a sign of digoxin toxicity; taking more digoxin will escalate the digoxin toxicity.

27) A client who has been taking the prescribed dose of zolpidem for 5 days returns to the clinic for a follow-up visit. Which statement by the client indicates the medication has been effective? "I have less pain." "I have been sleeping better." "My blood glucose is under control." "My blood pressure is coming down."

Rationale Zolpidem is a sedative-hypnotic that produces central nervous system depression in the limbic, thalamic, and hypothalamic areas of the brain. Zolpidem is not an analgesic, antidiabetic, or antihypertensive medication.

Question 14 A nurse is preparing to administer prescribed maintenance dose of digoxin to a client who has heart failure. Which action should the nurse to take? Question 14 Answer Choices AWithhold the medication if the heart rate is above 100/min Withhold the medication if the heart rate is above 100/min BInstruct the client to eat foods that are low in potassium Instruct the client to eat foods that are low in potassium CMeasure apical pulse rate for 30 seconds before administration Measure apical pulse rate for 30 seconds before administration DEvaluate the client for nausea, vomiting, and anorexia Evaluate the client for nausea, vomiting, and anorexia Question Explanation

Rationale A client with heart failure who is prescribed digoxin should be assessed for digoxin toxicity. Manifestations of digoxin toxicity include nausea, vomiting, and anorexia. Digoxin is used to decrease heart rate and should be held if the heart rate is less than 60 beats per minute. Digoxin toxicity can occur when the client has low potassium. When administering digoxin, the nurse should measure the client's apical pulse for a full minute.

Question 13 The nurse is caring for a female client who is requesting hormonal contraceptives. Which of the following questions should the nurse ask to assess for contraindications? Question 13 Answer Choices AHave you ever had a blood clot?&rdquo; "Have you ever had a blood clot?" BHow many children do you have?&rdquo; "How many children do you have?" CDo you drink alcohol?&rdquo; "Do you drink alcohol?" DDid you experience acne in adolescence?&rdquo; "Did you experience acne in adolescence?" Question Explanation

Rationale A history of thromboembolic disorders is a contraindication to hormonal contraceptives; therefore, any history of thrombus should be assessed. The number of children/pregnancies and use of alcohol are probable history questions but are not contraindications to this method. Acne is a side effect of oral contraceptives but not a contraindication.

Question 7 The nurse is reviewing discharge instructions with the parent of a 3-year-old client who was admitted for poisoning after ingesting cherry-flavored acetaminophen. Which statement by the parent would require follow up by the nurse? Question 7 Answer Choices A "I should use non-flavored medications." B "I will reach out to the poison control center if this happens again." C "I will use ipecac syrup to induce vomiting." D "I will have all medications in a locked cabinet." Question Explanation

Rationale Accidental ingestions (poisoning) are the most frequent accident in toddlers. Therefore, it is imperative to focus on keeping all poisonous substances, drugs, and small objects securely out of the reach of children and medications in a locked cabinet. Parents should be instructed to call the poison control center in case of accidental ingestion and to have the number listed on their cell phone. Since 2003, the American Academy of Pediatrics has discouraged the use of syrup of ipecac to induce vomiting after accidental ingestion. Instead, families should call the poison control center immediately. Using non-flavored medications will decrease the likelihood the child will consume a large amount; it does not prevent the ingestion of the medication or determine what should be done after accidental ingestion.

Question 7 The nurse is planning care for a pediatric client with a new prescription for adenosine to treat symptomatic supraventricular tachycardia (SVT). Which action should the nurse include in the plan of care? Question 7 Answer Choices AMonitor for ventricular dysrhythmias Monitor for ventricular dysrhythmias BMonitor for shortness of breath Monitor for shortness of breath CMonitor for hypertension Monitor for hypertension DMonitor for nausea. Monitor for nausea. Question Explanation

Rationale After giving adenosine, the nurse would monitor for shortness of breath, dyspnea, and a worsening of asthma, as they are expected effects/outcomes with this medication. Monitoring for ventricular dysrhythmias is necessary when giving dobutamine, dopamine, and epinephrine but not adenosine. Vomiting is not an expected outcome of adenosine. The nurse should include monitoring for hypotension, not hypertension, in the plan of care after administration of adenosine and instruct parents to change positions slowly to minimize orthostatic hypotension.

Question 3 The nurse is monitoring the client who is taking newly prescribed antihypertensive medication. Which finding should indicate to the nurse that the client might be experiencing an allergic reaction to the medication? Question 3 Answer Choices AMild decrease in blood pressure Mild decrease in blood pressure BIncreased urine output Increased urine output CLeft-sided weakness Left-sided weakness DDevelopment of a rash Development of a rash Question Explanation

Rationale Allergic reactions are often manifested by the presence of a rash, urticaria, gastrointestinal symptoms, and itching. A mild decrease in blood pressure is the intended effect of the medication. Increased urinary output and unilateral weakness are not indications of an allergic reaction.

Question 16 A nurse is assessing a client receiving alteplase for a pulmonary embolism. The client suddenly becomes confused and is unable to follow commands. What action does the nurse take first? Question 16 Answer Choices ANotify the healthcare provider Notify the healthcare provider BReorient the client Reorient the client CCheck the client pupils Check the client's pupils DStop the infusion Stop the infusion Question Explanation

Rationale Alteplase is a thrombolytic medication that causes lysis of blood clots. Alteplase is a high-risk medication that can cause internal bleeding. Sudden neurological deficits may indicate an intracranial bleed. The nurse should stop the infusion. Reorienting the client, checking the pupils, and notifying the healthcare provider are all necessary interventions after the infusion is stopped for safety.

Question 9 A nurse is providing dietary instructions to a client who is taking prescribed amiloride. Which information will the nurse include in the teaching? Question 9 Answer Choices AAvoid eating foods that are rich in potassium such as bananas "Avoid eating foods that are rich in potassium such as bananas." Answer BIt is important to control high-sodium foods such as canned soups "It is important to control high-sodium foods such as canned soups." CEat plenty of foods that contain calcium such as milk "Eat plenty of foods that contain calcium such as milk." DChoose foods that are high in iron content such as shellfish "Choose foods that are high in iron content such as shellfish." Question Explanation

Rationale Amiloride is a potassium-sparing diuretic used in the treatment of edema, hypertension, and potassium loss caused by other diuretic medications. Amiloride may cause hyperkalemia, so the client should be informed to limit their potassium intake. Sodium, calcium, and iron are not affected by the use of amiloride.

Question 13 The nurse is preparing to administer metoprolol to a client with a history of hypertension. Which of the following data is the priority for the nurse to review prior to administration? Question 13 Answer Choices APotassium level Potassium level BMost recent heart rate Most recent heart rate CCreatinine level Creatinine level DRespiratory rate Respiratory rate Question Explanation

Rationale Beta-blockers, such as metoprolol, can decrease heart rate and blood pressure, so the nurse should review these specific vital signs prior to administering the medication. Most prescriptions will state to hold the medication if the heart rate or blood pressure is less than a designated value. Potassium and creatinine levels are monitored with clients who are taking lisinopril, an ACE inhibitor. Respiratory rate is an important part of assessment but is not the priority for the administration of a beta-blocker.

Question 7 The nurse is preparing to administer prescribed digoxin to client with atrial fibrillation. The nurse notes the packaging for the medication is provided in a different route than prescribed. Which action should the nurse take? Question 7 Answer Choices AAdminister the medication as ordered Administer the medication as ordered BConsult the pharmacist regarding the error Consult the pharmacist regarding the error CAlert the charge nurse to the medication error Alert the charge nurse to the medication error DContact the health care provider Contact the health care provider Question Explanation

Rationale Careful consultation with a pharmacist regarding the error is the most appropriate action for the nurse to take if an error occurs when the pharmacy dispenses the medication. The medication as provided by the pharmacy is in and cannot be administered. The charge nurse may be alerted, but the pharmacy can the error.

Question 6 The nurse is caring for a client with a sore throat who developed urticaria after the administration of prescribed antibiotics. The client is now receiving cetirizine. Which finding indicates that the cetirizine is having the intended effect? Question 6 Answer Choices AThe client reports less itching. The client reports less itching. BThe tonsils are decreasing in size. The tonsils are decreasing in size. CThe client reports less muffled hearing. The client reports less muffled hearing. DThe pain rating is decreased. The pain rating is decreased. Question Explanation

Rationale Cetirizine is a second-generation H1 receptor antagonist (antihistamine). Cetirizine binds preferentially to peripheral rather than central H1 receptors. This selectivity reduces the occurrence of drowsiness and CNS depression. Second-generation antihistamines are now commonly used to treat pruritis in urticaria. Almost any drug can lead to an allergic response, especially in the case of sore throats, which can be mononucleosis misdiagnosed as strep throat. A characteristic of infectious mononucleosis is that up to 90 percent of the time that amoxicillin or ampicillin is taken, a rash then develops. The pattern of the rash is commonly maculopapular in appearance. It is very itchy.

Question 21 A client has been prescribed cholestyramine (Questran) in addition to other medications for coronary artery disease and hyperlipidemia. When should the nurse instruct the client to take the cholestyramine? Question 21 Answer Choices AAt least 1 to 2 hours after other medications At least 1 to 2 hours after other medications BAt least 1 hour before meals At least 1 hour before meals CAnytime is acceptable Anytime is acceptable DEarly in the morning on an empty stomach Early in the morning on an empty stomach Question Explanation

Rationale Cholestyramine is a bile acid sequestrant used to reduce LDL cholesterol levels. They are used primarily as adjuncts to statin therapy. Benefits derive from blocking cholesterol synthesis in the liver. The bile-acid sequestrants can form insoluble complexes with other drugs. Medications that undergo binding cannot be absorbed, and hence are not available for systemic effects. Drugs known to form complexes with the sequestrants include thiazide diuretics, digoxin, warfarin, and some antibiotics. To reduce the formation of sequestrant-drug complexes, oral medications that are known to interact should be administered either 1 to 2 hours before the sequestrant or 4 hours after. Cholestyramine works best when taken with meals.

Question 12 The nurse is preparing to administer prescribed warfarin to a client with a mechanical heart valve. Which finding should the nurse report to the healthcare provider? Question 12 Answer Choices AThe INR is 3.0. The INR is 3.0. BThe peripheral IV site has been oozing blood. The peripheral IV site has been oozing blood. CThe aPTT is 30. The aPTT is 30. DThe client has cola-colored urine. The client has cola-colored urine. Question Explanation

Rationale Cola-colored urine is a sign of hematuria. This may be caused by the warfarin or a sign of another problem. It is common to have oozing around IV sites in clients on anticoagulants. The INR of 3.0 is an expected finding. The aPTT should not be affected by warfarin and is also an expected finding.

Question 8 A nurse has administered sublingual nitroglycerin to a client in the emergency department. Which clinical finding indicates an adverse response to the medication? Question 8 Answer Choices APersistent chest pain Persistent chest pain BOrthostatic hypotension Orthostatic hypotension CDecreased heart rate Decreased heart rate DLabored breathing Labored breathing Question Explanation

Rationale Decreased blood pressure when changing positions is an unexpected response to nitroglycerin. The nurse should instruct the client to lay down and elevate the feet to promote venous return. Persistent chest pain is not an unexpected response. Additional doses may be required to alleviate angina. A side effect of nitroglycerin is tachycardia, not a decreased heart rate. Nitroglycerin is not associated with respiratory effects.

Question 6 The nurse is caring for a client prescribed furosemide and digoxin for the treatment of heart failure. The client reports seeing halos and bright lights. Which laboratory result would be anticipated? Question 6 Answer Choices ALow sodium level Low sodium level BLow digitalis level Low digitalis level CLow potassium level Low potassium level DLow serum osmolality Low serum osmolality Question Explanation

Rationale Digitalis toxicity is an accumulation of digitalis (digoxin) in the body that leads to nausea, vomiting, visual disturbances, atrial or ventricular tachydysrhythmias, ventricular fibrillation, sinoatrial block, and atrioventricular block. Clients with heart failure who take digoxin are commonly given diuretics. Hypokalemia can increase the risk of digitalis toxicity. Digitalis toxicity may also develop in the presence of hypomagnesemia. Clients with dig toxicity would have elevated digoxin levels. Sodium would likely be normal. The serum osmolality would likely be normal or high in a client on a diuretic.

Question 6 The nurse is caring for a client who received digoxin-specific immune fab. Which finding indicates the treatment is having the intended effect? Question 6 Answer Choices AIncreased heart rate Increased heart rate BDecreased potassium levels Decreased potassium levels CDecreased blood pressure Decreased blood pressure DIncreased serum digoxin levels Increased serum digoxin levels Question Explanation

Rationale Digoxin-specific immune fab is an antidote that binds molecules of digoxin, making them unavailable for binding at their usual sites of action in the body. After administration of the medication, serum digoxin levels may be misleading, as they will be elevated until the drug is excreted by the kidneys. The goal of treatment is to lower digoxin levels and treat symptomatic digoxin toxicity, specifically cardiac dysrhythmias including bradycardia. Potassium levels may be low, triggering digoxin toxicity, and then elevated due to shifts caused by digoxin toxicity, so fluctuating levels are not a sign of effective treatment. Effective treatment of dysrhythmia should raise blood pressure.

Question 9 A nurse is providing education to a client about newly prescribed diltiazem. Which statement will the nurse include in the teaching? Question 9 Answer Choices ASkip the dose if your systolic blood pressure is less than 120 mmHg "Skip the dose if your systolic blood pressure is less than 120 mmHg." BHold the dose if your heart rate is less than 50 beats/min "Hold the dose if your heart rate is less than 50 beats/min." CCall your healthcare provider if you experience any fever "Call your healthcare provider if you experience any fever." DNotify your healthcare provider if you notice any weight loss "Notify your healthcare provider if you notice any weight loss." Question Explanation

Rationale Diltiazem is a calcium channel blocker medication used in the treatment of hypertension and cardiac arrhythmias such as atrial flutter and fibrillation. Diltiazem can cause bradycardia. The nurse should instruct the client how to take their pulse and hold the dose if less than 50 beats/min. Diltiazem should be held if the systolic blood pressure is below 90 mmHg. Fever and weight loss are not effects associated with the use of diltiazem.

Question 5 The nurse is monitoring an older adult client prescribed diphenhydramine for contact dermatitis related to poison ivy exposure. Which finding should be reported to the provider as a potential drug-related side effect? Question 5 Answer Choices AConfusion Confusion BHypertension Hypertension CIncontinence Incontinence DBradypnea Bradypnea Question Explanation

Rationale Diphenhydramine and other first-generation H1 receptor antagonists may cause confusion (with impaired thinking, judgment, and memory), dizziness, hypotension, sedation, syncope, unsteady gait, and paradoxical central nervous system stimulation in older adults. Older adults may experience urinary retention, especially those with prostatic hypertrophy. Some of these adverse reactions derive from the anticholinergic effects of the drugs and are likely to be more severe if the patient is also taking other drugs with anticholinergic effects. Diphenhydramine is sometimes prescribed as a sleep aid for occasional use in older adults. As with many other drugs, smaller-than-usual dosages are indicated.

Question 5 The nurse is providing education to the client with sinusitis who has asked about taking over-the-counter pseudoephedrine. Which of the following statements is appropriate? Question 5 Answer Choices AIf you take pseudoephedrine and phenylephrine together, you will get more relief "If you take pseudoephedrine and phenylephrine together, you will get more relief." BContinue the medication until your congestion resolves "Continue the medication until your congestion resolves." CUsing these kinds of medications may make you jittery and restless "Using these kinds of medications may make you jittery and restless." DIt is safe to chew over the counter medications if you have trouble swallowing pills "It is safe to chew over the counter medications if you have trouble swallowing pills." Question Explanation

Rationale Do not combine two drug preparations containing the same or similar active ingredients. For example, pseudoephedrine is the nasal decongestant component of most prescription and over-the-counter (OTC) sinus and multi-ingredient cold remedies. Taking more than one preparation containing pseudoephedrine (or phenylephrine, a similar drug) may increase the dosage to toxic levels and cause irregular heartbeats and extreme nervousness. Oral OTC decongestants should not be used longer than one week. Excessive or prolonged use may damage nasal mucosa and produce chronic nasal congestion. Common side effects include tachycardia, impaired coordination, dizziness, excitability, headache, insomnia, restlessness, seizures, vertigo, dysuria, urinary retention, urinary difficulty, and thrombocytopenia.

Question 20 The nurse is completing a health history of a client diagnosed with Alzheimer's disease. The nurse reviews a list of the client's medications and supplements routinely taken at home. Which treatment should be a cause for concern by the nurse? Question 20 Answer Choices ADonepezil Donepezil BGinkgo biloba Ginkgo biloba COmega-3 fatty acids Omega-3 fatty acids DCoconut oil Coconut oil Question Explanation

Rationale Donepezil, rivastigmine, and galantamine are most commonly used in the treatment of Alzheimer's disease (AD). Complementary and integrative therapies used to treat AD include Gingko biloba (a plant extract) and omega-3 fatty acids. While there isn't sufficient research to support using these treatments, continued use won't necessarily be harmful. However, coconut oil, which is a source of caprylic acid, is a concern. While there has been limited research on Katasyn (an experimental drug containing caprylic acid), there is no scientific evidence that coconut oil is safe and effective or prevents cognitive decline.

Question 7 The nurse is collecting the health history for a client who reports a sudden onset of generalized weakness and fatigue. The nurse notes the client has a new prescription for spironolactone. Which action should the nurse take first? Question 7 Answer Choices AReview the drug formulary for side effects Review the drug formulary for side effects Answer BRequest the health care provider to stop the medication Request the health care provider to stop the medication CNotify the pharmacist of the findings Notify the pharmacist of the findings DDocument the findings Document the findings Question Explanation

Rationale During medication administration, it is important for the nurse to assess knowledge of drugs, including adverse effects and physiologic factors that affect drug action. Information about specific drugs is available in pharmacology texts and drug reference books. Calling the health care provider may be an option after reviewing the drug formulary. The nurse should notify the pharmacist if the medication is the cause of the symptoms. The nurse will document the findings, but the priority is to review the formulary.

Question 4 The nurse is assisting a client who is taking amlodipine with meal planning. Which fluid selected by the client would require follow up by the nurse? Question 4 Answer Choices ABlack coffee Black coffee BGrapefruit juice Grapefruit juice Answer CGreen tea Green tea DChocolate Milk Chocolate Milk Question Explanation

Rationale Grapefruit juice affects the metabolism of certain medications, such as amlodipine, and may cause toxicity if taken together. Clients who are taking antibiotics, such as tetracycline, should avoid consuming milk products. Clients who are taking warfarin should avoid consuming green tea. Clients who are taking stimulants should avoid consuming black coffee.

Question 11 The nurse is educating a client with end-stage renal failure about newly prescribed aluminum hydroxide. Which statement should the nurse include in the teaching? Question 11 Answer Choices AThis medication binds with phosphates from food to decrease absorption "This medication binds with phosphates from food to decrease absorption." BThis medication is used to decrease urea to prevent urticaria&nbsp; "This medication is used to decrease urea to prevent urticaria." CThis medication will coat the lining of the stomach to decrease acid production "This medication will coat the lining of the stomach to decrease acid production." DThis medication treats hyperkalemia by exchanging sodium for potassium in the intestines "This medication treats hyperkalemia by exchanging sodium for potassium in the intestines." Question Explanation

Rationale Hyperphosphatemia occurs in end-stage renal failure when kidneys can no longer filter out phosphorus. Treatment of hyperphosphatemia may include the administration of aluminum hydroxide as a phosphate-binding agent. The aluminum binds with phosphates which are excreted in the feces. Sodium polystyrene is used to treat hyperkalemia by exchanging sodium for potassium in the intestines. Dialysis is used to remove urea from the blood, and diphenhydramine is used to treat urticaria. Sucralfate is a medication that coats the stomach lining to decrease acid production.

Question 4 The nurse is assessing a postpartum client who is taking labetalol. Which client report should the nurse identify as a potential adverse effect of the medication? Question 4 Answer Choices ANausea Nausea BAnkle edema Ankle edema CAbdominal pain Abdominal pain DDizziness Dizziness Question Explanation

Rationale Labetalol is a beta-blocker that is used for blood pressure management in postpartum clients. The mechanism of action for labetalol is to vasodilate, which could lead to a decrease in blood pressure. A client with a sudden drop in blood pressure could report dizziness. Report of nausea or ankle edema is normal during pregnancy. Abdominal pain in pregnancy could be from active labor or constipation.

Question 17 A nurse is assessing a client receiving intravenous potassium chloride. The client verbalizes pain to the IV site. The site appears swollen and is warm to touch. Which action does the nurse perform? Question 17 Answer Choices ADecrease the rate of the infusion BApply ice to the IV access site CInform the client that this is an expected finding DDiscontinue the IV catheter Question Explanation

Rationale The nurse should discontinue the IV catheter. The client's symptoms are indicative of phlebitis, inflammation of the vein. Decreasing the rate of the infusion will not treat the swelling or injury to the vein. Applying ice to the access site does not address the possible vein injury caused by the medication. Pain, swelling, and warmth are not expected findings for a patent IV access site.

Question 11 The nurse is collecting the health history of a client with heart disease who reports experiencing episodes of diarrhea. The client reports taking loperamide at home. Which of the following statements should the nurse make? Question 11 Answer Choices ATaking this medication may increase your risk of an abnormal heart rhythm "Taking this medication may increase your risk of an abnormal heart rhythm." BIt is safe to drink alcohol while using this medication "It is safe to drink alcohol while using this medication." CUsing this medication may cause dependence "Using this medication may cause dependence." DStop taking this medication if your symptoms do not improve by tomorrow "Stop taking this medication if your symptoms do not improve by tomorrow." Question Explanation

Rationale Loperamide decreases GI motility and is a nonprescription drug. It does not cause the central nervous system effects associated with opiate derivatives and lacks the potential for abuse. Loperamide should not be continued after 48 hours if improvement has not occurred. Loperamide has a black box warning because torsades de pointes, cardiac arrest, and death have been reported in people using higher than recommended dosages. Alcohol can increase the nervous system side effects of loperamide, such as dizziness, drowsiness, and difficulty concentrating.

Question 1 A nurse is reviewing a client's medical history. The client has been newly diagnosed with hypertension and has been prescribed oral losartan as treatment. The nurse will clarify the use of losartan if which comorbidity is noted in the client's medical record? Question 1 Answer Choices ARenal stenosis Renal stenosis Answer BHyperlipidemia Hyperlipidemia Your Answer CAtrial fibrillation Atrial fibrillation DDiabetes Diabetes Question Explanation

Rationale Losartan is an angiotensin II receptor blocker used in the treatment of hypertension. Losartan is contraindicated in clients with renal stenosis due to the risk of kidney injury. Hyperlipidemia, atrial fibrillation, and diabetes are not known to be contraindicated in the use of losartan.

Question 11 The nurse is educating a client with end stage chronic obstructive pulmonary disease (COPD) about medication management. Which statement by the client indicates an understanding of the teaching? Question 11 Answer Choices AI will use the albuterol in the nebulizer before my other inhalers each morning "I will use the albuterol in the nebulizer before my other inhalers each morning." BI can use my tiotropium inhaler if I get short of breath "I can use my tiotropium inhaler if I get short of breath." CI will only use the fluticasone inhaler on the days I am really out of breath "I will only use the fluticasone inhaler on the days I am really out of breath." DThe side effects of these medications will be less severe because I&rsquo;m not taking them by mouth "The side effects of these medications will be less severe because I'm not taking them by mouth." Question Explanation

Rationale Medication regimens used to treat COPD are based on disease severity. For grade III or IV (severe and very severe) COPD, medication therapy includes treatment with one or more bronchodilators and inhaled corticosteroids. Clients with COPD experience significant breathlessness and reduced FEV1 upon waking. Use of nebulized albuterol prior to administration of long-acting medications relaxes the airway and allows other medications to get deeper into the lungs. Tiotropium is a long-acting anticholinergic (muscarinic) and is not meant for rescue purposes. Fluticasone prevents inflammation and therefore, must be used every day. Clients with COPD will experience side effects of the medications due to the long duration of use.

Question 11 The nurse is providing education to the client prescribed montelukast for the treatment of asthma. What medication should the nurse instruct the client to avoid? Question 11 Answer Choices AIbuprofen Ibuprofen BPrednisone Prednisone CAmoxicillin Amoxicillin DFormoterol Formoterol Question Explanation

Rationale Montelukast should not be taken with NSAIDs. It increases the risk of bleeding as well as has the potential to make asthma symptoms worse. Prednisone, amoxicillin, and formoterol are all safe to administer to the client on montelukast.

Question 7 The nurse is monitoring a client who is taking prescribed nitroglycerin for angina. Which finding indicates the medication has a therapeutic effect? Question 7 Answer Choices AThe client blood pressure is 150/80 mm/Hg. The client's blood pressure is 150/80 mm/Hg. BThe client heart rate is 110. The client's heart rate is 110. CThe client reports a decrease in chest pressure. The client reports a decrease in chest pressure. DThe client reports a headache. The client reports a headache. Question Explanation

Rationale Nitroglycerin acts to decrease myocardial oxygen consumption. Dilatation of the veins reduces the amount of blood returning to the heart (preload), so the chambers have a smaller volume to pump resulting in decreased oxygen needs. Decreased oxygen demand reduces pain caused by dilating coronary blood flow. While blood pressure may decrease slightly due to the vasodilatory effects of nitroglycerin, it is a secondary effect and not the desired therapeutic effect of this drug. Increased blood pressure and increased preload would mean the heart would work harder, increasing oxygen demand and thus angina. Decreased heart rate is not an effect of nitroglycerin.

Question 7 A nurse is teaching a client with stable angina about newly prescribed SL nitroglycerin. Which statement should the nurse include in the teaching? Question 7 Answer Choices A"Take this medication after each meal and at bedtime." "Take this medication after each meal and at bedtime." B"Take one tablet 30 minutes before any physical activity." "Take one tablet 30 minutes before any physical activity." C"Take one tablet immediately when you experience chest pain." "Take one tablet immediately when you experience chest pain." D"Take this medication with 8 ounces of water." "Take this medication with 8 ounces of water." Question Explanation

Rationale Nitroglycerin is a vasodilator used to treat angina or ischemic chest pain. When teaching a client about SL nitroglycerin, the nurse should instruct the client to take one tab and place it under their tongue immediately when experiencing chest pain. The client only takes this medication when experiencing chest pain. The client should not eat or drink when taking this medication.

Question 5 The nurse is providing teaching to the client prescribed albuterol for the management of asthma. The nurse is including reportable side effects in the teaching plan. Which of the following side effects is the priority? Question 5 Answer Choices ANervousness Nervousness BHeadache Headache CPalpitations Palpitations DMuscle aches Muscle aches Question Explanation

Rationale Side effects of albuterol include nervousness, shakiness, headache, throat irritation, and muscle aches. Muscle tremor is the most frequent adverse effect. The main risks with adrenergic bronchodilators, particularly in older adults, are excessive cardiac and central nervous system (CNS) stimulation. Symptoms of cardiac stimulation include angina, tachycardia, and palpitations. Symptoms of central nervous system (CNS) stimulation consist of agitation, anxiety, insomnia, seizures, and tremors. Other reported effects may include serious dysrhythmias and cardiac arrest.

Question 15 A nurse is providing care to an older adult client with newly diagnosed heart failure. The nurse receives a prescription for digoxin PO 1.5 mg daily. Which action does the nurse perform next? Question 15 Answer Choices AInstruct the client to take the heart rate before administration Instruct the client to take the heart rate before administration BEducate the client on the purpose of digoxin Educate the client on the purpose of digoxin CAdminister the medication to the client Administer the medication to the client DClarify the prescription with the healthcare provider Clarify the prescription with the healthcare provider Question Explanation

Rationale Older adult clients (geriatric) have a high sensitivity to the toxic effects of digoxin. A dose of 1.5 mg daily is above the recommended range for adults. The initial daily dose for a geriatric client should not exceed 0.125 mg. Educating the client on the purpose of digoxin and performing related assessments are expected interventions. However, the nurse should clarify the dose first. Administering the prescribed medication dose to the client may result in significant side effects.

Question 8 A nurse is assessing a client who started taking prescribed olmesartan 2 weeks ago. Which finding indicates an expected response to the medication? Question 8 Answer Choices AHeart rate of 85 beats/min Heart rate of 85 beats/min BUrinary output of 45 ml/hr Urinary output of 45 ml/hr CBlood pressure of 125/79 mmHg Blood pressure of 125/79 mmHg Answer DRespiratory rate of 20 breaths/min Respiratory rate of 20 breaths/min Question Explanation

Rationale Olmesartan is an angiotensin II receptor antagonist used in the treatment of hypertension. The expected outcome is to maintain the blood pressure within normal limits. Although within normal limits, the heart rate, urinary output, and respiratory rate are not used to evaluate the efficacy of olmesartan.

Question 9 A nurse is providing discharge education on the use of sustained-release procainamide to a client with newly diagnosed atrial flutter. What will the nurse include in the teaching? Question 9 Answer Choices AYou will need to have laboratory blood tests performed every 3 months "You will need to have laboratory blood tests performed every 3 months." BHold the medication if your heart rate is below 70 beats/min "Hold the medication if your heart rate is below 70 beats/min." CNotify your healthcare provider if you begin experiencing joint pain "Notify your healthcare provider if you begin experiencing joint pain." DCrush your medication and mix it with food to mask the taste "Crush your medication and mix it with food to mask the taste." Question Explanation

Rationale Procainamide is an antiarrhythmic medication used in the management of atrial flutter. One of the adverse effects of procainamide is systemic lupus syndrome characterized by fever and painful joints. The client should be instructed to notify the healthcare provider of any adverse symptoms. Lab tests need to be conducted frequently (every week) at the start of therapy to monitor complete blood counts and procainamide blood levels. Procainamide does not have a direct effect on heart rate. Sustained-release medications should not be crushed or chewed.

Question 6 The nurse is caring for a client after cardiac surgery who has been prescribed protamine sulfate. Which finding indicates that the treatment is having the intended effect? Question 6 Answer Choices AThe international normalized ratio (INR) is trending down. The international normalized ratio (INR) is trending down. BThe bleeding from the surgical site has slowed. The bleeding from the surgical site has slowed. CThe client reports decreased chest pain. The client reports decreased chest pain. DThe respiratory rate is increased. The respiratory rate is increased. Question Explanation

Rationale Protamine sulfate is the antidote for standard heparin and low molecular weight heparins (LMWHs). Protamine is typically given for bleeding that may not respond to merely withdrawing the heparin or when hemorrhaging is present. INR is used to determine the therapeutic level of warfarin (Coumadin), not heparin. Chest pain would be treated with nitroglycerin, but not protamine. The respiratory rate would be increased by naloxone if opiates were prescribed.

Question 7 The nurse is collecting the health history of a client who reports taking over-the-counter pseudoephedrine for nasal congestion. Which statement by the client would require follow-up by the nurse? Question 7 Answer Choices AI take this medication at night before I go to bed "I take this medication at night before I go to bed." BI have to use a normal saline nasal spray since I started this medication "I have to use a normal saline nasal spray since I started this medication." CI avoid drinking beverages with caffeine while taking the medication "I avoid drinking beverages with caffeine while taking the medication." DI chew gum when I take this medication to help with my dry mouth "I chew gum when I take this medication to help with my dry mouth." Question Explanation

Rationale Pseudoephedrine is a nasal decongestion that causes vasoconstriction in the respiratory mucosa and bronchodilatation making it easier for the client to breathe. The medication is a stimulant, so clients should avoid taking the medication before bed to prevent insomnia. The use of caffeine will exacerbate the alpha-adrenergic effect of this drug. Chewing gum helps alleviate dry mouth that accompanies respiratory mucosa constriction that occurs when taking this medication. This medication can dry mucus membranes, so clients may use a normal saline nasal spray.

Question 21 Question 21 The nurse is educating a client about how to use a metered-dose inhaler with spacer. Place each step in the correct order by entering the numbers in order. Only enter numbers, no spaces or commas. 1. Breathe out slowly 2. Breathe in deeply 3. Release the medication into the spacer 4. Remove the mouthpiece from the lips 5. Hold breath for 10 seconds 32451 Question Explanation

Rationale Release the medication into the spacer. Breathe in deeply. Remove the mouthpiece, then hold breath for 10 seconds, then breathe out slowly. Spacers are highly recommended when inhalers are used because they increase the availability of the medication to the client.

Question 5 The nurse is teaching a client about newly prescribed inhaled budesonide. The nurse should teach the client to report which finding to the healthcare provider? Question 5 Answer Choices ARounded face Rounded face BBradycardia Bradycardia CIncreased thirst Increased thirst DCough Cough Question Explanation

Rationale Respiratory disorders, such as asthma, status asthmatic, chronic obstructive pulmonary disease (COPD), and rhinitis, may all be treated with corticosteroids, including budesonide. Corticosteroids have many common side effects including cushingoid features, such as "moon face" due to redistribution of fat. Fluid retention is also common when using corticosteroids. Increased thirst may be an indication of hyperglycemia and should be reported. Corticosteroids can increase heart rate. A cough is normal with corticosteroids as the airway is dilated.

Question 6 The nurse is caring for a client diagnosed with diabetic ketoacidosis who is receiving 50 mEq of sodium bicarbonate in 1 L of dextrose 5% in water via a central venous access device. The client has three new prescriptions for continuously infused medications. Which action is appropriate? Question 6 Answer Choices ARefer to an IV compatibility chart Refer to an IV compatibility chart BRequest that an additional IV access be inserted Request that an additional IV access be inserted CUse a Y-site connector to infuse two medications in the same port Use a Y-site connector to infuse two medications in the same port DInsert a peripheral intravenous access Insert a peripheral intravenous access Question Explanation

Rationale Sodium bicarbonate is incompatible with many other drugs and solutions. The nurse should consult a drug compatibility reference for more information on which drugs can be administered via connection at the most distal IV tubing port. Y tubing should not be added to an IV until compatibility is determined. Y tubing does not prevent the mixing of infusions. Adding an additional access may be unnecessary if compatibility is determined and may pose an unnecessary infection risk to the client. A central line is the preferred access for drugs that have a pH less than 5 and greater than 9. Certain drugs are venous irritants regardless of pH or concentration; therefore, a PIV would be inappropriate.

Question 12 The nurse is caring for a client with a diagnosis of cardiogenic shock who has been prescribed dobutamine infusion. Which action should the nurse take first? Question 12 Answer Choices ACompare the packaging of the medication to the prescription Compare the packaging of the medication to the prescription BPrime the IV tubing with the medication Prime the IV tubing with the medication CSet the infusion pump for the infusion rate Set the infusion pump for the infusion rate DIncrease the frequency of blood pressure and heart rate monitoring on the bedside monitor Increase the frequency of blood pressure and heart rate monitoring on the bedside monitor Question Explanation

Rationale The Food and Drug Administration and Institute for Safe Medication Practices has maintained a list of drug name pairs and trios that look and sound similar. These medications are called out with tall man lettering on dissimilar lettering on the packaging (capital letters). Dobutamine is often confused with dopamine and is included in the list. Ensuring the use of the right medication is a part of the rights of medication administration. This should be done before priming the tubing with the medication and setting the infusion pump. While frequent vital signs are important for the client with shock, it isn't the most important action in this scenario.

Question 12 The nurse is teaching a pediatric client and family about prescribed albuterol sulfate extended-release tablets. Which statement should be included? Question 12 Answer Choices AIf you cannot swallow the tablet, it is ok to chew it "If you cannot swallow the tablet, it is ok to chew it." BThis medication can cause restlessness "This medication can cause restlessness." CRinse your mouth after taking this medication "Rinse your mouth after taking this medication." DOral albuterol can cause an increase in urination "Oral albuterol can cause an increase in urination." Question Explanation

Rationale The adverse reactions to albuterol are the same whether administered orally or via inhalation. The most frequent adverse reactions to albuterol are nervousness, tremors, headache, tachycardia, and palpitations. Less frequent adverse reactions are muscle cramps, insomnia, nausea, weakness, dizziness, drowsiness, flushing, restlessness, irritability, chest discomfort, and difficulty in urination. Extended-release medications should not be chewed or crushed. Doing so can release all of the drug at once, increasing the risk of side effects. Inhaled corticosteroids require the mouth to be rinsed. This medication is not inhaled and is not a corticosteroid.

Question 16 During morning rounds, a healthcare provider informs a client with hypertension that a calcium channel blocker will be added to their treatment regimen. The nurse notes a new prescription for amiloride 10 mg PO daily. Which action does the nurse perform next? Question 16 Answer Choices AClarify the prescription with the healthcare provider Clarify the prescription with the healthcare provider BEducate the client on the new prescription Educate the client on the new prescription CAdminister the medication with food Administer the medication with food DAssess the client blood pressure Assess the client's blood pressure Question Explanation

Rationale The nurse should clarify the new prescription. Amiloride is a potassium-sparing diuretic. It is also a look-alike/sound-alike medication commonly confused with amlodipine, a calcium channel blocker. Educating the client on a new prescription, administering the medication with food, and assessing the blood pressure are important interventions for amiloride. However, this prescription should be clarified.

Question 16 A nurse is reviewing new prescriptions for a client diagnosed with heart failure. The nurse notes captopril 25mg PO. Which action does the nurse perform next? Question 16 Answer Choices AAdminister the medication before meals Administer the medication before meals BClarify the prescription with the healthcare provider Clarify the prescription with the healthcare provider CTake the client weight Take the client's weight DCheck the client latest creatinine level Check the client's latest creatinine level Question Explanation

Rationale The nurse should clarify the prescription with the healthcare provider. The prescription is missing a frequency, a necessary component of a medication prescription. Captopril should be administered before or after meals. However, the prescription does not have a frequency and should be clarified. Taking the client's weight and checking renal labs are important interventions after the prescription is clarified.

Question 16 A nurse is preparing to administer intravenous mannitol to a client with increased intracranial pressure. Which action will the nurse perform prior to administering the medication? Question 16 Answer Choices AConnect an in-line filter to the infusion tubing Connect an in-line filter to the infusion tubing BDilute the medication with lactated ringers Dilute the medication with lactated ringers CPrepare an infusion warmer Prepare an infusion warmer DEnsure the client has a patent central line Ensure the client has a patent central line Question Explanation

Rationale The nurse should connect a filter to the infusion tubing prior to administering mannitol. Mannitol is an osmotic diuretic that may contain crystals within the solution. The in-line filter prevents the administration of particulates into the bloodstream. Mannitol should be administered undiluted. An infusion warmer is not required for the administration of mannitol. Mannitol can be administered through a peripheral line.

Question 15 A nurse is providing care to a client in cardiogenic shock. The client is on a prescribed dopamine infusion at 10 mcg/kg/min with orders to titrate as needed. The latest blood pressure is 75/40 mmHg. Which action does the nurse perform next? Question 15 Answer Choices ARecheck the client blood pressure Recheck the client's blood pressure BIncrease the infusion rate to 12 mcg/kg/min Increase the infusion rate to 12 mcg/kg/min CReport the findings to the healthcare provider Report the findings to the healthcare provider DDecrease the infusion rate to 8 mcg/kg/min Decrease the infusion rate to 8 mcg/kg/min Question Explanation

Rationale The nurse should increase the dose of dopamine. The therapeutic goal of dopamine is to increase blood pressure and improve cardiac output. A blood pressure of 75/40 mmHg indicates the current rate is not effective. Rechecking the client's blood pressure is not necessary. The client is in cardiogenic shock and hypotension is an expected finding. The nurse can recheck the blood pressure after titrating the dose. Reporting the findings to the healthcare provider is important. However, the nurse should first titrate the dose to ensure the client's blood pressure is maintained. Decreasing the infusion rate will cause further hypotension.

Question 12 The nurse is educating a client on self-administration of a fluticasone inhaler. What statement indicates an understanding of the teaching? Question 12 Answer Choices AI will rinse my mouth with water after using the inhaler "I will rinse my mouth with water after using the inhaler." BDisinfectant wipes can be used to clean the spacer "Disinfectant wipes can be used to clean the spacer." CI need to wait 15 minutes between puffs "I need to wait 15 minutes between puffs." DThis inhaler should be used before the others "This inhaler should be used before the others." Question Explanation

Rationale To prevent thrush, the client should rinse his or her mouth with water and spit it out. The spacer should be washed with warm water and dish detergent. The client may need two puffs but does not have to wait 15 minutes between. Bronchodilators should be used before corticosteroids.

Question 8 A nurse is assessing a client with heart failure who is taking prescribed torsemide. Which clinical finding indicates effectiveness of the medication? Question 8 Answer Choices ASymmetrical pulses bilaterally Symmetrical pulses bilaterally BFull strength to bilateral extremities Full strength to bilateral extremities CIntact whisper test Intact whisper test DAbsence of peripheral edema Absence of peripheral edema Question Explanation

Rationale Torsemide is a loop diuretic used in the treatment of hypertension and fluid overload. The expected therapeutic response of torsemide is a decrease in fluid retention evidenced by the absence of peripheral edema. Symmetrical pulses bilaterally and full strength to bilateral extremities do not evaluate the effectiveness of torsemide. An intact whisper test indicates the absence of ototoxicity, an adverse effect of torsemide. However, this does not evaluate medication effectiveness.

Question 9 A nurse is providing education on activities of daily living to a client taking warfarin. Which statement made by the client indicates further teaching is required? Question 9 Answer Choices A "I will brush my teeth using a soft-bristled toothbrush." B "I will wear a medical alert bracelet on my wrist." C "I will be sure to consume plenty of green leafy vegetables." D "I need to shave using an electric razor." Question Explanation

Rationale Warfarin is an anticoagulant medication used in the treatment of blood clotting disorders. Green leafy vegetables contain a high amount of vitamin K, the antidote for warfarin. The client should be instructed to limit their intake of vitamin K-containing foods. Using a soft-bristled toothbrush and an electric razor decrease the risk of bleeding. A medical alert bracelet is necessary for clients who are on blood-thinning medications to alert first responders in case of an emergency.

Question 3 The home health nurse is completing a medication reconciliation of a client who has a new prescription for warfarin. Which medication should the nurse question the healthcare provider about? Question 3 Answer Choices AAspirin Aspirin BNifedipine Nifedipine CNPH insulin NPH insulin DVitamin D supplement Vitamin D supplement Question Explanation

Rationale Warfarin is an anticoagulant that prevents blood from clotting by blocking the synthesis of vitamin K. Clients taking warfarin are at increased risk for bleeding. Aspirin, which is an anti-platelet aggregation, prevents platelets from clumping together. Taking warfarin and aspirin together could increase the risk of bleeding and should be questioned. Nifedipine is a calcium channel blocker and does not interact with warfarin. Insulin and vitamin D supplement do not cause adverse effects when taken with warfarin.

Question 20 The nurse is caring for a client prescribed warfarin therapy for treatment of persistent atrial fibrillation. Which of the following may potentiate the effect of this medication? Question 20 Answer Choices ASt. John wort St. John's wort BEstrogen Estrogen CVitamin K Vitamin K DGreen tea Green tea Question Explanation

Rationale Warfarin, an anticoagulant agent used to prevent thrombosis and risk of stroke in clients with atrial fibrillation, is associated with many drug and food interactions. Careful assessment with a pharmacist/formulary is recommended to avoid potential complications. Green tea can potentiate the effect of warfarin and increase bleeding. St. John's wort, estrogen, and vitamin K may inhibit the action requiring higher doses of the anticoagulant.

Question 1 A charge nurse is observing a staff nurse prepare 1 ml of intravenous digoxin for a client with heart failure. After the staff nurse prepares the medication, the nurse notices precipitate in the syringe. Which action by the staff nurse likely caused this reaction? Question 1 Answer Choices AD5W was used as the diluent. D5W was used as the diluent. Your Answer BThe medication was not allowed to reach room temperature. The medication was not allowed to reach room temperature. CThe medication was added to 1 mL of diluent. The medication was added to 1 mL of diluent. Answer DAir was not inserted into the vial. Air was not inserted into the vial. Question Explanation

Rationale When administering digoxin, 1 milliliter of digoxin should be mixed into at least 4 milliliters of diluent. Using a smaller amount of diluent will cause precipitation of the medication. Dextrose 5% in water (D5W) is compatible with digoxin and can be used to dilute the medication. Digoxin is not a temperature-controlled medication. Precipitation occurs as a result of incompatibilities or improper mixing. The insertion of air into a vial facilitates the withdrawal of the medication. Omission of this does not cause medication precipitation.

179) Which

Rationale explains why the nurse would question a benzodiazepine prescription for an individual experiencing acute grief? The depression is magnified, and the risk of suicide increases. Brain activity is suppressed, and the risk of depression increases. Lethargy results, and it prevents a return to interpersonal activity. The period of denial is extended, and the grieving process is suppressed. +Rationale With this sedating medication, the individual does not face the reality of the loss and merely delays the onset of the pain associated with it. Because most support is available at the time of the death and the funeral, a benzodiazepine at this time denies the individual the opportunity to use this assistance. This class of medications does not magnify the risk of suicide or cause or prevent depression. Although sedation and muscle relaxation initially may occur with these medications, they are not reasons for the medications not to be ordered.

247) Which

Rationale will the nurse provide to a client with Crohn's disease who asks why the prescribed vitamins have to be given intravenously (IV) rather than by mouth? Select all that apply. One, some, or all responses may be correct. "They provide more rapid action results." "They decrease colon irritability." "Oral vitamins are less effective." "Intestinal absorption may be inadequate." "Allergic responses are less likely to occur." :Rationale Absorption through the gastrointestinal (GI) tract is impaired, and parenteral administration goes directly into the intravascular compartment. Disease of the GI tract hampers absorption. Because the mucosa of the intestinal tract is damaged, its ability to absorb vitamins taken orally is greatly impaired. IV vitamins do not decrease colonic irritability. Route of administration does not affect allergic response. STUDY TIP: Becoming a nursing student automatically increases stress levels because of the complexity of the information to be learned and applied and because of new constraints on time. One way to decrease stress associated with school is to become very organized so that assignment deadlines or tests do not come as sudden surprises. By following a consistent plan for studying and completing assignments, you can stay on top of requirements and thereby prevent added stress.

958) Warfarin is prescribed for a client who has been receiving intravenous (IV) heparin for a partial occlusion of the left common carotid artery. The client expresses concern about why both medications are needed at the same time. Which

Rationale would the nurse include to address the client's concern? This permits the administration of smaller doses of each medication. Giving both medications allows clot dissolution while preventing new clot formation. Heparin provides anticoagulant effects until warfarin reaches therapeutic levels. Administration of heparin with warfarin provides immediate and maximum protection against clot formation. +Rationale Warfarin is administered orally for 2 to 3 days to achieve the desired effect on the international normalized ratio (INR) level before heparin is discontinued. These medications do not dissolve clots already present. Because each medication affects a different part of the coagulation mechanism, dosages must be adjusted separately. That this approach immediately provides maximum protection against clot formation is not the reason for the administration of both medications; warfarin will not exert an immediate therapeutic effect. Test-Taking Tip: Multiple-choice questions can be challenging, because students think that they will recognize the right answer when they see it or that the right answer will somehow stand out from the other choices. This is a dangerous misconception. The more carefully the question is constructed, the more each of the choices will seem like the correct response.

6. 6.ID: 20127756656 Upon admission to the emergency center, an adult client with acute status asthmaticus is prescribed this series of medications. In which order should the nurse administer the prescribed medications? (Arrange from first to last.) Incorrect A. Prednisone (Deltasone) orally. B. Gentamicin (Garamycin) IM. C. Albuterol (Proventil) puffs. D. Salmeterol (Serevent Diskus).

The correct order is: E. Albuterol (Proventil) puffs. F. Salmeterol (Serevent Diskus). G. Prednisone (Deltasone) orally. H. Gentamicin (Garamycin) IM. Status asthmaticus is potentially a life-threatening respiratory event, so albuterol, a beta2 adrenergic agonist and short acting bronchodilator, should be administered by inhalation first to provide rapid and deep topical penetration to relieve bronchospasms, dilate the bronchioles, and increase oxygenation. In stepwise management of persistent asthma, a long-action bronchodilator, such as salmeterol (Serevent Diskus), with a 12-hour duration of action should be given next. Prednisone, an oral corticosteroid, provides prolonged anti-inflammatory effects and should be given after the client's respiratory distress begins to resolves. Gentamicin, an antibiotic, is given deep IM, which can be painful, and may require repositioning the client, so should be last in the sequence.

60) Which conclusion would the nurse make about the development of loose stools and increased salivation two days after a client with myasthenia gravis begins taking pyridostigmine? The client is experiencing a myasthenic crisis. The medication is causing cholinergic side effects. The medication is triggering a paradoxical reaction. The client is exhibiting toxic effects of the medication.

The medication is causing cholinergic side effects rationale Because this medication inhibits the destruction of acetylcholine, parasympathetic activity may increase, resulting in cholinergic side effects such as diarrhea and increased salivation. The signs do not indicate a myasthenic crisis. Myasthenic crisis is characterized by difficulty breathing or speaking, morning headaches, feeling tired during the daytime, waking up frequently at night, not sleeping well, a weak cough with increased secretions (mucus or saliva), an inability to clear secretions, a weak tongue, trouble swallowing or chewing, and weight loss. Side effects are not temporary and not paradoxical; they continue as long as the medication is continued. The dosage may be adjusted or an anticholinergic may be given to limit side effects. Toxicity or cholinergic crisis is manifested by increased muscle weakness, including muscles of respiration.

13) Which medication class includes amitriptyline? Tricyclics Monoamine oxidase inhibitors (MAOIs) Selective serotonin reuptake inhibitors (SSRIs) Serotonin-norepinephrine reuptake inhibitors (SNRIs)

Tricyclics rationale Amitriptyline is one of several tricyclic antidepressants used to treat anxiety disorders. It is not an MAOI, SSRI, or SNRI. Test-Taking Tip: Make certain that the answer you select is reasonable and obtainable under ordinary circumstances and that the action can be carried out in the given situation. 48) A client with a history of methamphetamine use is admitted to the medical unit. Which clinical manifestation would the nurse expect when assessing the client who is under the influence on admission? Constricted pupils Intractable diarrhea Increased heart rate Decreased respirations :rationale Methamphetamine is a stimulant that causes the release of adrenaline, which activates the sympathetic nervous system and causes increased heart rate. The pupils will dilate, not constrict, because the sympathetic nervous system is activated. Clients withdrawing from opioids, not methamphetamine, experience diarrhea. The respirations will increase, not decrease, because of the activation of the sympathetic nervous system.

208) Which adverse effect of imipramine requires further assessment and possible immediate medical intervention? Dry mouth Weight gain Blurred vision Urinary hesitancy

Urinary hesitancy rationale Urinary hesitancy and retention are adverse effects of imipramine that may require immediate medical intervention. Dry mouth, weight gain related to increased appetite, and blurred vision may occur as side effects of imipramine; they usually decrease over time or can be managed through nursing interventions.

Question 20 The nurse is observing a new graduate nurse preparing to administer bumetanide 4 mg orally to a client with heart failure. Which client finding requires the nurse to intervene immediately? Question 20 Answer Choices AThe client's most recent serum potassium level is 2.9 mEq or mmol/L. The client's most recent serum potassium level is 2.9 mEq or mmol/L. BThe client has crackles in both lung bases. The client has crackles in both lung bases. CThe client has 4

pitting edema in both lower legs. The client has 4+ pitting edema in both lower legs. DThe client's most recent blood pressure is 96/60 mmHg. The client's most recent blood pressure is 96/60 mmHg. Question Explanation +Rationale Bumetanide is a powerful, potassium-wasting loop diuretic. It promotes diuresis in clients suffering from heart failure (HF) and fluid retention. Prior to administration, the nurse should verify that the client's potassium level is within normal range (3.5 to 5.0 mEq or mmol/L). A serum potassium level of 2.9 mEq or mmol/L is very low. The new graduate nurse should hold the bumetanide and notify the health care provider (HCP) immediately. Bibasilar crackles and pitting edema are expected findings for a client with HF and are indications for the use of diuretics. Although loop diuretics can cause hypotension related to diuresis, a BP of 96/60 is within acceptable limits for a client with HF.

820) When a client exhibits severe bradycardia, which type of medication will the nurse be prepared to administer? Nitrate Anticholinergic Antihypertensive Cardiac glycoside

rationale An anticholinergic medication will block parasympathetic effects, causing an increased heart rate. Nitrates will decrease preload, not increase the heart rate. Antihypertensive medications will lower the blood pressure and may decrease the heart rate. Cardiac glycoside will improve cardiac contractility but will decrease the heart rate.

908) Which action is likely to reduce the pancreatic and gastric secretions of a client with pancreatitis? Encourage clear liquids. Obtain a prescription for morphine. Assist the client into a semi-Fowler position. Administer prescribed anticholinergic medication.

rationale Anticholinergic medications block the neural impulses that stimulate pancreatic and gastric secretions. Oral fluids stimulate pancreatic secretion. Morphine sulfate is an analgesic and does not decrease gastric secretions. The semi-Fowler position decreases pressure against the diaphragm to help relieve discomfort, but it does not decrease pancreatic secretions. STUDY TIP: Enhance your time-management abilities by designing a study program that best suits your needs and current daily routines by considering issues such as the following: (1) amount of time needed; (2) amount of time available; (3) 'best' time to study; (4) time for emergencies and relaxation.

710) The anticholinesterase medication pyridostigmine is prescribed for the client with myasthenia gravis. When providing medication teaching, the nurse explains that the client should expect a decrease in which function? Bowel function Heart rate Skeletal muscle contraction Urinary frequency

rationale Anticholinesterase medications inactivate cholinesterase, the enzyme that mediates breakdown of acetylcholine. As a result of increased cholinergic activity, the heart rate will typically decrease. As a result of the increased cholinergic activity, the client can anticipate increased (not decreased) bowel movements and increased (not decreased) urinary frequency. Skeletal muscle contractions are increased (not decreased); this is the reason the medication is administered for myasthenia gravis.

950) During a procedure, the client's heart rate drops to 38 beats/min. Which medication is indicated to treat bradycardia? Digoxin Lidocaine Amiodarone Atropine sulfate

rationale Atropine blocks vagal stimulation of the sinoatrial (SA) node, resulting in an increased heart rate. Digoxin slows the heart rate; hence, it would not be indicated in this situation. Lidocaine decreases myocardial sensitivity and will not increase the heart rate. Amiodarone is an antidysrhythmic medication used for ventricular tachycardia; it will not stimulate the heart rate.

799) A client suspected of having myasthenia gravis is scheduled for an edrophonium chloride test. To treat a common complication associated with the test, the nurse will have which medication available? Atropine Phenytoin Neostigmine Diphenhydramine

rationale Atropine, an anticholinergic, should always be available to treat a cholinergic crisis (sudden, severe episode of muscle weakness that affects breathing and swallowing) should the edrophonium chloride test trigger this response. Phenytoin is an anticonvulsant that will not avert or treat complications resulting from an edrophonium test. Neostigmine is a cholinergic that has the same action as edrophonium chloride; it is contraindicated if a cholinergic crisis occurs. Diphenhydramine is an antihistamine that will not avert complications or effectively treat a cholinergic crisis.

638) A client with myasthenia gravis begins taking pyridostigmine. Two days later, the client develops loose stools and increased salivation. Which conclusion would the nurse make about these new developments? The client is experiencing a myasthenic crisis. The medication is causing cholinergic side effects. The medication is triggering a paradoxical reaction. The client is exhibiting toxic effects of the medication.

rationale Because this medication inhibits the destruction of acetylcholine, parasympathetic activity may increase, resulting in cholinergic side effects such as diarrhea and increased salivation. The signs do not indicate a myasthenic crisis. Myasthenic crisis is characterized by difficulty breathing or speaking, morning headaches, feeling tired during the daytime, waking up frequently at night, not sleeping well, a weak cough with increased secretions (mucus or saliva), an inability to clear secretions, a weak tongue, trouble swallowing or chewing, and weight loss. Side effects are not temporary and not paradoxical; they continue as long as the medication is continued. The dosage may be adjusted or an anticholinergic may be given to limit side effects. Toxicity or cholinergic crisis is manifested by increased muscle weakness, including muscles of respiration.

744) Cholinergic agonists are prescribed for which type of urinary condition? Kidney stones Urine retention Spastic bladder Urinary tract infections

rationale Cholinergics intensify and prolong the action of acetylcholine, which increases the tone in the genitourinary tract, preventing urinary retention. Cholinergics will not prevent renal calculi. Anticholinergics, not cholinergic agonists, are prescribed for the frequency and urgency associated with a spastic bladder. Preventing urinary tract infections is a secondary gain because cholinergics help prevent urinary retention that can lead to a urinary tract infection, but this is not the purpose for administering these medications.

668) The health care provider prescribes a cholinergic medication to treat a client's urinary problem. Which effect would the nurse anticipate? Urinary frequency decreases. Urinary retention is prevented. Pain is controlled. Urinary urgency decreases.

rationale Cholinergics intensify and prolong the action of acetylcholine, which increases tone in the genitourinary tract, preventing urinary retention. Cholinergic antagonists are prescribed for frequency and urgency associated with a spastic bladder; therefore cholinergics would worsen rather than decrease these problems. Cholinergic do not have analgesic effects.

604) The nurse is teaching a client about tricyclic antidepressants. Which potential side effects would the nurse include? Select all that apply. One, some, or all responses may be . Dry mouth Drowsiness Constipation Severe hypertension Orthostatic hypotension

rationale Dry mouth is a common anticholinergic side effect of tricyclic antidepressants. Drowsiness can be a common side effect but usually decreases with continued treatment. Constipation is a common side effect that usually can be managed with stool softeners and a high-fiber diet. Orthostatic hypotension is a common side effect of tricyclic antidepressants; the client should be instructed to rise slowly from a sitting to a standing position. Hypertension of any type is not a side effect of tricyclic antidepressants.

1052) A client with myasthenia gravis is receiving pyridostigmine bromide to control symptoms. Recently, the client has begun experiencing increased difficulty in swallowing. Which nursing action is effective in preventing aspiration of food? Place a tracheostomy set in the client's room. Assess respiratory status after meals. Request for the diet to be changed from soft to clear liquids. Coordinate mealtimes with the peak effect of the medication.

rationale Dysphagia should be minimized during peak effect of pyridostigmine bromide, thereby decreasing the probability of aspiration. A tracheostomy set is a treatment for, rather than equipment to prevent, aspiration. Although it is vital that the client's respiratory function be monitored, assessing the client's respiratory status will not prevent aspiration. There are insufficient data to determine whether changing the diet from soft foods to clear liquids is appropriate; also, liquids are aspirated more easily than semisolids.

586) The nurse is assessing an adolescent after the administration of epinephrine. Which side effect is most important for the nurse to identify? Tachycardia Hypoglycemia Constricted pupils Decreased blood pressure

rationale Epinephrine is a sympathetic nervous system stimulant that causes tachycardia. Hyperglycemia, not hypoglycemia, may result. The pupils will be dilated, not constricted. Epinephrine is more likely to cause hypertension than hypotension.

1494) A 12-year-old child has just received a dose of epinephrine. Which assessment is the priority after this medication is administered? Heart rate Glucose level Neurological status Blood pressure

rationale Epinephrine is a sympathetic nervous system stimulant that may cause significant tachycardia, so assessment of heart rate is the priority. Assessing for hyperglycemia (glucose level), pupil dilatation (neurological status), and hypertension (blood pressure) are all assessments that the nurse would also perform, but these would be done after assessing heart rate for tachycardia.

1485) Which side effect would the nurse assess for after epinephrine is administered to an infant with severe bronchospasms? Tachycardia Hypotension Respiratory arrest Central nervous system depression

rationale Epinephrine stimulates beta- and alpha-receptors; its actions include increasing heart rate and blood pressure and inducing bronchodilation. Increased blood pressure, not hypotension, is a potential side effect. Epinephrine relieves respiratory problems; it does not cause respiratory arrest. Epinephrine stimulates, not depresses, the central nervous system.

478) Which instruction would the nurse give to clients prescribed psychotropic medications who are experiencing anticholinergic-like side effects? 'Restrict fluid intake.' 'Eat a diet high in carbohydrates.' 'Suck on sugar-free hard candies.' 'Avoid products that contain aspirin.'

rationale Hard candy may produce salivation, which helps alleviate the anticholinergic-like side effect of dry mouth that is experienced with some psychotropics. Dry mouth increases the risk for cavities; candy with sugar adds to this risk. Fluids should be encouraged, not discouraged; fluids may alleviate dry mouth. Eating a diet high in carbohydrates and avoiding aspirin are unnecessary. Test-Taking Tip: What happens if you find yourself in a slump over the examination? Take a time-out to refocus and reenergize! Talk to friends and family who support your efforts in achieving one of your major accomplishments in life. This effort will help you regain confidence in yourself and get you back on track toward the realization of your long-anticipated goal.

773) The health care provider prescribes neostigmine for a client with myasthenia gravis. Which client statement indicates understanding regarding medication management plans? 'I must keep the medication in a container in the refrigerator.' 'I should take the medication at the exact time that is listed on the prescription.' 'I will plan to take the medication between meals.' 'I expect that the onset of the medication's action will occur several hours after I take it.'

rationale Neostigmine should be taken as prescribed, usually before meals, to limit dysphagia and possible aspiration. Keeping neostigmine refrigerated is not necessary; it may be kept at room temperature. Neostigmine should be taken with milk to prevent gastrointestinal irritation; usually it is taken about 30 minutes before meals. The onset of the action of neostigmine occurs 45 to 75 minutes after administration; the duration of its action is 2.5 to 4 hours.

1156) A client with myasthenia gravis has been receiving neostigmine and asks about its action. Which information would the nurse consider when formulating a response? Stimulates the cerebral cortex Blocks the action of cholinesterase Replaces deficient neurotransmitters Accelerates transmission along neural sheaths

rationale Neostigmine, an anticholinesterase, inhibits the breakdown of acetylcholine, thus prolonging neurotransmission. Neostigmine's action is at the myoneural junction, not the cerebral cortex. Neostigmine prevents neurotransmitter breakdown, but it is not a neurotransmitter. Neostigmine's action is at the myoneural junction, not the sheath.

1016) Which body system is affected by excessive use of amphetamine/dextroamphetamine? Renal Cardiac Musculoskeletal Gastrointestinal

rationale Norepinephrine is released when amphetamine/dextroamphetamine is taken. Excessive doses of amphetamine/dextroamphetamine can cause an increase in heart rate and dysrhythmias. Amphetamine/dextroamphetamine overuse does not directly affect the renal, musculoskeletal, or the gastrointestinal systems. Test-Taking Tip: What happens if you find yourself in a slump over the examination? Take a time-out to refocus and reenergize! Talk to friends and family who support your efforts in achieving one of your major accomplishments in life. This effort will help you regain confidence in yourself and get you back on track toward the realization of your long-anticipated goal.

423) The nurse is teaching a nursing student about tricyclic antidepressant medications. Which statement made by the student indicates the need for further teaching? Select all that apply. One, some, or all responses may be . Nortriptyline is contraindicated in older adult clients. Desipramine is preferred for use in older adult clients. Imipramine is used as an adjunct in the treatment of childhood enuresis. Tricyclic antidepressant medications are prescribed for clients with seizure disorders. Tricyclic antidepressant medications are contraindicated in clients with a history of seizures.

rationale Nortriptyline is a preferred tricyclic antidepressant that can be administered safely in older adult clients. Antiepileptic medications are prescribed to clients with seizures. Desipramine and nortriptyline are preferred tricyclic antidepressants for use in older adult clients. Childhood enuresis necessitates the administration of imipramine. Clients with epilepsy should not be prescribed tricyclic antidepressants to avoid the risk of medical complications. Test-Taking Tip: Multiple-choice questions can be challenging because students think that they will recognize the right answer when they see it or that the right answer will somehow stand out from the other choices. This is a dangerous misconception. The more carefully the question is constructed, the more each of the choices will seem like the response.

1053) Pyridostigmine is prescribed for a client with myasthenia gravis. Why would the nurse instruct the client to take pyridostigmine about 1 hour before meals? This timing limits first pass metabolism. Taking it on an empty stomach increases absorption. Taking it before meals decreases gastric irritation. Taking it before meals improves the ability to chew.

rationale Peak action of the medication will occur during meals to promote chewing and swallowing and prevent aspiration. It should be given with a small amount of food to prevent gastric irritation. First pass metabolism is a process of metabolism, which is not affected by medication timing. Absorption is not affected significantly by the presence of food in the stomach. Gastric irritation is reduced best by the administration of medications with food, not on an empty stomach.

1504) Pilocarpine is used as part of the diagnostic process of a toddler suspected of cystic fibrosis. The nurse knows that the pilocarpine would stimulate which process? Secretion of mucus Activity of sweat glands Excretion of pancreatic enzymes Release of bile from the gallbladder

rationale Pilocarpine is a cholinergic that is applied to the skin to stimulate sweat production; the sweat is then tested to confirm the diagnosis of cystic fibrosis. Pilocarpine does not stimulate the secretion of mucus, the excretion of pancreatic enzymes, or the release of bile from the gallbladder.

802) Pyridostigmine bromide is prescribed for a client with myasthenia gravis. The nurse evaluates that the medication regimen is understood when the client makes which statement? 'I will take the medication on an empty stomach.' 'I need to set an alarm so I take the medication on time.' 'It will be important to check my heart rate before taking the medication.' 'I should monitor for an increase in blood pressure after taking the medication.'

rationale Pyridostigmine is a vital medication that must be taken on time; a missed or late dose can result in severe respiratory and neuromuscular consequences or even death. Pyridostigmine should be taken with a small amount of food to prevent gastric irritation. It is unnecessary to take the pulse rate before taking pyridostigmine. Pyridostigmine may cause hypotension, not hypertension, which is a sign of cholinergic crisis. Test-Taking Tip: If the question asks for an immediate action or response, all the answers may be , so base your selection on identified priorities for action.

896) A client presents to the clinic for a follow-up appointment after starting pyridostigmine for management of myasthenia gravis. Which new client problems are adverse effects of pyridostigmine? Select all that apply. One, some, or all responses may be . Respiratory depression Increased urinary frequency Diplopia Muscle twitching Diarrhea

rationale Pyridostigmine is an acetylcholinesterase inhibitor. By inhibiting the enzyme that breaks down acetylcholine, it increases cholinergic activity. The increased cholinergic activity is responsible for the most common adverse effects of pyridostigmine such as diarrhea and increased urinary frequency. Another adverse effect is muscle twitching that occurs as pyridostigmine affects the muscle weakness that is characteristic of myasthenia gravis. Respiratory depression is a clinical manifestation of myasthenia gravis that can occur due to respiratory muscle weakness. Weakness of eye muscles results in ptosis or diplopia in about half of clients with myasthenia gravis.

1150) A client is undergoing diagnostic testing for myasthenia gravis. Which test would the nurse identify as the most specific for this diagnosis? Electromyography Pyridostigmine test Edrophonium chloride test History of physical deterioration

rationale The edrophonium chloride test uses a medication that is a cholinergic and an anticholinesterase; it blocks the action of cholinesterase at the myoneural junction and inhibits the destruction of acetylcholine. Its action of increasing muscle strength is immediate for a short time. The results of electromyography will be added to the database, but they are nonspecific. Pyridostigmine is a slower-acting anticholinesterase medication that is commonly prescribed to treat myasthenia gravis; edrophonium chloride is used instead of pyridostigmine to diagnose myasthenia gravis because, when injected intravenously, it immediately increases muscle strength for a short time. The results of a history and physical are added to the database, but the data collected are not as definitive as another specific test for the diagnosis of myasthenia gravis.

1151) A client begins treatment with pyridostigmine bromide therapy for myasthenia gravis. Which action would the nurse perform in administration of the medication? Administer the medication after meals. Administer the medication on an empty stomach. Evaluate the client's psychological responses between medication doses. Evaluate the client's muscle strength every hour after the medication is given.

rationale The onset of action of pyridostigmine is 30 to 45 minutes after administration, and the effects last up to 6 hours; the client's response will influence dosage levels. Pyridostigmine usually is administered before meals to promote mastication. Pyridostigmine should be administered with food to prevent gastric irritation. There are no psychological side effects associated with pyridostigmine.

Question 7 The nurse is caring for a client who has been prescribed atropine preoperatively. The nurse understands the intended purpose for administering this preoperatively is to induce which effect? Question 7 Answer Choices AReduce heart rate BElevate blood pressure CEnhance sedation DDecrease secretions Question Explanation

rationale: Atropine is a common anesthesia adjunct. It decreases the amount of secretions which, in turn, decreases the risk of aspiration during the operative procedure.

Question 19 The nurse in a urology office is developing a plan of care for a client newly diagnosed with urge urinary incontinence due to an overactive bladder. Which interventions should the nurse include? AAdminister ant-seizure medications BAdministration of cholinergic drugs CAdministration of anticholinergic drugs DAdministration of loop diuretics Question Explanation

rationale: Incontinence is an involuntary loss of urine severe enough to cause social or hygienic problems. It is not a normal consequence of aging or childbirth and can have several possible causes and can be either temporary or chronic. Urge incontinence is often referred to as overactive bladder (OAB). Nocturnal frequency and incontinence are common with OAB. Interventions include treatment with drugs that relax the smooth muscle and increase the bladder's capacity including anticholinergics, not anti-seizure, loop diuretics or cholinergic drugs.

Question 6 The nurse is teaching a client diagnosed with depression about a new prescription of nortriptyline. What information would be essential for the nurse to emphasize about this medication? Question 6 Answer Choices AEpisodes of diarrhea can be expected BThe medication must be stored in the refrigerator CThe use of alcohol should be avoided DSymptom relief occurs in a few days Question Explanation

rationale: Nortriptyline is a tricyclic antidepressant used to manage chronic neurogenic pain and depression. Adverse reactions include central nervous system (CNS) side effects such as suicidal thoughts, drowsiness, fatigue, lethargy, and confusion. Clients who are prescribed this medication should be educated to avoid the use of alcohol consumption or other CNS depressant drugs as this can worsen the adverse reactions of the medication and cause injury.

Question 14 The nurse has administered fentanyl, atropine, cefazolin and benzocaine to a client for an endoscopic procedure. The nurse is monitoring the client and notes that the heart rate has increased from the pre-procedure baseline. The nurse knows that which of the following medications is most likely responsible for the client's increased heart rate? Question 14 Answer Choices AFentanyl BAtropine CCefazolin DBenzocaine Question Explanation

rationale: Procedural sedation is used in endoscopic procedures as an effective way to provide an appropriate degree of pain and anxiety control; memory loss; and decreased awareness. The most commonly used medication regimen for gastrointestinal endoscopic procedure is still the combination of benzodiazepines, opioids, anticholinergics and topical anesthetics. Atropine is an anticholinergic drug that is used to dry secretions during the procedure. However, it can also increase the heart rate and dilate the pupils and is the most likely cause for the increased heart rate. Fentanyl is an opioid analgesic and short-term central nervous system (CNS) depressant and tends to slow breathing and lower heart rate and blood pressure. Benzocaine is a topical anesthetic and cefazolin is an antibiotic; neither should affect the heart rate.

Question 7 To which nursing home resident could a nurse safely administer tricyclic antidepressants without questioning the health care provider's order? Question 7 Answer Choices AA client with mild hypertension BA client with narrow-angle glaucoma CA client with coronary artery disease (CAD) DA client with benign prostatic hypertrophy (BPH) Question Explanation

rationale: Tricyclics can be safely administered to the hypertensive client. The expected anticholinergic effects of tricyclic antidepressants include difficulty in urination, which is why TCAs are contraindicated with BPH. TCAs are also contraindicated in narrow-angle glaucoma (they can cause elevated pressure in the eyes) and for certain heart abnormalities.


Ensembles d'études connexes

pt 1. care of patients with immune hypersensitivity reaction

View Set

Career Counseling QUIZ ANSWERS - CH 2

View Set

Geometry Segments Proofs Reference

View Set

Vocabulary from Classical Roots D Chapter 15 & 16

View Set

Midterm #2: Roman History p.15-19 (STUDY GUIDE NOTES)

View Set